Unz评论•另类媒体选择$
美国主流媒体大都排除了有趣,重要和有争议的观点
 博客浏览詹姆斯·汤普森档案馆
不断发展的情报

书签 全部切换变革理论添加到图书馆从图书馆中删除 • B
显示评论下一个新评论下一个新回复了解更多
回复同意/不同意/等等 更多... This Commenter This Thread Hide Thread Display All Comments
同意不同意谢谢LOL轮唱
这些按钮可将您的公开协议,异议,感谢,LOL或巨魔与所选注释一起注册。 仅对最近使用“记住我的信息”复选框保存姓名和电子邮件的频繁评论者可用,并且在任何八个小时的时间内也只能使用三次。
忽略评论者 关注评论者
搜寻文字 区分大小写  确切的词  包括评论
列表 书签

生物群落年平均温度

如果人是庄稼,他们最适合种植在哪里?

像许多人一样,我读了一些使我误入歧途的书。 它们是合理的,尽管我可以看到它们中的错误,但我继续阅读以学习新事物。 我愿意接受作者在某些事情上可能是错误的,而在另一些方面是正确的。 这是一个程度的问题。 对Baloney的检测并不那么困难,并且必须将某些书放在一边,以免它们胡说八道。

至于《枪炮、病菌和钢铁》,我看得出作者对情报的看法是错误的,是故意错的,但我心痛地理解,很多公众人物在这种事情上撒谎是政治性的,我愿意继续阅读以了解了解地理和动植物,我知道的太少了。 卡瓦利-斯福尔扎(Cavalli-Sforza)还发现在“基因,民族和语言”的前几页中解散情报是有政治意义的,然后无论如何在书的其余部分中都给出了他的结果。 也许他知道大多数人只阅读前25页。 许多喜欢贬低智力的心理学研究者也是如此,尤其是在群体差异方面。 他们的秘密对我来说是安全的。 我不希望看到它们困扰,只是因为它们使水变得浑浊,并使点不相连。 活下去,活下去。

地理学作为一门学科已经不那么受欢迎了。 也许这是由于人们认为地理已经掌握了,所以他们的家园晚了,现在可以超越它,进入一个完全安全,隔热良好的空调世界。 这种过度自信可能导致居住在洪泛区的人们在被洪水淹没时感到惊讶。

但是地理学是关于什么使某些国家比其他国家更富裕,组织更好和更令人愉快的辩论中的共同主题。 贾里德·戴蒙德(Jared Diamond)在书中将这些差异归咎于地理因素,认为人人都是一样,但必须应对不同的情况。 (实际上,在没有任何支持证据的情况下,他宣布巴布亚新几内亚的居民是最聪明的人,也许是在逆境中寻求美德)。 相反的立场是一种更长远的观点,即世代相传(至少有 16 人?),人们逐渐适应了环境,并通过自然选择改变了他们的性格和能力。

在这一点上,研究人员通常将纬度视为地理影响的指标。 与赤道的距离可以很好地预测结果。 是否可以做得比这更好,并包括其他相关措施以使人类类型与其原籍地区之间最合适? 最后一个问题就是复杂性。 人们最初是在船上使用,现在又在宽体喷气机中以更快,更便宜的价格来回走动。 实际上,回顾历史,移徙并不是什么新鲜事物,尽管在现代运输之前花费了更长的时间。 应该有可能将遗传群体追溯到其起源地。 卡瓦利-斯福尔扎(Cavalli-Sforza)要求他的臣民展示可追溯至1492年的定居住宅,如果我们每个世纪估计有21代的话,那只是4代前。 当然,使用遗传方法,我们应该能够做得更好。 但是,下面要考虑的工作并不涉及该特定问题。 相反,它试图创建可能与智力有关的生物群落类型学。 这是一项正在进行中的工作,最初是会议介绍(请参阅下面的链接),带有一组解释性注释,其中列出了一些论点以及仍需解决的问题。 它探讨了一些选项,并可能为更好地讨论哪些环境有利于智能发展提供了一种更好的方法。

使用生物群系测绘和权重更精确地预测智能方面的生物地理差异
史蒂文·赫特勒(Steven C.

纬度和年平均气温有力地预测了情报的生物地理分布。 作为一个单独的指标,纬度和年平均温度彼此之间只有一个竞争者。 当然,它们之间是高度相关的,具有明显的因果关系。 实际上,由于年平均气温具有更明确的综合性质,因此它实际上可能优于纬度。 如果以复杂的方式测量多个测量点的年平均温度,然后合并在一起,则尤其如此。 如果这样做,年平均温度可以隐含地解释海洋变暖趋势以及高海拔草原和山脉。 但是,还有其他物理生态成分无法解释或无法充分说明年平均温度。 一个例子是水分或水文学。 例如,撒哈拉和撒哈拉以南非洲国家的年平均温度变化不大,即使可用水量相反。 另一个例子是土壤质量。 在宽广的纬度范围内都可以发现Insoisols(早期土壤形成)和其他土壤类型,因此它们以各种年均温度存在。 尽管如此,诸如气候和土壤类型之类的气候因素仍然在平均年温度的预测能力范围之外,对人类的认知进化具有重要意义。

为了改进气候的元指标,在这种情况下是纬度,菲格雷多及其同事包括了温带阔叶落叶林因子。 结果证明这是一个强大的复合预测变量,因为温带阔叶落叶林仅存在于某些参数内; 具体来说,是在特定温度值范围内,北纬40至60度之间,北边是多年冻土,南边是竞争的针叶树发。 它们的存在还表示中等湿度和丰富的棕色铝溶胶(通常在硬木森林覆盖下),可能特别有利于早期农业增产,同时阻止蠕虫内寄生物(蠕虫)。 温带阔叶落叶林生物群落的存在似乎对于某些具有较高偏远情报的国家而言特别有价值。 尽管有这些观察结果,但大多数地区或国家都不存在温带森林。 因此,本研究使用了世界野生动物联合会提供的全球生物群落图,将温带阔叶落叶林因子扩展为可以广泛应用的生物群落分类系统,可以指导各种选择制度。 我们通过与认知进化的假设关系来介绍通常排名较高的生物群落。 在某种程度上,分析了跨国情报的当前分布,根据移民,人口异质性和预测的移民路线对其进行了分析。 这种方法的启发式价值令人信服。 在直接测量社区生态的同时,可以更好地预测物理生态。

此时,您可能会选择查看会议幻灯片,然后返回笔记。
https://drive.google.com/open?id=0B3c4TxciNeJZTVUzWGtZanZDazg

可以根据肥力差异对土壤进行排名,并且在世界不同地区,土壤差异很大。

生物群落土壤

Hertler引用了Walter&Breckle(1999)的以下文章,这些文章说明了有关他的生物群落模型的研究成果。

尽管热带雨林土壤很老,通常很深,但它们的类型为草索酚和utisol,覆盖率约为三分之二。 氧化溶胶和乌索托尔仅具有中等至低的生育力。 而且,热带地区的百分之七是由“石英砂丰富的冲积层”组成的,它被风化和浸水,因此营养不足。 因此,“当今的农业实践只能在大约20%的热带土壤上进行,即较年轻的火山土壤(浮石)和大河谷中丰富的冲积土壤……”

“土壤中的养分通常极度贫瘠,并且是酸性物质(pH = 4.5-5.5),乍一看似乎与植被茂盛相矛盾。 实际上,森林所需的几乎所有养分储备都包含在地上植物中。 每年,这种植物的一部分会死亡并迅速矿化,从而释放出的营养元素立即被根部吸收。 尽管降雨很高,但几乎没有因淋洗而损失养分。 溪流中的水具有蒸馏水的电导率,至多被腐殖质细菌染成浅褐色”

这解释了悖论。 它可与土地财富相提并论……没有流动资本(可利用的地面养分并存储在土壤中),而仅是固定资本(体现在树木中,因此不可用于增长)。 白蚁和其他生物的快速循环,然后快速再吸收保持矿物质被隔离; 但关键是,砍伐森林会导致迅速腐烂,然后水浸,使土壤不适合持续农业。 相反,温带阔叶落叶林的分解速度较慢,其分解原因是越来越少的只能在半个季节工作的特殊分解剂,例如缓慢释放的药物……将养分缓慢地循环回土壤。

如此迅速的养分循环,雨林可以在同一地点生长数千年,但是一旦砍伐森林并烧毁木材,就会突然浸出矿化的营养储备。 土壤细菌只吸收一小部分,栽培植物可以利用几年。 如果停止耕种,就会形成次生林; 然而,这些森林从未达到原始森林的茂盛习性。 如果再将其清除以进行临时栽培,则由于浸提会导致营养素的新鲜流失,直到一系列此类开采之后,土壤仅能支撑蕨菜。 在这些地区被烧毁后,营养需求有限的草,例如白茅(白茅),可以站稳脚跟。 这些草没有放牧价值。

“在贫瘠的土壤中生长的热带雨林对人类住区来说是荒凉的,通常是人们所避免的。 它通常是原住民部落的避难所。 在非洲,这些是the格米人,在拉丁美洲,是原始的印第安部落。 即使在东南亚,也有一些原始部落幸存下来。 与这些栖息地相反,营养丰富的年轻火山土壤上的前雨林遗迹是人口稠密的耕地(例如,爪哇,中美洲)。 只有在这些地区,才有可能维持长期农业。 在所有其他地区砍伐树木会导致营养的灾难性损失,这非常清楚地表达了热带地区的“生态”劣势(Weischet 1980)。 数年后,森林砍伐的地区一文不值,因为它们容易受到侵蚀,很快就会被价值不菲的Gleichenia或Imparata灌木丛覆盖。

更有趣的发现之一是热带土壤的贫瘠程度。 我以为它们很丰富,但它们的养分却被隔离在生物质中,然后在砍伐和燃烧时迅速流失。 相反,温带森林保持着不断循环的叶片所产生的丰富而深厚的营养层。 即使在森林被砍伐之后,这也提供了数百年的肥沃价值。

实际上,某些土壤在耕种庄稼和人类方面比其他土壤更好。

关于日本地图所暗示的生物群系/智能联系,赫特勒解释说:

我希望看到更完整的日本智商地图。 我在这张不完整的作品中看到右图上红色为红色的低智商。 这属于与理论相符的热带地区。 通常,我会认为少数几个热带地区以及最北端的岛屿的智商都低于中部地区。 在某种程度上,这是由于不同民族的历史分布(我记得北部岛屿被遗传上不同的人所占据)。 但是,这在一定程度上是由于情报和高K族人争夺并占领了最理想的气候区以及与他们的文明和祖先进化的进化条件相称的气候区。 有哪些数据表明在TBDF温带阔叶落叶林地带的智商较高。

关于生物群系分类的理论依据,在生物群系分类中,按鼓励智力增长的能力对生物群系进行排名,赫特勒承认这是临时性的和理论性的,而不是系统性的或经验性的。 (他对加权系统有更详尽的解释)。

生物群系 14

如表3所示,这些生物群系通常通过与认知进化的假设关系从1991到1995排名。 这种排序的首要基础是对寒冷冬季理论的改编。 根据寒冷的冬季理论(Lynn,2015; Rushton,2016),高认知能力的演变源于迁徙后在欧亚气候中暴露于极端寒冷中。 工具使用、庇护所建设和大型合作狩猎的新高度是理论上推动认知收益的认知需求必需品。 后来对寒冷冬季理论的使用更加强调季节性(Hertler,2105 年;XNUMX 年)。 根据这个理论,在允许的温暖和严寒之间的可预测的循环需要更大的责任心。 尽责性带有认知内涵,因为尽责性本质上是面向未来的,并且表现出更多的自我控制能力。 人们更有可能让现在服从于未来,有人可能会说,在北方环境中的人有更多的执行控制和执行控制的行为相关性,我们通常称之为人格文学中的尽责性 (Hertler, XNUMX)。

根据这些观察,在本文中预测智力和寒冷之间的关系是曲线的。 因此,最重要的是温带生物群落:温带阔叶混交林(TBMF),温带针叶林(TCF),地中海森林,林地与灌木丛(MFWS)以及温带草原,热带稀树草原和灌木丛(TGSS) 。 根据此曲线预测,这四个生物群落被有意地置于北方森林/塔伊加(BFT)和苔原(T)生物群落之上。 然而,就本分析而言,这一点或多或少是有争议的,因为该数据集不包括北方森林/冻土带,而苔原仅占中国领土的一小部分。 另一方面,一般来讲,气候温暖。 这些在赤道地区占主导地位。 寒冷变得越来越少,季节性(如果存在的话)通常更多地与水的可利用性有关,而与温度波动无关。 实际上,最明显地将这些亚热带和热带气候区分开是水的可利用性。 例如,湿热的结合产生了热带和亚热带湿阔叶林(TSMBF),按照寒冷的冬天理论,这对智力的发展没有积极的推动作用,但它也带来了丰富的生物活性,其中很大一部分是有害的对人类的生活。 在此之后,沙漠和干燥灌木丛(DXS)发挥热量的作用,但不会使热量与过多的水供应混合在一起。

新加坡是一个离群值。 新加坡的人口统计数据使其成为排除的主要候选对象,这不是因为它是一个极端的阳性值,几乎是下一个阳性异常值的两倍,而是因为在1800年代有大量的中国人涌入,目前约占百分之七十占总人口的百分比。 因此,尽管新加坡位于赤道上,但那里没有进化出该地区的人。 当然,在其他情况下,迁移会显着影响生物群落分析的预测能力,但这些极端性较低的情况将留给其他方法进行事后分析。

对于拥有多个生物群落的大国(例如南非),尤其是中国,平均生物群落可能会遇到问题。 例如,中国人口位于温带生物群落的东海岸,尽管其内部很大一部分是沙漠。

总之,只要地理学家愿意同意特定生物群落的长期居住可能导致进化变化,那么关于地理影响人类及其文明发展的论点就不必在遗传论者和环境保护主义者之间争论。几代人。 新加坡,而不是例外,可能是规则的体现。

https://drive.google.com/open?id=0B3c4TxciNeJZTVUzWGtZanZDazg

 
• 类别: 科学 •标签: 遗传多样性, 地理, IQ 
隐藏423条评论发表评论
忽略评论者...跟随Endorsed Only
修剪评论?
    []
  1. 只要环保主义者愿意同意,在特定生物群落中的长期居住可能会导致几代人的进化变化。

    我不明白为什么这很难同意。 这只是“常识/知识”。

    实际上,某些土壤在耕种庄稼和人类方面比其他土壤更好。

    这对我来说并不奇怪。 这也是非洲营养不良的另一个原因。

    “生长在贫瘠土壤中的热带雨林不适合人类居住,通常被人们避开。 它通常是原住民部落的避难所。 在非洲,这些人是俾格米人,在拉丁美洲,他们是原始印第安部落。

    而这些民族是在缺乏能源的地方进化的,因此他们的体型反映了他们的进化历史(参见 Daniel Lieberman 2013 年的书 人体的故事 获取更多信息)。

  2. 新加坡非但不是例外,反而可能成为规则的体现。

    新加坡不能代表任何其他东西,但事实是,位于世界上最繁忙的海上贸易路线上的避税天堂城邦必须是一个成功的国家。

    但是:
    -新加坡不是世界上最富有的城邦
    -不是世界上最富有的税收天堂
    -不是最富有的贸易中心。

    夏威夷是一个更好的证明,因为夏威夷的大部分人口都是日本苦力的后裔。 夏威夷绝对不是新加坡,甚至相差甚远。

    此外,疾病负担与智商的相关性比纬度或温度的相关性要好得多。

    您显示的生物群落排名仅反映了宜居性(疾病负担、土壤肥力、温度、降水……),并且与世界人口密度的分布密切相关。

    如果没有进化论的故事,这一切都是有道理的。

    • 回复: @Anonymous
  3. 如果人是庄稼,他们最适合种植在哪里?


    或者在现代医学普及之前

    • 回复: @Afrosapiens
  4. @Afrosapiens

    既然我们谈论的是农作物

    农业适宜性:

    土壤肥力:

  5. 对于拥有多个生物群落的大国(例如南非),尤其是中国,平均生物群落可能会遇到问题。 例如,中国人口位于温带生物群落的东海岸,尽管其内部很大一部分是沙漠。

    哈哈,看看我对沙漠环境的排名有多高,我想那些头脑简单的作者将中东和北非国家算作沙漠。 那就是迟钝了。

    -马格里布大部分是沙漠,但人口集中在享有地中海气候的沿海地区,黎凡特也是如此。

    -埃及、苏丹以及美索不达米亚大部分地区都是沙漠,但人口居住在尼罗河、幼发拉底河和底格里斯河沿岸,这些都是凉爽的湿地

    - 阿拉伯大部分地区都是沙漠,但大多数人口居住在汉志和也门的高地。 此外,海湾国家还有大量非阿拉伯移民。

    这和往常一样是垃圾研究。

  6. Mactoul 说:

    如果人是庄稼,他们最适合种植在哪里?

    如果要对数量(即人口)进行优化,那么中国和印度是历史上和目前最好的。
    如果针对质量(即智商)进行优化,那么温带地区会获胜。

  7. Someone 说:

    良好的土壤 > 农业 > 久坐的生活 > 人口稠密 > 较高程度的离群智商(较低和较高的智商) > 选择。
    但是,在后冰河时代的三种生活方式中,狩猎/采集、农业和畜牧业,畜牧业在不同的立场上也取得了令人难以置信的成功,甚至比农业更成功或处于同等/竞争状态。 为什么? 好吧,人们开始推测,从商品和商业角度来看,畜牧业是一种更丰富的生活方式,这就是为什么采用这种生活方式的人与农学家一样成功、聪明等。
    这不会破坏地理模型吗?

    现实是:选择就是一切,人们的生活方式(基于、影响或他们所居住的地理/环境的任何影响)驱动着群体的选择——偏爱智力、对其保持中立或忽视它。

    每个环游世界、养过宠物、孩子、在“多元化”的地方工作过的人都可以完美地指出,遗传学就是一切。
    但是,为什么这些遗传学最终会变成这样呢? 必定有某种外部因素引导它发展到今天的样子——一个缓慢的、多代人的过程。
    那么,是的,外部性对于不同的选择很重要,但它们对于已经选择的内容也很重要? 说吧,你? 你是什​​么? 不。
    它们对后代很重要,他们可能会或可能不会携带“你”。
    我不知道我是否说清楚了,但就是这样。

    • 同意: CanSpeccy
    • 不同意: Afrosapiens
  8. “作为一门学科,地理学已经不再那么受欢迎了。”

    恕我直言,其中一些原因是“人文地理学”成为课程中更重要的部分。 现在显然土地利用和人类影响(河流疏浚/堤防、荒漠化、森林砍伐等)理所当然地属于其中。 但是 2016 年 XNUMX 月的 A level 试卷呢?

    选项 6 – 当代冲突和挑战

    图 6a 显示了克里米亚在其区域环境中的位置。 图 6b 提供了截至 2014 年有关克里米亚的一些关键信息。

    使用图6a和6b对克里米亚冲突进行评论。[7分]

    描述与您研究过的一场国际冲突相关的环境和经济问题。[8 分]

    讨论分离主义的原因和后果。[10 分]

    讨论与英国多元文化社会发展相关的问题。[40 分]

    3 (b)针对第 6 项中提出的问题,讨论如何改进水力压裂的监管。[10 分]

    (图 6b 中缺少真正重要的地缘政治信息,即克里米亚是黑海舰队的基地)。 带回牛弓湖和大陆性气候!

    好消息是地质学几乎没有受到这种趋势的影响。

  9. m___ 说:

    巨大的努力,远远超出了我最初的肤浅理解。

    不过,有一点是很重要的,虽然不是专门针对这篇文章的,但它的范围很广泛。

    “个体之间智商的差异对于群体差异来说是很大的。 过去两百年来,更加复杂、充满活力的社会将首先在挖掘个人智商天赋方面找到价值和针对性。

    行星上的群体差异将不再产生同样的相关影响,当然不会在现代时间尺度上,几十年,而不是几个世纪和几十代人。

    所有其他因素都将首先遵守这些卓越的智商‘补偿’。”

    这项令人印象深刻的研究可能仅具有历史价值,这本身当然是进一步理解的一层,但结果可能不会指向未来。 否则的话,作为预测工具它可能会失败。

  10. 为什么乌兹别克斯坦对这种情报材料如此狂热? 这完全没有任何科学价值,一切都是为了证实一个信念。 Ir 与生育论同等重要。

    • 回复: @Anon
    , @Santoculto
    , @Anon
  11. JimH 说:

    我可以在很多事情上看到北半球常见的偏见。 事实上,除了欧亚大陆之外,还有很多地方都有寒冷的冬天和/或无法种植任何东西的冬天。 新西兰下雪,南部非洲一年中有 6 个月没有下雨(XNUMX 月至 XNUMX 月),阿根廷两者都有; 然而当地人却从来没有想到过轮子。 南美洲和澳大利亚甚至没有铁。

    除此之外,技术和政治体系都是在干燥的亚热带(肥沃的新月区)或地中海发展起来的。

    这是基因的问题,笨蛋!

  12. Santoculto 说:

    极端气候不仅会增加,还会限制……。 创造力,我们真正谈论的是什么,[智力的增加也可以提高更高的创造力]。 因此,太干燥、太冷或太热,很可能会成为提高智力[和创造力]的挑战,为了赢得环境,这种恶劣的条件将迫使生物最大限度地适应=表型变异性和超性的减少; 增加务实的适应性方法。

    也许不是:

    人类暴露在寒冷地区并变得更聪明,但首先,他们是如何到达那里的*在向极地地区的扩张过程中,人类变得更聪明/又名自我控制,而当这种扩张受到恶劣气候的限制时,他们停止增加智力。

    比较气候/生物质与性格类型的适合程度似乎很有趣。 “赤道型人格”...

    语法不好,
    还在呼吸……

  13. @JimH

    我不确定印加人的情况,但玛雅人不使用轮子进行运输,因为这不利于他们的城邦布局。 然而,他们知道轮子及其用途,因为他们制作了带有轮子的儿童玩具。 还值得注意的是,玛雅人在没有受到外界影响的情况下也使用了“0”的概念。

    值得注意的是,文明在一些地方独立出现,玛雅人就是其中之一。 文字也只是在五个地区发明的,没有受到影响:中美洲(玛雅人)、中国人、苏美尔人、哈拉潘人和埃及人。

    所以,是的,由于环境/城市布局,人们可以理解某种事物的概念,而无需在社会中使用它。 我不会将其归因于基因。 因为如果你只是在事物发明的地方继续前进,你就会过得很糟糕。

    除此之外,技术和政治体系都是在干燥的亚热带(肥沃的新月区)或地中海发展起来的。

    这就是为什么文明只出现在气候适宜、可以通过粮食生产维持大量人口的地区。 像这样的领域是在没有任何外界影响的情况下独立发明文字的地方。

    • 回复: @Afrosapiens
    , @Jim
  14. Desert Fox 说:

    政府正在向美国和世界各地的天空喷洒有毒化学物质,称为“化学凝灰”,这种喷雾由纳米铝、钡和锶等组成,进入我们呼吸的空气、我们种植农作物的土壤和土壤中。我们喝的水。

    这是对类固醇的天气控制,政府正在使用 HAARP 和激光来加热大气并引导风暴,即飓风和龙卷风,他们最近在德克萨斯州、佛罗里达州和波多黎各所做的事情,他们已经这样做了几十年,但是最近,他们全力以赴地实施天气控制议程,以实现联合国 21 世纪议程中的全球变暖和美国去工业化的目标。

    化学尾迹中的铝对人类有毒,并进入我们生活的各个方面,即空气、水和土壤,毒害着这些物质以及我们地球上的人们。

    有关详细信息,请参阅地球工程 守望者组织

    • 回复: @RaceRealist88
  15. Hu Mi Yu 说:
    @JimH

    (阿根廷)当地人从来没有想到过轮子。 南美洲和澳大利亚甚至没有铁。

    如果没有道路和相当平坦的地形,车轮就没有多大用处。 南美洲开发的粮食作物超过了其应有的份额。 我无法想象在没有土豆和西红柿之前欧洲的食物是什么样子。 大米与铁、火箭、火药等许多高科技发明一起来自中国。

    除此之外,技术和政治体系都是在干燥的亚热带(肥沃的新月区)或地中海发展起来的。

    这是闪米特人的文明观,圣经是世界上最古老的书等等。现代考古学与此相矛盾。 相对古老的发达社会位于印度和中国,可能还有其他地方。

    • 同意: RaceRealist88
    • 回复: @szopen
    , @Pat Boyle
  16. songbird 说:
    @Afrosapiens

    我对德国邻近地区的早期分布提出质疑。 可能是错误的解释,但我听说罗马时代的德国人被描述为与美洲原住民接触的时代有些相似。 主要生活在森林茂密的地区,播种抢夺作物。

    我认为在公元前 1000 年之前这几乎是确定的。 在严酷的冬季气候下,老橡树可能只能用铁来砍伐,而不能用青铜砍伐。

    • 回复: @Afrosapiens
  17. Desert Fox 说:
    @RaceRealist88

    抬头仰望天空,看到那些飞机在天空中喷洒白线,在天空中划出巨大的X和井字格等等,这些飞机的配置就像在越南和其他地区的丛林上空喷洒橙剂的飞机一样。越南人民和我们的军队。

    我在蒙大拿州亲眼目睹了这种化学尾气喷洒30年,昨天中午,十几架或更多飞机在喷洒这种化学尾气,这种喷洒将蓝天变成了白色条纹的薄雾,这种喷洒不仅影响了我们的空气,还影响了我们的生活。水、土壤、植物和树木。

    阅读 Nick Begich 所著的《ANGELS DONT PLAY THIS HAARP》一书,可以在 amazon.com.

    • 回复: @Chinookwind
  18. songbird 说:

    我想很多人都没有意识到农业的重要性。 让我大开眼界的是镰状细胞突变的分子钟:大约7300年前。 一种理论认为,人们不可能在很长一段时间内感染疟疾,但这仍然会留下一个很大的预耕窗口期。

  19. 赫特勒那个家伙对森林生态学很有了解。

  20. @Santoculto

    还值得注意的是,说寒冷的冬天会增加智力是事后、故此的谬论。

    “Post hoc,ergo propter hoc 是一个拉丁语短语,意思是“在此之后,因此,因此”。 该术语指的是一种逻辑谬误,即由于两个事件连续发生,前一个事件导致了后一个事件。[1][2]

    在讨论事后、故此的论证时,重要的是要认识到相关性并不等于因果关系。
    魔法思维是事后、故此谬误的一种形式,其中迷信是基于在一系列巧合中看到的模式而形成的。 例如,“这是我的幸运裤子。 有时,当我穿上它们时,好事就会发生在我身上。”

    以下是论证的形式:

    “论证的形式[编辑]
    P1:X发生在Y之前。
    P2:(未说明)Y 是由某件事(发生在 Y 之前)引起的。
    C1:因此,X导致了Y。”

    这是一个相当大的谬误。 当然,这是“相关性不等于因果关系”的一种奇特方式,但是,正如您所看到的,这种推理在逻辑上是错误的。

  21. songbird 说:

    我认为冷选择在很多方面都是合理的,但它作为一个总体理论并不能很好地发挥作用,特别是如果你把冰河时代作为你的主要选择压力的话。 这会给基因提供很长的传播机会。 我认为至少,有一些力量在某些领域选择了对抗更高智慧。

    很可能存在很多不同的选择压力。 几个积极的和几个消极的。 土壤类型必须位于列表的顶部附近。

  22. @James Thompson

    “的始作俑者是谁?”相关性可能不是因果关系,但这是打赌的方式“?

    这是赛勒主义吗?

    • 回复: @James Thompson
  23. @James Thompson

    感谢您的链接。

    但这并不能改变错误的推理。 仅仅因为X发生在Y之前并不能证明Z(这个实例变量X是寒冷的冬天,Y是大脑尺寸的增加,Z是由于寒冷的冬天和大脑尺寸的增加而导致的智商的增加),这是错误的推理。

    还值得注意的是,社交竞争与智力/大脑大小的相关性高于温度。 将其与疾病/寄生虫负担(其也比温度和距稀树草原的距离具有更高的相关性,参见Eppig等人的论文)结合起来,你会发现关于智能增加的温度理论是没有意义的。

    海利彗星的出现与冰淇淋销量之间存在相关性。 哪个导致哪个? 冰淇淋销售和谋杀之间也存在相关性。 当天气变热时,犯罪确实会增加,但只是在一定程度上,当天气变得太热时,犯罪就会开始减少。

    我不认为温度可以解释智商差异,我相信大脑大小的差异反映了工具使用/制造和专业知识能力的差异。

    https://notpoliticallycorrect.me/2017/03/14/brain-size-increased-for-expertise-capacity-not-iq/

  24. @songbird

    可能是错误的解释,但我听说罗马时代的德国人被描述为与美洲原住民接触的时代有些相似。

    如果你指的是美国原住民,我认为古代凯尔特人和日耳曼人比美洲原住民更先进,他们更接近西非人,他们有金属加工,更大的定居点,更复杂的政治组织,我' d 主观地将他们的工件评价为更先进。

    但是,欧洲北半部茂密的树木必然阻碍了进一步的发展,直到中世纪的大规模森林砍伐。

    • 回复: @songbird
  25. @James Thompson

    当然也有一些没有因果关系的相关例子。 以吸烟为例。 两者之间存在很强的相关性,但癌症的起因是一种癌基因,随着时间的推移,吸烟等因素可能会激活该基因。 吸烟只会增加因果元素被激活的机会。

    问题实际上不在于是否存在某种相关性,无论相关性有多弱,而在于是否存在足够强的相关性以至于值得研究因果关系。

    相关性还取决于用于寻找相关性的样本的结构。 这是网站上的一个有趣的例子 http://www.christopherspenn.com/2014/11/causation-can-exist-without-correlation/

    “许多人(包括我自己)在统计中的某个时刻有一个假设,即虽然相关性并不等于因果关系,但没有相关性,因果关系就不可能存在。 然而,事实证明事实并非如此。 因果关系可以在没有相关性的情况下存在!

    “当某种关系似乎是为了因果关系的存在而必须存在时,这怎么可能呢? 它看似简单,归根结底取决于您如何选择数据。 让我们举一个虚构的例子; 假设我在一家酒类公司工作,我想证明酒精不会导致机动车死亡。 为了清楚起见,这两者都不是真的——我不为酒类公司工作,醉酒驾驶是明显不安全的。 不要这样做。

    “如果我对随机、有代表性的人群进行相关性抽样,其中一些人饮酒并不安全地驾驶车辆,而另一些人则没有,你确实会发现饮酒与交通事故死亡人数之间存在密切关系。 。 这似乎表明,为了存在因果关系,相关性是强制性的。

    “但是假设我将我的‘研究’限制在那些在我的非专业人士看来最有可能酒后驾车的人身上。 假设我只关注每天喝 10 杯或更多酒的人? 您可能会发现负相关,事实上,您喝得越多,您因醉酒驾驶而死亡的可能性就越小,因此醉酒驾驶必须是安全的。 那些超级酗酒者中到底发生了什么? 他们很可能死于酒后驾驶以外的原因。 每天喝 10 杯以上,这并不难想象。

    “现实情况是,通过选择没有变异的人群——也就是说,研究中没有人不喝酒——你可以在数据中制造扭曲,从而“证明”你的观点,即使它们在统计上无效。 毫无疑问,我们知道饮酒确实会导致车祸死亡人数增加,但可以操纵数据来“证明”事实并非如此。”

    • 回复: @James Thompson
  26. @James Thompson

    一个著名的例子是山药产量和国民智商,这在不同国家之间呈负相关。 说山药生产在某种程度上抑制智商是错误的,因为这种关联实际上是由温度和纬度等因素介导的。

    或者疾病负担! 呃。

    意识形态太强大了。

    与温度和纬度相反,有经验证据表明寄生虫病和传染病与智商之间存在关系,并且两者之间有更强的相关性。

    是的,显然因果关系需要相关性,但因果关系是通过经验检验来确认的,而不是通过统计调整来证实,统计调整使相关性更强,但没有证明更强的因果关系。

  27. 以下是统计学助理教授 Michael Lamar 在链接中提出的数学证明: https://www.quora.com/Are-there-any-examples-of-causation-without-correlation

    令 X 和 Y 为参数为 0.5 的独立伯努利随机变量。 换句话说,每个都同样可能独立于另一个而为零或一。

    现在让Z=1
    如果 X=Y,如果 X≠Y,则设 Z=0。 很快就会发现 Z 也同样可能是 0,1 和 1。 但即使 Z 的值是根据 X 和 Y 的值明确确定的,但事实证明 Z 是彼此独立的。 为了解为什么,对于 x∈{0.5},我们发现: P(Z=1|X=x)=P(X=Y|X=x)=P(Y=x|X=x)= P(Y=x)=XNUMX=P(Z=XNUMX)

    所以 Z 的条件分布
    给定 X 与 Z 的无条件分布相同。因此 Z 和 X 是独立的(因此不相关)。 同样,Z 和 Y 是独立的(因此不相关)。 但显然 X 和 Y 以最强的因果关系导致了 Z,因为 Z 的值完全取决于 X 和 Y 是否匹配。

    • 回复: @utu
  28. 作为一名园丁,我发现有关雨林土壤动态的信息很有趣。 不过,并不完全感到惊讶。

    从生态角度来说,烧毁雨林并不是事情的结束。 最终,那些低营养草的腐烂将使一系列植物重新生长成森林。 但通过自然过程可能需要数千年。

  29. @YetAnotherAnon

    不确定,但我记得我之前博客上的一位评论员使用过它。

  30. @simplyamazed

    当然也有一些没有因果关系的相关例子。 以吸烟为例。 两者之间存在很强的相关性,但癌症的起因是一种癌基因,随着时间的推移,吸烟等因素可能会激活该基因。 吸烟只会增加因果元素被激活的机会。

    仅仅?

    • 回复: @simplyamazed
  31. 关于吸烟,有些文化吸烟较多,但患癌症的几率很低。 我见过一些令人信服的论点,即吸烟是否会增加患癌症的风险本身可能取决于遗传学,这意味着吸烟可能会大大增加某些人的风险,而对其他人的风险没有影响。 如果你相信人类的生物多样性,那么某些基因会比其他基因更好或更差地处理吸烟的影响,这是有道理的。

    • 回复: @res
  32. res 说:
    @James Thompson

    感谢您提供额外的链接。

    在您的第一个链接中,很多人提到了 Pearl,但可能没有足够的细节来跟进。 参考的是 Judea Pearl,其在该领域最著名的作品是 https://www.amazon.com/Causality-Reasoning-Inference-Judea-Pearl/dp/052189560X
    以下是他于 50 年发表的统计中因果推理的 2009 页概述(也称为 R-350): https://projecteuclid.org/euclid.ssu/1255440554

    蒂莫西·贝茨 (Timothy Bates) 的评论 9 在这方面也提到了艾森克和费舍尔。 谁能推荐他们关于因果关系的特定作品?

    关于第二个链接中的挑战:

    “当然,关联不等于因果关系,但发现我只是因果关系的一个实例,而没有关联(只要一个就足够了)。”

    作为一项挑战,我将向最好的例子提供传统的葡萄酒瓶。 伍德利法官,如果需要,我会仲裁,并提供一瓶酒。

    对于一个简单的人为示例,只需在相等地覆盖负数和正数的范围内使用任何纯二次关系。 例如,整数 [-2, 2] 上的 y = x^2。 作为一个例子,看看与 Z 分数呈强二次关系的关系可能是富有成效的。

    举一个不太人为的例子,考虑一下加速度和速度的关系。 随着时间的推移,观察一系列正负加速度(最终速度大致为 0,并涵盖 + 和 – v)。 我看到相关性为-0.07。 我没有耐心将其减少到基本上为 0,但我很确定这是可能的。

    在底部还有另一个稍微更真实的示例(它依赖于未观察到的中间变量) https://theincidentaleconomist.com/wordpress/causation-without-correlation-is-possible/

    但我认为这些都不值得一瓶酒。 有人给你一个真正的竞争者吗?

    我怀疑基因组中存在一个性状/SNP 组合的例子,其线性效应接近于零(等位基因与性状的相关性),但具有一些二次效应(因果关系)。 推测主要基于给定可能性数量的奇怪情况的可能性。 但百万分之一或更多的可能性并不能真正反驳伍德利的观点。

    • 回复: @Bill
  33. Santoculto 说:
    @RaceRealist88

    这不是我说的,你再看看我写的。

  34. res 说:
    @Lars Porsena

    关于吸烟,有些文化吸烟较多,但患癌症的几率很低。

    对于肺癌来说也是如此吗? 肺癌与吸烟之间的联系之所以引人注目,有多种原因:
    – 与吸烟者的比率相比,基本比率较低
    – 肺部长时间存在致癌性(动物)烟雾的明显物理联系

    心脏病和吸烟形成了有趣的对比。 心脏病的基础发病率足够高,尽管心血管疾病死亡的绝对数量很高,但相对危险却相当低,而且我认为很有可能存在相关的遗传因素。

    以下是一些数字。 请注意肺癌的风险比高出多少:

    全文位于 http://jamanetwork.com/journals/jamainternalmedicine/fullarticle/484996

    值得一提的是,由于不便、费用、耻辱等原因,吸烟的遗传倾向在很多人吸烟的人群(1960 年代的美国)中可能比在相对较少吸烟的人群(2010 年代的美国)中更重要。

  35. 认为答案在于土壤的观点的问题在于,人类进化可能主要是由文化因素驱动的,尤其是宗教等社会结构,它可以产生强大的群体选择优势, 正如我今天在其他地方碰巧讨论过的那样。

  36. @RaceRealist88

    他们还犯了单一原因的谬误:

    单一原因的谬误,也称为复杂原因、因果过度简单化、因果还原论和还原谬误,[1] 是一种可疑原因的谬误,当假设结果有一个单一的、简单的原因时,就会发生这种谬误。事实上,它可能是由许多共同充分的原因造成的。

    https://en.wikipedia.org/wiki/Fallacy_of_the_single_cause

    简单地折磨数据和挑选变量来证实他们已经得出的结论。

    在这篇文章中,有趣的是他们使用生物群落作为变量,而生物群落与种族密切相关,以至于它们基本上可以互换。 然后,他们计算生物群落与种族偏见之间的相关性,从而计算出生物群落偏见的智商估计值。

    汤普森先生,您为什么不解决许多与智商更紧密相关的问题,例如教育或疾病?

    • 回复: @Okechukwu
    , @YetAnotherAnon
  37. RES,

    在我看来,低基本利率本身就是一个统计问题。 据我了解,肺癌是一种非常罕见的疾病,大多数人根本不会患上这种疾病。 即使绝大多数吸烟者也从未因此患上肺癌(有些不吸烟者无论如何也会患上肺癌)。 因此,如果统计数据显示吸烟者患肺癌的几率更高,那么好吧,但这些病例是均匀分布在整个人群中还是发生在高风险亚群中?

    我不能自称是健康统计方面的专家,但我听其他人证明肺癌发病率确实如此。 这是我在谷歌搜索中找到的第一个包含统计数据的网站,我并不声称它显示了任何内容。 http://www.wcrf.org/int/cancer-facts-figures/data-specific-cancers/lung-cancer-statistics

    我特别听说过土耳其和中东部分地区被用作肺癌发病率总体较低的地方的例子,尽管该地区的文化是严重的连续吸烟。

    • 回复: @res
    , @anon
  38. @James Thompson

    “仅仅”是指它不是一个因果因素,而只是一个激活因果因素的因素,无论多么强大。

    当肿瘤抑制基因失活时,当其功能缺失时,它们是某些类型癌症的致病因素。 因此,使用统计方法很难证明它们的相关性; 有些人将统计相关性描述为统计上不显着。 它们是通过功能分析发现的。

    • 回复: @res
    , @simplyamazed
  39. Bill 说:
    @res

    是的,Pearl 就是你在这件事上想要的人。 对于真正了解因果关系的人来说,链接的帖子甚至没有一点说服力。 例如,多变量方法并不是一种从相关性神奇地推断因果关系的方法。

    • 回复: @res
  40. res 说:
    @Lars Porsena

    好的,但是这些实例是均匀分布在整个人群中还是发生在高风险子集中?

    恕我直言,高(呃)风险子集可能与此相关。 但除非这些子集是可以先验识别的(顺便说一句,这也是医生关心家族史的一个重要原因),那么我们只能根据总体趋势做出决定。

    你必须注意的一件事是,人们会陷入单一原因谬误的变体。 这种变体是,另一个可能原因(例如砷、遗传)的存在在某种程度上使得正在讨论的原因(吸烟)变得无关紧要或不重要。

  41. Santoculto 说:
    @RaceRealist88

    在你再次重复之前,尝试理解我写的内容……并根据我的陈述进行回复,而不是你误解了一半的句子。

    • 回复: @RaceRealist88
  42. @Santoculto

    你的这句话就是我引用这个谬论并将《今日心理学》中关于寒冷冬天和智力的链接联系起来的原因。

    人类接触寒冷地区变得更加聪明

    • 回复: @Santoculto
    , @Afrosapiens
  43. Okechukwu 说:
    @Afrosapiens

    汤普森先生,您为什么不解决许多与智商更紧密相关的问题,例如教育或疾病?

    如果他这样做,他就无法将那些讨厌的移民拒之门外。 他需要让全世界相信,他们因基因而丧失了智力。 而且它们具有与西方规范相反的不可改变的特征。 其实这都是一场闹剧。

    • 同意: Afrosapiens
    • 回复: @RaceRealist88
    , @utu
  44. res 说:
    @simplyamazed

    如果你要提出这样的论点,我认为你应该定义“因果”的用法。

    我对“因果”的工作定义往往类似于 Prob(event) < Prob(event|cause) 并且对于为什么“原因”应该影响“事件”以及为什么没有更好的基础都有一个合理的解释解释变量(请参阅智商/教育作为因果变量对话,请注意,由于相关变量可能都是因果关系,因此问题涉及效果的大小和依赖性)。
    (注解,P 或 Prob(A | B) 表示给定 B 时 A 的概率: https://en.wikipedia.org/wiki/Conditional_probability )
    我还期望 < 具有有意义的大小。 量化“有意义的幅度”的一些方法是绝对差异、风险比(例如,如果统计上显着和/或具有一定大小)或解释的百分比方差。

    “因果”并不意味着必要或充分。

    扩展这一点,我认为研究“原因”的可识别性、频率和外显率是有用的。 这些可能会影响干预的有用之处。 即使外显率较低,可识别且频繁发生的事情(例如吸烟)可能是最好的干预场所。 例如,如果外显率由以下因素决定 无法辨认 遗传学存在于不到百分之一的人口中。

    流行病学提供了另一个有用的概念来量化这样的想法:需要治疗的数量: https://en.wikipedia.org/wiki/Number_needed_to_treat
    值得注意的是,与大多数统计数量一样,NNT 指的是特定人群。 这是改进筛选(如前面的帖子中所讨论的)如此重要的原因之一。

    PS 看到诸如“有些人将统计相关性描述为统计上不显着。 它们是通过功能分析发现的。” 也可能被解释为理论上相关但在实践中并不那么重要的东西(例如因为非常不常见)。

  45. szopen 说:
    @Hu Mi Yu

    你是否认为_铁是从中国来到欧洲的_?

    不过,第一批轮式车辆似乎来自北欧

  46. Santoculto 说:
    @RaceRealist88

    不,重新阅读我的评论。 有不懂的可以问我。

  47. res 说:
    @Bill

    是的,Pearl 就是你在这件事上想要的人。 对于真正了解因果关系的人来说,链接的帖子甚至没有一点说服力。

    我很想听听您对此的更多想法。 我发现土壤肥力论点是令人信服的部分原因。 如果没有足够和强大的土壤肥力,就很难维持一个大型(临界质量,而不是巨大)功能社会。 在长期成功的社会中,似乎有一个共同主题是充分维护周围环境(土壤、鱼类、猎物、森林等)的惯例。

    也就是说,我发现接近海鲜的论点更有说服力。

    我的感觉是,就像许多事情一样,关键是要受到挑战(例如寒冷、农业),但又不能达到极端破坏社会的程度。

    例如,多变量方法并不是一种从相关性神奇地推断因果关系的方法。

    同意它们并不神奇,特别是当可能的解释变量高度相关时可能会很麻烦。 你有什么建议呢? 当有意识地努力排除可能的因果变量以支持有效代理(这本身也可能是因果关系,SES 就是一个典型的例子)时,多变量方法尤其危险。

    我没有充分参与珀尔的工作。 我开始对概率图模型感兴趣,将其作为推断因果关系的有用方法,但我对这方面做得很少感到失望。

    • 回复: @Bill
  48. szopen 说:

    当一个人链接到别人写的一篇论文,然后无能的评论者攻击这个人和他的动机,就好像他是写这篇文章的人一样,这不是很有趣吗?

    • 回复: @Afrosapiens
  49. Santoculto 说:

    某些相关关系可以/可能成为因果关系。 例如。 蓝眼睛与较高认知技能之间的相关性。 蓝眼睛似乎不会导致认知技能的提高,但如果您在给定人群中强烈选择这两个变量,并且不选择具有较高认知技能的非蓝眼睛的人,这种相关性可能会成为因果关系(??)。 也许我们有直接和间接的因果关系。

  50. @Okechukwu

    一些寄生负荷/疾病负担论文的主要作者在《科学美国人》上发表了一篇关于此事的文章。 与温度和金泽稀树草原假说相比具有更高相关性的寄生负荷和疾病负担的来源在 Eppig 提供的引文中。

    https://www.scientificamerican.com/article/why-is-average-iq-higher-in-some-places/

    当然,生理学解释在这里最有意义。 患有寄生虫和疾病对新陈代谢的要求很高。 再加上营养不良,这将是智力低下的一个重要原因(无论是什么)。

    • 回复: @Okechukwu
  51. @Afrosapiens

    “你为什么不解决许多与智商更紧密相关的问题,比如教育或疾病?”

    您可以提供一些链接。

    目前测量的智商与很多因素相关,但我认为它们可能是高智商的结果,而不是原因。 每 100,000 万人拥有汽车、笔记本电脑拥有量、高尔夫球场。

    教育是一件棘手的事情。 我怀疑英国人的平均智商在过去 200 年里发生了多大变化,但现在英国受教育的人数比 200 年前要多得多,而且学位水平也比 XNUMX 年前高得多。

    疾病——您是说可治疗的疾病正在降低某些地方的平均智商吗? 好像不太可能,有研究吗? 当然,在英国,缺碘曾导致“行动迟缓”的人。

    • 回复: @Afrosapiens
    , @RaceRealist88
  52. Okechukwu 说:
    @Santoculto

    人类接触寒冷地区变得更加聪明

    这是一个可爱的理论,但与实际证据不一致。 你知道,证据。

    在向极地地区扩张的过程中,人类变得更加聪明/又名自我控制,而当这种扩张因恶劣的气候而受到限制时,他们就停止了增加智力。

    人类只有在条件变得有益时才会迁徙。 那时的北半球就像现在一样,夏天会变得酷热难耐。 那时就像现在一样,这里气候温和,郁郁葱葱,几乎每一平方英寸都可以耕种、放牧或森林覆盖。 冬季降雪实际上储存了作物生长季节所需的水分。

    • 回复: @Santoculto
  53. @RaceRealist88

    汤普森先生,

    如果人类的智力是为了适应寒冷而进化的,或者现在你说温带的温带气候。 那么为什么我们更接近赤道非洲雨林的黑猩猩(图表中排名最低的生物群落)而不是来自温带生物群落排名最高的日本猕猴呢?

    此外,你能提供什么经验证据来证明智商(而不是智力)为前现代人群提供了生存优势,并且它的强度足以让自然选择发挥作用?

    另外,你的生物群落理论能解释弗林效应吗?

  54. utu 说:
    @Okechukwu

    他需要让全世界相信,他们因基因而丧失了智力。

    其实,他不需要做太多的说服工作。 世界上大多数人,包括非洲人民,出于错误或正确的原因已经相信撒哈拉以南非洲人存在严重的认知缺陷。 偏见是巨大的。 像汤普森博士、戴维德·皮弗或我们这里的研究人员这样的人只是想得到对这种偏见的科学证实。 问题是他们从事的并不是一门科学,但不幸的是有足够多的人相信这是有效的科学探索和探究。 然而,如果在双胞胎研究中,智商表型与基因型的遗传力相关,并且将建立显着的种族差异,那么你和非洲人就完蛋了。 这将是几个世纪以来积累的经验和偏见所造成的棺材上的最后一颗钉子。 那么你和非洲人将成为第一个想把自己的智商纹在额头上的人,以表明你高于你的种族群体的预期值。

    但就移民而言,智商是次要的。 显然,人们不想要白痴,但同时人们也可能不想要天才,因为他们确实会破坏文化和传统的社会结构。 尽管如此,如果白痴和天才具有与大多数人相同的外在表型,他们就不会具有破坏性,并且能够通过模仿首先实现完美的同化,除非他们有反社会的宗教模因,例如伊斯兰教或犹太教中的模因。 然而,如果人们具有强烈的外部表型,例如不同的肤色或其他外部种族特征,他们就不应该被邀请作为移民。他们最多可以获得临时工作许可证,但数量非常少。 外部表型使同化变得不可能,并使所谓的种族问题不可避免。 部分原因是模仿是不可能的。 很抱歉,Okechukwu 和 Afrosapiens Europe 并不适合您。 你的存在将不利于欧洲习俗和文化的和谐共存。 我相信你们两个可以为海地和比欧洲大很多倍的非洲任何地方做出很大贡献。 年轻人,去南方吧。

    • 回复: @Afrosapiens
    , @Okechukwu
  55. Anonymous • 免责声明 说:

    新加坡的人口结构使其成为排除的主要候选者,并不是因为它是一个正异常值,其极端程度几乎是下一个正异常值的两倍,而是因为在 1800 年代,大量华人涌入,现在约占 XNUMX%占总人口的比例。 因此,虽然新加坡位于赤道上,但它的人口却不是在该地区进化的。

    但移居新加坡的华人主要是在中国南方的亚热带气候下进化的。 然而今天,新加坡人的智商比中国北方人还要高,更不用说韩国人和日本人了,据说他们的进化环境更有利于智商的发展。

  56. @YetAnotherAnon

    您可以提供一些链接。

    搜索 Eppig 等。 等人。

    目前测量的智商与很多因素相关,但我认为它们可能是高智商的结果,而不是原因。 每 100,000 万人拥有汽车、笔记本电脑拥有量、高尔夫球场。

    嗯,遗传主义对相关性的解释存在问题。 例如,当有人暗示社会经济地位在一定程度上导致了智商差异时,遗传主义者表示这种影响可以忽略不计。 但当人们说智商导致社会经济地位差异时,这种贡献突然变得很强。 很奇怪,不是吗?

    相关性很好,但它们不是证据,它们只是指明了实证检验的方向,这是 HBD 人从来不做的事情。

    教育是一件棘手的事情。 我怀疑英国人的平均智商在过去 200 年里发生了多大变化,但现在英国受教育的人数比 200 年前要多得多,而且学位水平也比 XNUMX 年前高得多。

    听说过弗林效应吗?

    疾病——您是说可治疗的疾病正在降低某些地方的平均智商吗? 好像不太可能,有研究吗? 当然,在英国,缺碘曾导致“行动迟缓”的人。

    是的,有研究,我的朋友 RaceRealists 有一些链接,这对我来说太耗时了。

    我说的是教育和疾病,但还有许多其他因素会影响智商测试分数,比如动机或瑜伽(是的,瑜伽)。 当你意识到这一点时,编造进化场景是有点愚蠢的。

  57. Okechukwu 说:
    @RaceRealist88

    谢谢。 这是一篇非常有用的文章。

  58. @szopen

    当一个人链接到别人写的一篇论文时,这不是很有趣吗?

    他所做的不仅仅是链接到一篇论文,他还认可了论文的结论。

    然后无能的评论者攻击这个人和他的动机

    谁无知? 是我,还是那个说新加坡证实了这一理论的人,就好像新加坡是一个正常的国家一样,而夏威夷(或朝鲜和中国)等其他地方则强烈反驳了同一理论。

    在攻击这个家伙时,我并没有攻击他,但是当问题被提出而没有得到回答时,我可以合理地假设作者不愿意进行逻辑论证。 具有讽刺意味的是,他提出的唯一反对意见是回顾相关性并不等于因果关系,尽管这是最基本的逻辑事实之一,但他仍然认为这一点有争议。 另外,您可以将这种态度与 Chanda Chisala 在评论部分写得很好的帖子和回复进行比较。 没有什么比我们在这里看到的系统性地回避相关言论更好的了。

    这种可疑的态度无法用对科学的真正兴趣来解释,原因可能是时间限制或无聊,但您也可以从发布文章的网站做出推断。 这可能是一种巧合,但这个网站似乎并不是一个高度尊重人类平等和接受多样性的网站。

    一旦你把这些点联系起来(只需要千分之一秒),你就可以清楚地看到作者的动机。 太棒了,不是吗?

    • 回复: @Bill
  59. Santoculto 说:
    @Okechukwu

    你的言论有来源吗? 在没有人类活动的情况下,欧亚大陆的环境如何变得有益健康?

    我并不是说“当人类到达非非洲环境时会变得更聪明”,而是说当他们到达这个地方时,他们已经变得更聪明,阅读,具体来说,(更多)自我控制,并且因为这个次极地地区的极端性质该特征的增加停止了。

    首先,他们变得更加“面向未来”:夏季提供食物,准备冬季,就像适应这种温和的环境一样。 其次,他们中的一些人会变得更有创造力,或者创造性思想家/战略家的比例增加,从而导致农业和文明。

    我不相信非洲人中没有面向未来的人,但这种人在他们/你们中只是少数,而这类人更多地首先是自然环境,其次是人工环境选择的。

    我再说一遍,人类在迁徙过程中变得“更聪明”/面向未来,而不仅仅是在最寒冷的地方。 我们正在谈论创造力。 在非常恶劣的地方,创造力或创造力的人往往会成为不受欢迎的人,务实的思维方式成为黄金法则。

  60. Santoculto 说:

    “当他们到达寒冷的非非洲环境时”。 纠正..

  61. @YetAnotherAnon

    您可以提供一些链接。

    疾病——您是说可治疗的疾病正在降低某些地方的平均智商吗? 好像不太可能,有研究吗? 当然,在英国,缺碘曾导致“行动迟缓”的人。

    是的。 这是一个生理学的解释。 疾病/寄生虫导致身体首先抵抗感染; 能量被转移来对抗疾病,而身体的其他部位显然开始缺乏营养(想想大脑如何吸收我们每日摄入量的 25%)。

    以下是疾病/寄生负荷的引用:

    国家寄生虫负担指数与国民智商密切相关,并且温度在模型中也具有很强的相关性,这一发现得到了支持。 然而,我们暂时建议通过对宿主与寄生虫相互作用的影响进行生理学解释,而不是对温度影响的进化解释。 我们提出这项研究主要是为了强调忽略空间扩展数据中的自相关性的危险,并概述了一种适当的方法(如果认为有必要进行空间显式分析)。

    http://cyber.sci-hub.bz/MTAuMTAxNi9qLmludGVsbC4yMDExLjA1LjAwMQ==/10.1016%40j.intell.2011.05.001.pdf

    墨西哥疟疾干预目标地区的智商高于非目标地区:

    http://cyber.sci-hub.bz/MTAuMTAxNi9qLmpoZWFsZWNvLjIwMTIuMDYuMDAz/10.1016%40j.jhealeco.2012.06.003.pdf

    这在美国也同样适用:

    正如预测的那样,我们发现 0.67 个州的平均智商与传染病压力之间的相关性为 − 0.0001 (p < 50)。 此外,在控制各州财富和教育差异的影响时,传染病压力是各州平均智商的最佳预测指标。

    http://cyber.sci-hub.bz/MTAuMTAxNi9qLmludGVsbC4yMDExLjAyLjAwOA==/10.1016%40j.intell.2011.02.008.pdf

    最后:

    使用三种平均国民智商 (IQ) 指标,我们发现平均智商与寄生虫应激之间的零阶相关范围为 r = -0.76 到 r = -0.82 (p < 0.0001)。 这些相关性在全球范围内以及世界六分之五的地区内都是很强的。 当温度、与非洲的距离、人均国内生产总值和多项教育指标受到控制时,传染病仍然是国民平均智商最有力的预测因素。 这些发现表明,弗林效应的部分原因可能是随着国家的发展,传染病强度的下降。

    http://rspb.royalsocietypublishing.org/content/277/1701/3801

    https://www.scientificamerican.com/article/why-is-average-iq-higher-in-some-places/

  62. @utu

    世界上大多数人,包括非洲人民,出于错误或正确的原因已经相信撒哈拉以南非洲人存在严重的认知缺陷。

    世界上大多数人,尤其是非洲人都会嘲笑测量智力的想法。 世界上大多数国家,尤其是非洲国家,对人类价值有着其他的观念。

    那么你和非洲人将成为第一个想把自己的智商纹在额头上的人,以表明你高于你的种族群体的预期值。

    哈哈! 在做这种蠢事之前我会自杀。 我知道你用的是一个比喻,但无论如何,我从来没有觉得我有什么需要证明的,因为我是黑人,我的成就不言自明,而且毫不费力。

    很抱歉,Okechukwu 和 Afrosapiens Europe 并不适合您。 你的存在将不利于欧洲习俗和文化的和谐共存。 我相信你们两个可以为海地和比欧洲大很多倍的非洲任何地方做出很大贡献。 年轻人,去南方吧。

    LMAO! 我根本不在乎“和谐存在等等”。 我只关心我的女朋友、我的家人和朋友,他们需要我,就像我需要他们一样。

    欧洲人确实不需要我们互相毁灭。 如果没有我们,他们就会像日本一样走向衰落。

    • 同意: Okechukwu
  63. Jim 说:
    @RaceRealist88

    中美洲最早的文字是奥尔梅克文字,而不是玛雅文字。 奥尔梅克文化似乎与玛雅文化有些相似,但由于奥尔梅克文字从未被破译,所以不确定它们与玛雅的确切关系。 除玛雅文化和奥尔梅克文化外,大多数其他中美洲文化也有文字。

    有一些证据表明埃及文字源自美索不达米亚文字,而且中国的文字可能来自西方。

    • 回复: @RaceRealist88
  64. Okechukwu 说:
    @utu

    其实,他不需要做太多的说服工作。 世界上大多数人,包括非洲人民,出于错误或正确的原因已经相信撒哈拉以南非洲人存在严重的认知缺陷。

    事实上,没有。 撒哈拉以南非洲人最终拒绝了殖民主义,因为在与欧洲人的互动中,他们发现欧洲人在认知上并不优越。 他们常常发现情况恰恰相反。

    就世界其他地方而言,我还没有发现他们将非洲的无数问题归咎于智力缺陷。 这几乎不在前 100 个潜在原因的列表中。 世界上最大的公司正直奔非洲以利用那里的智力资本。

    像汤普森博士、戴维德·皮弗或我们这里的研究人员这样的人只是想得到对这种偏见的科学证实。

    答对了! 大奖! 你是绝对正确的。 但他们将会失败,就像他们的前辈已经失败了数百年甚至数千年一样。

    然而,如果在双胞胎研究中,智商表型与基因型的遗传力相关,并且将建立显着的种族差异,那么你和非洲人就完蛋了。

    是的,和一群坐在拖车公园里的键盘种族主义者一起性交。 在崇尚个性的现实世界中几乎没有性交。 此外,没有人质疑智力具有可遗传的成分。 在双胞胎研究中确立这一事实并不能支持至上主义意识形态。

    那么你和非洲人将成为第一个想把自己的智商纹在额头上的人,以表明你高于你的种族群体的预期值。

    哈哈。 你应该为好莱坞写作。 这是一个虚构的反乌托邦未来,只需要一位才华横溢的编剧来处理。 重要提示:这在现实世界中永远不会发生。

    欧克楚库和非洲人 欧洲并不适合你

    如你所知,我不住在欧洲。 我永远不会住在欧洲,因为我无法忍受足球。 然而,爆炸头在那里似乎表现得很好,就像数百万其他黑人一样。 我自己的亲戚住在英国,在那里过得很好。 他们中的一些人说着带有浓重口音的英国英语,我几乎听不懂他们在说什么。 通常,当我渐渐陷入南加州特有的说话模式时,他们也会对我说同样的话。 这些肯定是模仿的例子,不是吗?

  65. @Jim

    中美洲最早的文字是奥尔梅克文字,而不是玛雅文字。

    我非常确定我们掌握的玛雅文字最古老的年代是在公元前 200 年至公元前 300 年之间。

    http://cyber.sci-hub.bz/MTAuMTEyNi9zY2llbmNlLjExMjE3NDU=/10.1126%40science.1121745.pdf

    对于奥尔梅克,我认为日期是公元前 500 年。 你有文件来证实这一说法吗?

    我知道玛雅人和奥尔梅克人也进行人口和文化贸易。

    奥尔梅克文化似乎与玛雅文化有些相似,但由于奥尔梅克文字从未被破译,所以不确定它们与玛雅的确切关系。

    他们相互交融,文化交流值得注意:

    该遗址显示了早期的广场和金字塔建筑早于低地或奥尔梅克地区的建筑,这表明随着玛雅文化的发展,更广泛的文化交流一直延伸到太平洋海岸。

    http://cyber.sci-hub.bz/MTAuMTEyNi9zY2llbmNlLjEyMzQ0OTM=/10.1126%40science.1234493.pdf

    除玛雅文化和奥尔梅克文化外,大多数其他中美洲文化也有文字。

    是的,但我指定的写作是 独立创作.

    有证据表明埃及文字源自美索不达米亚文字

    说得通。 引用?

    中国的文字可能来自西方

    引用?

    http://chemsites.chem.rutgers.edu/~kyc/Five%20Original%20Writing%20Systems.html

    • 回复: @Afrosapiens
  66. Bill 说:
    @res

    我对不同地方的不同智力水平如何以及为何进化的问题没有意见。 不是我所知道的。

    同意它们并不神奇,特别是当可能的解释变量高度相关时可能会很麻烦。 你有什么建议呢?

    取决于上下文。 实验方法。 自然实验方法。 工具变量(经济学家指的是这个短语)方法。

    如果所有这些都失败,您只需做出假设即可确定因果关系的模式。 珀尔的东西——我指的是应用的东西——是指导你弄清楚你到底必须假设什么,以便确定任何特定模型中的因果关系。

  67. @RaceRealist88

    一些历史学家和人类学家区分了独立发明写作的想法和发明脚本。 因此,他们声称,苏美尔和玛雅文明是唯一独立发明了文字观念的文明,而中国、埃及和哈拉帕人可能是通过外来接触而获得文字观念,然后发展出独立的文字观念。脚本。

  68. Bill 说:
    @Afrosapiens

    另外,您可以将这种态度与 Chanda Chisala 在评论部分写得很好的帖子和回复进行比较。 没有什么比我们在这里看到的系统性地回避相关言论更好的了。

    那是妄想。

    • 同意: res
    • 回复: @Afrosapiens
    , @Okechukwu
  69. songbird 说:
    @Afrosapiens

    没错:美国原住民。 毫无疑问,德国人并不先进得多。 我的意思更纯粹是指人口密度。 我想古代德国也是一个非常动荡的地区。 可能会发生很多战争,这可能会使人数低于半岛或岛屿。

  70. Okechukwu 说:
    @Bill

    那是妄想。

    非洲人是正确的。 昌达与批评他的人打交道,而且常常与他们擦地板。 当你的主张植根于现实和基本常识时,它会有所帮助。

  71. 我的意思更纯粹是指人口密度。 我想古代德国也是一个非常动荡的地区。 可能会发生很多战争,这可能会使人数低于半岛或岛屿。

    我对战争一无所知,但我同意你对地形的评论。 半岛、地峡、岛屿、山谷和高原增加了人口密度,有利于文明发展。 意大利或希腊是我们通常认为的半岛,但从更大的范围来看,欧洲是欧亚大陆的一个半岛,而北美是典型的大陆块。 除此之外,地图将高纬度区域渲染得比实际更大。 北边的波罗的海和北海、南边的阿尔卑斯山和喀尔巴阡山脉之间的区域实际上比地图上看起来要小得多。 德国和蒙大拿州一样大。

  72. Factorize 说:

    这篇文章开头的平均气温数据让我想到了为什么国家会失败。
    在本书中,作者认为,在海地等热带地区建立的殖民地是基于“榨取式”发展模式,利用奴隶劳动来收获甘蔗和其他集约型农产品。

    相比之下,在加拿大和美国北部只能种植集约程度较低的农业(例如小麦)。 作者提出,这种差异导致了几个世纪后截然不同的社会结果。 加拿大和美国北部的农业经济所产生的经济和政治力量为更广泛的平等社会创造了条件。 一些殖民地(包括美国和澳大利亚)曾尝试实行精英统治,并以暴力作为惩罚,但当地情况使这些策略站不住脚。

    • 回复: @Santoculto
  73. Santoculto 说:
    @Factorize

    欧洲也是一个非常不平等的地方,许多人生活在贫困之中。 事实上,(工业化)发达国家享有很高的生活水平是最近才发生的事情。 我并不是说殖民地的经济模式不是部分解释这种差异的因素,但我知道这不仅仅是一个因素。

  74. @Desert Fox

    这些化学痕迹绝对是真实的,并且“他们”正在这样做。 我从来没有考虑过让人们变得愚蠢的角度,这是一个新的问题。 我不会把它抛在他们身后。 我一直认为化学尾迹是为了减少太阳能增益。
    肯定有邪恶的成分在里面。 现在已经是21世纪了,我们还在谈论邪恶。

    • 回复: @Desert Fox
  75. utu 说:
    @simplyamazed

    这是一个有趣的数学结构,但我认为它没有太大的实际意义。

    为了讨论 X 和 Y 之间的相关性,两个变量必须具有某种实数表示形式。 为了谈论因果关系X->Y,这意味着必须存在某个将X映射到Y的函数,即X的值决定Y的值。构造产生零相关性的函数是很容易的。 事实上,所有周期函数都会这样做,如果 X 在大量周期内变化,则称 Y=sin(X)。 周期函数不是一对一的。 并非所有 1 对 1 函数都会产生非零相关性。 然而,如果我们施加约束,即该函数也必须是单调的,例如增加如果 X1

    • 回复: @simplyamazed
  76. @Okechukwu

    事实上,没有。 撒哈拉以南非洲人最终拒绝了殖民主义,因为在与欧洲人的互动中,他们发现欧洲人在认知上并不优越。

    别荒唐了。 撒哈拉以南非洲人从未接受过殖民主义,他们只是不得不接受它,因为,正如希莱尔·贝洛克所写:

    无论发生什么,
    我们有。
    马克沁机枪,
    他们没有。

    曾经,苏联人为你们提供了很多自己的枪支,你们就像任何有自尊的人一样,踢了殖民列强的屁股。 做得好。 但不要将所发生的事情归因于智商的种族差异,从而对历史进行无稽之谈。

  77. Desert Fox 说:
    @Chinookwind

    一定比例的化学尾气喷雾是铝的纳米颗粒,我们通过水、空气和土壤吸收它,而铝是阿尔茨海默氏症的原因之一,它正在破坏一个人的大脑,所以如果我们把这些点联系起来,即。

  78. 该帖子似乎是一个合适的地方,可以询问对此问题的专业或经过深思熟虑和知情的答复

    https://www.amren.com/ar/2009/02/
    “道德与抽象思维”。

    我对格达拉·布劳恩一无所知,只知道他告诉我们他多年来在非洲教授哲学的经历,这些非洲包括尼日利亚、肯尼亚和南非,显然他现在居住在那里。

    有些人可能会发现他所说的一些内容令人反感、愚蠢地错误(也许是非洲人和奥克丘库?当然还有 SPLC 的任何实习生),但他确实提供了实际的观察证据,但他显然并没有过分重视这些证据。 所以…。

    特别是对于 JT 博士,我想继续假设他所报告的大部分内容都是真实的,并寻求解释它并考虑其含义。

    让我特别震惊的问题是那些依赖于纯粹口头文化的人们的问题,相应地,与长期的文字文化相比,人们对单词的分析要少得多,可能也更少。

    测试问题包括与识字文化的年龄有关的问题,因为它有助于精确性和复杂性,拥有至少占人口 X% 的活跃识字阶层是否足以区分大多数人为文盲的社会和那些语言本身不是书面语言的。

    (我在这里向语言学家提出了一个问题——也许类似于 Sapir-Whorff 问题——它会根据通过一些简单的后缀或其他手段创造抽象表达的难易程度来区分语言)。

    然后我会质疑布劳恩的广泛概括,他将缺乏抽象思维与不遵守诺言联系起来,例如等同于缺乏对时间、数字和空间的抽象思维。 为什么? 因为澳大利亚原住民以其创世神话和梦幻故事而闻名。 (如果有人想说这些实际上支持布劳恩论文,因为这种对过去的思考是自成一体的,与实际因果思维或对未来的思考没有任何联系,那么我正在听)。

    抽象思维与智商分数相关吗? 我希望如此,但部分原因是弗林本人提出了对弗林效应的解释之一。 他在我脑海中留下的例子是,农民和儿子在回答“狐狸和兔子有什么共同点?”这个问题时,很可能会在 1890 年与 1990 年说的话。 1890年他们说“狩猎”。 在 1990 年,他们会说“两者都是哺乳动物”。

    承认布劳恩的观察并没有完全被不诚实或明显的错误所破坏,通过合理的分析来预测他的有效观察将如何、为什么以及有多大的变化会引起人们对非洲人享受现代性最好方面的关注,这将是一件很有趣的事情。在未来。

    • 回复: @res
    , @Okechukwu
  79. @simplyamazed

    有必要进行澄清。 当我提到泛函分析时,我在更一般的意义上使用了这个描述,并且忘记了它在数学中有一个技术定义。

    我应该明确表示我的意思是对实际细胞间功能的分析。 正常细胞中的细胞增殖受到通俗称为“加速器”和“制动器”的控制。 加速器刺激细胞增殖,而制动器则停止或减缓细胞增殖。 抑癌基因与“刹车”有关。 这些功能在信号通路中发挥作用。 由于与“刹车”相关的基因通常广泛分布在体内的许多器官和结构中,因此将它们的失调(功能缺乏)与特定癌症联系起来会产生如此弱的相关性,以至于可以安全地判断为缺乏相关性。 然而,研究它们在信号通路中的作用建立了失调与肿瘤生长之间的因果关系。

  80. @utu

    对于这个想法在现实世界中如何有意义的实际例子,我们可以看看控制功能。 没有因果关系的控制显然是不可能的,但成功的控制意味着——粗略地说——某个数量保持不变,这意味着它不会与任何事物相关,包括任何导致它不变的事物。

    所以在这种情况下,从缺乏相关性得出没有因果关系的结论是错误的。

    我建议您参考以下链接,以对真实世界示例进行有趣的讨论:

    http://worthwhile.typepad.com/worthwhile_canadian_initi/2012/07/why-are-almost-all-economists-unaware-of-milton-friedmans-thermostat.html

    • 同意: Stephen R. Diamond
    • 回复: @utu
  81. @Afrosapiens

    “像日本一样衰落”

    非常机智。 我希望英国能像日本一样衰落。

    http://www.nytimes.com/2012/01/08/opinion/sunday/the-true-story-of-japans-economic-success.html

    https://www.theatlantic.com/international/archive/2011/02/the-myth-of-japans-lost-decades/71741/

    “我分析的核心是一个日本制造业非凡进步的故事。 你现在很少听到日本制造商的原因是他们中最好的已经从制造消费品转向专注于所谓的生产商产品——虽然消费者看不见的产品碰巧对世界经济至关重要. 这些商品包括高度微型化的组件、先进的材料和超精密的机器,这些都是中国等不太复杂的国家制造最终消费品所需的。 从手机到笔记本电脑的任何东西上的标签可能都写着“中国制造”,但实际上,通过生产商品,日本的高度资本密集型和技术密集型制造商已经悄悄地完成了大部分技术要求最高的工作。

    第二次世界大战后的最初几年,美国完全主导了生产资料行业的高端领域。 然而,在国外竞争的压力下,美国厂商在过去XNUMX年里纷纷倒闭或外包。 竞争主要来自日本,日本现在与 1960 世纪 XNUMX 年代的美国一样,享有广泛的主导地位和地缘政治重要地位。 即使你没有从东京的谈话者那里听到太多关于这一点的信息,在全球贸易数据中也很难忽视它。 (事实:截至 20 年的 2010 年间,美国的经常账户赤字翻了五倍,其原因在很大程度上是因为美国公司已经退出了生产资料业务。)”

    • 回复: @Anon
  82. Santoculto 说:

    许多、大多数非洲人的狂妄程度令人印象深刻。 “非洲人并不认为欧洲人的认知能力优越”。 即使我反对工业化的方式,什么??? 并且还要记住道德上非常错误的(但逻辑上正确的/通过食物链)更强的法则。 直到二十世纪初,欧洲人一直统治着整个世界。 西欧文明成为进步的代名词,是人类能够打造的最佳技术标准。 现在,原始西方人正在输掉这场战斗,但不是通过力量之间的直接对抗。 好吧,所谓的雅利安人已经被证明是一个关于其超人类优越性的神话,但是……

  83. Factorize 说:

    桑托库托,谢谢您的回复。

    我不知道美国、加拿大、澳大利亚等地的新殖民地最初的计划是建立新的封建国家。 美国所有州都试图奴役他们的白人公民,尽管边境无边无际,这是根本不可能的。
    只有当人们清楚地认识到精英统治不可行之后,民主国家才在新世界出现。

    • 回复: @Santoculto
  84. @Afrosapiens

    看到中国的不断衰落也非常令人难过。 特别是需要更多的非洲人。 他们的技能未经选拔,也不愿意为未来牺牲中国人所谓的“血液和资本”。

    • 回复: @Anon
  85. Santoculto 说:
    @Factorize

    定居和开发殖民地是我们在历史课上学到的第一个东西,毫无疑问,它有一些或很多事实真相,但是……

    我真的不知道欧洲地理探险的最初原因是什么,它似乎没有很好的组织或深思熟虑,因为欧洲人不知道他们会发现什么。

    说“他们想在其他地方建立封建国家”似乎是不正确的。

    • 回复: @Santoculto
  86. Santoculto 说:
    @Santoculto

    我们了解到,他们(伊比利亚人)首先开始这次“冒险”,是为了寻找通往印度香料的新路线,因为地中海和索伦地区由“阿拉伯”商人/中间商主导。 这就是解释,或者说我们已经了解了为什么他们开始将“美国原住民”称为印第安人,因为他们相信自己已经抵达印度。 关于葡萄牙在非洲海岸的环球航行,我不知道为什么,也许是其他方式找到通往印度香料的新路线。

  87. res 说:
    @Wizard of Oz

    这很有趣。 谢谢。 布劳恩就此写了一本书。 评论可在: https://www.amazon.com/s/field-keywords=Gedaliah+Braun
    https://store.amren.com/product/racism-guilt/

    抽象思维与智商分数相关吗?

    我认为简单的答案是肯定的。 接下来的问题是:这在子测试中是如何反映的? 也许这里一位对智商测试更了解的人可以对此发表评论?

    一本书中关于种族智商差距的一些评论:
    https://books.google.com/books?id=LfTlBwAAQBAJ&pg=PA292
    他们给出的参考是 https://www.researchgate.net/publication/19795259_Sex_race_residence_region_and_education_differences_on_the_11_WAIS-R_subtests

    该论文讨论了 11 项 WAIS-R 子测试的性别和种族差异(以及更多),并以多种方式呈现结果。 对于性别差异,我认为表 4 最容易解释。 它将每个年龄组和子测试的性别差异表示为 Cohen's d。 女性在数字符号子测试中表现始终较好。 男性在信息、算术、模块设计(见下文)和其他一些方面表现更好。

    黑白差异以类似的形式显示在表 6 中。

    有关 WAIS-R 及其子测试的更多信息: http://www.cps.nova.edu/~cpphelp/WAIS-R.html

    该评分表特别指出了子测试 6. 相似性和 9. 块设计,分别用于衡量言语和非言语抽象推理: http://www.htctu.net/publications/dsps/content/ldforms/WAIS-Rfo.pdf

    块设计是显示最大黑白差距的子测试(恕我直言,在您的评论上下文中具有暗示性),而相似性位于中间。

    结论 3 指出,应避免流行的词汇块设计二元简式测试,因为它不公平地惩罚黑人(他们的两个相对最差的子测试)。

    PS 还有按教育程度划分的分项测试分数的详细信息。 表 12 将其呈现为相关性。

    PPS 虽然那篇论文很旧(1988 年),但提供的详细数据非常有趣。

  88. utu 说:
    @simplyamazed

    在现实世界中,没有任何控制功能可以保持任何东西完全恒定。

    • 回复: @Stephen R. Diamond
  89. Okechukwu 说:
    @Wizard of Oz

    请注意来源:美国文艺复兴时期。

    像布劳恩这样的人所怀有的至上主义倾向以及他们对非白人的轻蔑态度对欧洲人本身来说是毁灭性的。 在伊桑德瓦纳,1300 名英国士兵死在战场上,哭泣着寻找妈妈。 最终,它导致白人被赶出非洲。 它导致了越南的悲剧,以及最近的伊拉克和阿富汗的悲剧。 你们似乎永远不会知道自己并不比任何人优越。

    顺便说一句,语言是一个抽象系统。 这就是所有人类语言。 在我的成长过程中,我接触过两种语言:英语和伊博语。 我发现伊博语更难谈判,也更复杂。 伊博语有一百万种方式可以传达布劳恩声称非洲语言中缺失的所有思想、想法和概念。

    我对我母亲的语言 Efik 也有一定程度的流利。 如果我沉浸在其中几个月,我就可以像母语一样说英语了。 因此,尽管这两种非洲语言的样本量确实很小,但我并没有发现它们缺乏任何复杂或抽象表达的装备。 说到我的母亲,她能说流利的英语、伊博语、埃菲克语和豪萨语。 在我看来,在这四种语言中,如果英语是增强其母语者认知能力的语言,那么我母亲自己的流利程度就会受到影响——而事实却没有。 她的能力远高于平均水平,或者优于以英语为母语的平均水平。 然而对她来说这是第四种语言。

    人与人之间的区别在于文化。 作为一个跨越两种文化的人,我对这种现象有深入的了解。 人们的大脑实际上并没有什么不同,但他们的经历、历史和影响将塑造他们与世界互动的方式。 欧洲文化的某些方面对非洲人来说似乎是原始的、粗俗的和不成熟的。 例如,进入一个区域并拍摄移动的一切的概念。 欧洲人灭绝了与非洲人和谐相处了数千年的动物。 他们现在才开始认识到,他们确实落后了,他们所谓的抽象思维能力无法想象或预见非洲人几千年来所知道的东西。 破坏环境就是破坏你自己。 大自然的平衡很脆弱,例如,消灭狮子可能会导致整个系统崩溃。

    从文章:

    我相信道德需要抽象思维——就像规划未来一样——抽象思维的相对缺乏可以解释许多典型的非洲事物。

    你确实意识到这个起诉书很容易适用于欧洲白人,对吧? 也许比非洲人更是如此。

  90. Anonymous • 免责声明 说:
    @Okechukwu

    这是我一段时间以来读过的关于像小母狗一样不断被殴打的最精心设计的借口。 你知道,阿富汗和越南人民也不是特种部队。

    • 回复: @Okechukwu
  91. hyperbola 说:
    @JimH

    秘鲁古土丘发掘突破性地发现了可追溯到 15,000 年前的先进文明
    http://www.ancient-origins.net/news-history-archaeology/ancient-mound-excavation-peru-leads-groundbreaking-discovery-advanced-021407?nopaging=1

    秘鲁瓦卡普列塔古土丘遗址发现了数十万件可追溯到 15,000 年前的文物。 这些遗物包括精心制作的手工编织的篮子和深海捕鱼的工具,这需要使用能够承受汹涌海水的船只,​​以及大规模农业生产和贸易的证据。 研究结果表明该地区存在早期文明,比最初想象的要先进得多。
    “从瓦卡普列塔取回的成堆文物包括食物残骸、石器和其他文化特征,例如华丽的篮子和纺织品,这确实引发了人们对该地区早期人类发展速度及其知识和技术水平的疑问。该研究的合著者、佛罗里达大西洋大学的考古学家 James M. Adovasio 博士告诉我们,他们曾经从陆地和海洋中开采资源。 Phys.org。 “我们发现的这些一系列事件表明,这些人具有利用不同类型食物资源的非凡能力,这导致了更大的社会规模以及随之而来的一切,例如官僚机构和高度组织化的宗教的出现。 ”……

  92. Okechukwu 说:
    @Anonymous

    这是我一段时间以来读过的关于像小母狗一样不断被殴打的最精心设计的借口。 你知道,阿富汗和越南人民也不是特种部队。

    这应该意味着什么吗? 也许有人可以解释这个角色想说什么。

    • 回复: @Wizard of Oz
  93. utu 说:
    @Okechukwu

    成功文化和文明的语言实际上在语法上通常不那么复杂。 中文和英语是语法非常简单的语言的很好的例子。 语言的复杂性可能是一个障碍。

    例如,进入一个区域并拍摄移动的一切的概念。

    显然你简化了。 土著人民对许多物种的灭绝负有责任。 这是手段的问题。 非洲人只是没有办法杀死一切会动的东西。 谁杀死了北美的猛犸象或新西兰的大型鸟类? 告诉我海地繁盛的动物群吗? 大萧条时期,在美国的树林里很难找到鹿。 对于贫穷的美国农村来说,狩猎是生存的必需品。 现在鹿太多了。

    • 回复: @Wizard of Oz
  94. Art 说:

    苦难激发智慧。 生活在寒冷的气候中需要聪明的人。 也需要合作。

    寒冷需要工业。 只有聪明合作的人才能在寒冷的气候中生存。

    温和的寒冷天气难道不能预测智力吗?

    • 回复: @Okechukwu
    , @Santoculto
  95. hyperbola 说:
    @Okechukwu

    顺便说一句,语言是一个抽象系统。 这就是所有人类语言。 在我的成长过程中,我接触过两种语言:英语和伊博语。 我发现伊博语更难谈判,也更复杂。 伊博语有一百万种方式可以传达布劳恩声称非洲语言中缺失的所有思想、想法和概念。

    语言是一件有趣的事情。 到目前为止,我已经管理了五个,其中三个是“欧洲”的,其中一个是美拉尼西亚的,其中一个是印度次大陆的。 所有这些都是在当地学习的,即在该语言所在的国家学习。 我发现有些东西用一种语言表达得最流畅——同样的东西通常可以用其他语言(某种程度上)表达,但要笨拙得多。 这些主要是文化差异,但它们是真实的。 至于英语,谁的英语? 我从未学过尼日利亚英语,但我确实在国外掌握了四种不同的当地英语。 开玩笑,我认为英语是最难掌握的语言之一(在高水平上 - 有许多本地洋泾浜版本非常容易学习),因为它是精确单词的混杂/混蛋/偷来的混合物其他很少或没有语音或拼写规则的语言。

    至于同质化——老天爷不允许。 没有什么比世界因好莱坞而变得文化同质化更无聊的了。

  96. Okechukwu 说:

    这是手段的问题。 非洲人只是没有办法杀死一切会动的东西。

    他们当然做到了。 任何技术水平的人都可以灭绝任何大型动物。 你是否认真地暗示,如果非洲人齐心协力消灭狮子或大象,他们就会失败?

    谁杀死了北美的猛犸象或新西兰的大型鸟类?

    你在这里自相矛盾。 事实上,猛犸象并不是唯一被史前人类杀死的物种。

    大萧条时期,在美国的树林里很难找到鹿。 对于贫穷的美国农村来说,狩猎是生存的必需品。

    狩猎食物,是的。 但欧洲人不仅仅寻找食物。 他们狩猎是为了毛皮,或者是为了练习打靶,或者只是为了好玩。 他们经常狩猎只是为了清除该地区的动物,他们认为动物没有生命权。 人们正在努力通过基因工程和选择性育种来重新引入许多被欧洲人杀死的动物。 这些动物中很少有曾经是食物来源。 这些程序永远无法复活已灭绝的物种,但它们可以尝试近似其独特的物理特征。

    • 回复: @Wizard of Oz
  97. Okechukwu 说:
    @Art

    苦难激发智慧。 生活在寒冷的气候中需要聪明的人。 也需要合作。

    寒冷需要工业。 只有聪明合作的人才能在寒冷的气候中生存。

    温和的寒冷天气难道不能预测智力吗?

    寒冷天气假说是无效的。

    • 同意: RaceRealist88
    • 回复: @Art
  98. anon • 免责声明 说:
    @Lars Porsena

    吸烟者有两种类型:吸食者和深呼吸者。

    河豚似乎永远持续下去。 深呼吸烟草烟雾?
    没有那么多的未来。

  99. Santoculto 说:
    @Art

    我认为人们在到达次极地地区之前就变得合作或更注重未来,当他们固定在这些地区时,他们就会停止进化以适应环境挑战/变化,而寒冷的气候与太干燥或太热的气候具有类似的影响。 R选择使人们变得更加性欲旺盛、性导向,因为他们没有足够的时间来建造他们的房子并在之后组建家庭。 R选择的人往往大多不适应现代地方,至少收集足够的资源为家人提供非常好的标准生活,同时他们仍然非常适应,因为R选择似乎更通用,就像桉树植物一样,他们几乎不需要开始一个新的家庭。家庭。

    相反,过于恶劣的气候或地区会推动不断的进化,从而限制它并推动获胜者表型/生活方式的不断保护。 至少在自然场景中,真正或字面上的进化的推动力是当环境不仅恶劣而且不可预测或不稳定时。 温带气候,特别是大陆性温带气候似乎是加强保护和自我实现或进化的好地方。

  100. @Okechukwu

    事实上,没有。 撒哈拉以南非洲人最终拒绝了殖民主义,因为在与欧洲人的互动中,他们发现欧洲人在认知上并不优越。 他们常常发现情况恰恰相反。

    哈哈。 如果说“拒绝”,你的意思是欧洲维持殖民主义的决心因西方道德辩论而下降。 葡萄牙是一个主要的例外。 即便如此,你也需要共产党的支持才能获胜。

    我是韩国人。 我并不妄想韩国“拒绝”了日本殖民主义。 我们收到它是因为盟军击败了日本帝国。 韩国从中国独立也依赖于美国的存在。

    你说拥有全球交通、土木工程、医学和其他技术的人并不“在认知上优越”。 好吧,理论上这可能是真的。 但如果这是真的,那么至少一些非洲社会会表现出快速向上流动的轨迹,如中国、台湾、韩国或日本,它们的智商都与欧洲人相当,但并不优于欧洲人。 尽管发生了毁灭性的战争和文化大革命等针对知识分子的残酷内部暴力,亚洲国家仍然繁荣发展。

    在欧洲殖民主义之前,撒哈拉以南非洲人最高的技术成就是什么? 自独立以来? 撒哈拉以南非洲人发表的最重要的科学论文是什么?没有明显的欧洲混合体? 当被问及这个问题时,奇萨拉回避了这个问题,因为这与他的论点不符。

    • 回复: @Okechukwu
  101. Factorize 说:

    韩剧,我发现你对中国将韩国包括在内的民族认同观念的评论很有趣。

    您能否告诉我,中国可能在多大程度上对一个广大的亚洲民族国家提出主权要求?

    • 回复: @Daniel Chieh
  102. @Okechukwu

    “巨魔”将是对他/她/它的一种奉承的回应。

  103. @utu

    是的,这不是他的优点之一。 当狩猎采集者居住在澳大利亚时,澳大利亚仍然有巨型动物和许多早已灭绝的物种……。

  104. Art 说:
    @Okechukwu

    寒冷天气假说无效

    为什么意大利北部产生工业,而意大利南部产生黑手党?

    • 回复: @Santoculto
  105. @utu

    你对无关紧要的事物有天赋。 嗯,是的,没有完全没有相关性,但这完全无关紧要。

    • 回复: @utu
  106. @Okechukwu

    我不认为你在强调欧洲人和其他人在狩猎方面有任何有效的区别。 许多运动被合理地描述为一种发挥古老的、曾经有用的本能的方式,并且应该从这个角度看待各种形式的狩猎*除非指的是对一个物种的肆意破坏*(参见伊斯兰国和塔利班文化破坏)。 除了模仿古老的猎人做法之外,还有一种不那么原始的愿望,那就是用虎皮或象牙等美丽的东西来装饰房间。 请注意,对于大型动物来说,这在很大程度上是一个阶级问题。 事实上,考虑到欧洲从很早开始就惩罚下层阶级偷猎者的方式,就驳斥了欧洲文化倾向于灭绝物种的观点。 (不过,自 1788 年以来,澳大利亚的物种消失是偶然的,就像赤道热带地区各地持续的荒漠化一样,由于无知而加剧)。

    具体来说,您关于非洲人可以消灭狮子和大象的观点最终并不重要,除非与事实相反,欧洲人被视为寻求消灭动物而不仅仅是狩猎动物(为了运动和地位/声望)的人。

  107. Okechukwu 说:
    @K-Drama Kumiho

    哈哈。 如果说“拒绝”,你的意思是欧洲维持殖民主义的决心因西方道德辩论而下降。 葡萄牙是一个主要的例外。 即便如此,你也需要共产党的支持才能获胜。

    不。我说的拒绝是指非洲人从来不认为欧洲的霸权是自然条件。 他们从未接受过这个概念。 他们充分理解欧洲人在技术上优越,但在智力潜力上并不优越。 如果非洲人温顺地接受他们的困境,西方就不会有道德辩论了。

    非洲人对欧洲人的反应并没有像困扰其他土著人民那样充满误导性的敬畏。 他们并不认为欧洲人是某种神奇的、神授的超级生物。 从非洲人的角度来看,欧洲人只是他们自己的怪异版本。 此外,非洲人和欧洲人一样对土地充满狂热。

    非洲从来都不是容易征服的,这就是为什么殖民主义在那里发生得这么晚,这就是为什么殖民主义者从未获得足够的立足点,这就是为什么他们更容易被赶出去,不像在美洲或澳大利亚。 欧洲人直到 19 世纪末在武器技术(马克西姆机枪等)方面获得了相应的优势后才得以进入非洲,这并非巧合。 在此之前,非洲人确实与欧洲军队和定居者交战并击败了他们。 欧洲人被限制在沿海的小飞地,充当非洲国王的附庸。 欧洲人以平等的条件与非洲人进行贸易,因为他们不能通过武力夺取任何东西,包括奴隶。 他们试图袭击非洲村庄寻找奴隶,但遭到惨败。 随后他们签订了条约并支付了奴隶的费用。

    葡萄牙是一个主要的例外。 即便如此,你也需要共产党的支持才能获胜

    除了来自南非和罗得西亚的关键本地支持外,葡萄牙在很大程度上还得到了西方列强的支持。 你的观点是?

    你说拥有全球交通、土木工程、医学和其他技术的人并不“在认知上优越”。

    这是正确的。 你知道人类智力是如何运作的吗? 你们几代人前技术还很原始的韩国人认知能力低下吗?

    但如果这是真的,那么至少一些非洲社会会表现出快速向上流动的轨迹,如中国、台湾、韩国或日本

    首先,他们为什么要这么做? 为什么非洲人生活在不同的环境中,承受着不同的社会和政治压力,在台湾 1/4 大小的土地上拥有多种种族、语言和文化,却仍被指望精确复制同样的发展路径? 但是,尽管存在这些困难,尽管无法像亚洲国家那样从冷战中获取援助,尽管前殖民列强造成并继续煽动战争和社会动乱,尽管没有毗邻主要国际航运通道,但与亚洲四小龙经济体相比,非洲狮经济体的发展轨迹更快。

    他们的智商都与欧洲人相当,但并不优于欧洲人。

    谁说的? 谁进行了这些亚洲智商调查? 谁来报告结果? 谁参加了? 地点、时间和方式? 我怀疑亚洲的智商数据和非洲的智商数据一样具有欺诈性。 智商行业在宣传所谓的高亚洲智商作为反对种族主义指控的堡垒方面拥有既得利益。 在过去,亚洲人被认为是迟钝、懒惰和愚笨的人,他们的智商分数很低,以反映这些特征。 不要相信我的话,亚洲人曾经被认为是愚蠢的。 只需观看老电影和电视节目即可了解它们是如何描绘的。

    尽管发生了毁灭性的战争和文化大革命等针对知识分子的残酷内部暴力,亚洲国家仍然繁荣发展。

    不,他们没有。 哪个亚洲人? 中国有1.4亿人口。 如果人口数量相同,我们可以推断美国 GDP 约为 100 万亿美元,是当前水平的 5 倍。 就目前情况而言,数亿非洲黑人比数亿中国人更富有,生活也更好。 亚洲南部和东南亚地区当然并不繁荣。 几乎从每一个指标来看,很大一部分非洲黑人的境况都比他们好。

    在欧洲殖民主义之前,撒哈拉以南非洲人最高的技术成就是什么? 自独立以来? 撒哈拉以南非洲人发表的最重要的科学论文是什么?没有明显的欧洲混合体?

    这个问题最好针对亚洲提出。 您是否意识到,只有少数美国黑人发明和创新的现代技术比整个亚洲加起来还要多? 其中包括中国、日本和韩国。 中国在遥远的过去非常具有创新性和创造力,但在现代,亚洲人却忙于复制、窃取和借鉴他人的智力成果。

  108. @Okechukwu

    谢谢你的有趣回复。

    令我惊讶的是,你没有指出林恩、拉什顿和布劳恩等人所讨论的问题,即他们的数据样本太小且有限,无法代表非洲境内巨大的遗传多样性,这与非洲以外的狭隘遗传多样性形成鲜明对比其他人类的血统。 由于偶然和自然选择,非洲地区可能存在迷你印度,甚至可能存在种姓的某一语言群体的人们中也可能存在迷你印度。 伊博人不是以聪明而闻名吗(我认为“非洲犹太人”被一些人视为不仅仅是隐喻)? 还有种姓?

    也许布劳恩已经发现了在除了最简单的情况之外的任何情况下支持道德的抽象思维与智商之间的真正联系。 这可能与阶级、弗林效应、从农村生活到复杂城市生活的转变以及认知基因有关。

    • 回复: @Okechukwu
  109. @Factorize

    我不相信这就是他的意思,更多的是“中国影响力”。

    据我所知,中国人并不将朝鲜族纳入他们的民族认同之中(虽然从基因上来说,我相信它实际上与中国东北人相差不远); 然而,中国历来将整个亚洲视为朝贡国。

    这种态度的某些版本可能会以现代化的方式保留下来。

  110. @Okechukwu

    不,他们没有。 哪个亚洲人? 中国有1.4亿人口。 如果人口数量相同,我们可以推断美国 GDP 约为 100 万亿美元,是当前水平的 5 倍。 就目前情况而言,数亿非洲黑人比数亿中国人更富有、生活更好

    这是一种荒谬的衡量方法。 更准确的考虑是,尽管覆盖超过1亿人口且收入差异巨大,但中国的人均GDP甚至比南非还要高,而且几乎所有衡量指标都高于南非,从军事实力到清洁水供应或电网访问情况正在改善(即使在最贫困的地区)。 当然,有肥胖问题而不是营养不良问题。

    如果你认为一个泛非洲国家也能做到类似的事情,那么,祝统一好运。 还有电网。

    但是,您显然有一个议程,因此不值得进一步讨论。

  111. Santoculto 说:
    @Art

    又是这个例子**

    你还记得我已经和你辩论过了*

  112. Factorize 说:

    非洲人,您在本帖子顶部发布的一系列世界人口密度数据极大地帮助我找到了过去数千年人类文明的中心。 当你从一张地图移动到下一张地图时,你可以见证人类的发展历程。

    人类文明的三大支柱(欧洲、印度、亚洲)在这些地图上清晰可见 5,000 年来。 我发现特别值得注意的是,相对最近(上一两个世纪),出现了人类的其他三个支柱(美国、巴西和撒哈拉以南非洲)。 鉴于当前地图所示的人口密度,撒哈拉以南非洲现已达到自我永续发展周期的人口先决条件。

    同样的致密化过程——>认知复杂性——>对更高智力的选择力,以及仅仅通过人口规模的增加而产生的智商异常值,已经推动了创始文明向前发展了数千年。 这些地图向我表明,我们现在正在进入一个时代,促进有益认知结果的背景力量将变得无处不在。 然而,即将到来的基因改造浪潮将在更短的时间内产生更大的影响。

  113. @Okechukwu

    我停下来等待掌声。 挑剔的人可以等等!

    但是,鉴于您可能接受认知能力(无论是测量的表型,包括环境影响还是基因型)在全球范围内分布在大致高斯正态曲线上,您是否可以处理某些大家庭群体具有非常不同的平均值和标准差的概率

  114. 我的意思是我目前编号的 #118 是对此的答复……我的意思是在“sds”后面添加“在[黑人或撒哈拉以南]非洲境内”一词。 我认为现在是一些伊博族至上主义的时候了。
    🙂
    我想你不会有任何问题接受这样的事实:一些通婚的大家庭已经进化出了认知能力的遗传差异(通过更好的智商测试来衡量)。 不难看出近亲繁殖、有读写能力的阿什肯纳兹部落是如何进化出等位基因来进行智商测试所测量的事情的。 同样,格雷格·克拉克(Greg Clark)的《告别施舍; 《世界经济简史》并没有强调欧洲西北部(尤其是英国)的遗传方面,到 1400 年后下层阶级的近亲繁殖在商业和职业上取得了成功(直到 100-140 年前发生了逆转),但没有留下任何内容。其他可能的推论。 正确的?

  115. @Okechukwu

    非洲人对欧洲人的反应并没有像困扰其他土著人民那样充满误导性的敬畏。

    这真是无心之失啊。 你基本上承认 SSA 整体上是不好奇的。 蒸汽机、有机化学、先进火器给亚洲儒家留下了深刻的印象……因为这是一个重视文明和技术进步的人的正常反应。

    你们几代人前技术还很原始的韩国人认知能力低下吗?

    不,但我们显然在文化上不适应。 作为一名女性,我对前现代韩国文化并不怀念。 韩国人表明,一旦我们不再退缩并融入现代世界,我们就在两代人的时间内赶上了并超越了许多欧洲人。 我们现在开发领先的消费技术产品; 您现在甚至可能正在使用其中之一。 看看 1953 年我们的处境多么混乱。即使在人均经过审查的科学出版物产量方面,我们也比日本更好:

    发表科学论文数量排名前 40 的国家数据美丽

    这不像特定的发明那么华而不实,但它是科学技术进步的一部分。

    当然,当我指出非洲同期的发展轨迹时,你会变得防御起来:

    首先,为什么他们应该?……尽管如此,他们仍被期望复制完全相同的发展道路?

    很好的稻草人对我的挑战进行了“完全相同”的歪曲。

    无论如何,如果SSA的智商与东北亚人和欧洲人相当或相当,那么在白魔出现之前的几千年里,你们肯定已经完成了一些值得注意的事情。 即使是智商较低的南亚人,也取得了一些令人印象深刻的成就,首先是从 10 进制数学开始。

    当我问你有关 SSA 成就时,你再次采取防御态度:

    这个问题最好针对亚洲提出。 您是否意识到,只有少数美国黑人发明和创新的现代技术比整个亚洲加起来还要多? 其中包括中国、日本和韩国。

    因此,正如预期的那样,您实际上无法命名任何特定于 SSA 的内容。 你的反击是滑稽的。 以下是日本的发明/发现列表(很长):
    https://en.wikipedia.org/wiki/List_of_Japanese_inventions_and_discoveries

    在与欧洲接触之前,韩国人发明了适合韩语的字母表和第一台金属活字印刷机。 我们有精良的木工、高档的制衣、精美的陶瓷等等。虽然不多,但我们也算不上野蛮人。

    至于“全亚洲”,诺贝尔物理学奖、化学奖、医学奖获得者名单不言而喻: https://en.wikipedia.org/wiki/List_of_Asian_Nobel_laureates。 这些领域唯一的“非洲”赢家是欧洲。 哎哟。

    美国黑人是欧洲人的混血儿,但即便如此,他们也无法开始与上面列出的成就相媲美。 请随意尝试。

    像奇萨拉一样,你用诡辩和康兹式的虚张声势来忽视房间里的大象。

    • 回复: @utu
    , @Okechukwu
  116. utu 说:
    @Stephen R. Diamond

    如果您有一个完美的控制器,例如恒温器,无论环境外部温度 T_amb 在 -74F 到 40F 之间变化如何,都可以将房间内的温度精确地固定在 T=105F,您到底要关联什么。 显然 T 和 T_amb 之间的协方差为零,但相关性是不确定的,因为 T 的方差为零。 但是协方差零告诉我们室温和环境温度之间没有因果关系。 这是正确的,因为恒温器是完美的。 如果某个物理量保持不变,因果关系的问题就不会出现。

    但是我们可以将加热器-AC 系统的能耗与环境温度相关联,并建立约定,当 AC 开启时能量为负,加热器开启时能量为正。 ± 符号表示能量在系统中流动的方向,显然能量消耗是由环境温度驱动的,而不是相反。 然而数学不会告诉你,因为协方差是一个对称函数。 如果你能理解但不能从数学中理解,你可以从其他地方建立的因果关系就像形成物理机制一样。

    我并没有真正看到链接的相关性 简直惊呆了 有些人对所谓的弗里德曼恒温器充满了傲慢。 他们谈论的都是非常微不足道的,因为我可以收集到 w/oa 点。 所谓的弗里德曼恒温器并没有证明与无因果关系的相关性或无相关性的因果关系确实存在。

    我发现这说明了人们一直对Million Friedman恒温器大喊大叫的深度(这一切都归功于尼克罗夫的帖子):

    http://justinhohn.typepad.com/blog/2013/01/milton-friedmans-thermostat-analogy.html
    我看到了优秀的尼克·罗的这段摘录。 这是米尔顿·弗里德曼(Milton Friedman)著名比喻的概述。 它显示了当您通过统计方法分析的“变量”不变时会发生什么。

    当变量不变化时,它就不是变量,也就没有什么可分析的了。 然后这位博主继续我上面给出的例子,并推测一些假设的经济学家会从这种情况下得出什么结论。 这完全是胡说八道,只是表明人们学习经济学的能力。

    在现实世界中,控制器是不完美的,当外部条件不断变化时,它们无法保持它们假设控制常数的参数。 它们基于反馈原理和具有延迟的简单或复杂的控制功能,当外力是随机的时,无法做出预测来预测变化,因此受控参数将不会是恒定的。 会有过冲、下冲和振荡。 控制参数变化的曲线是由控制函数调节的外力函数,所以数学上T=f(T_amb)≠const,所以我们最终得到相关性和明显的因果关系。

    任何人都可以证明一个真实世界案例的例子,其中两个不相关的参数 X 和 Y 之间存在因果关系? 首先请阅读我的评论#80,因为周期函数的琐碎案例不算数。

    PS 似乎是 Matthew Yglesias 将这个平庸的弗里德曼恒温器注入了博客圈,导致了回声室。

    PPS Friedman 为美联储的运作提出了一个恒温器的比喻(2003 年)。 美联储有几个可以调整的变量,比如利率。 除了让富人变得更富有之外,他们可能还有几个目标,例如持续失业或持续通货膨胀。 它们可以提高利率,也可以导致大规模失业,就像沃尔克在“对抗”所谓的通货膨胀时所做的那样,但实际上他所对抗的是工人的高工资。 所以,是的,他们确实有某种恒温器,从这个意义上说,弗里德曼的比喻是恰当的。 但为了让它像恒温器一样工作,他们必须有一个反馈,这样他们就必须不断调整利率,这样他们就不会超出或低于他们的目标,比如失业率太多。 这是相当平庸的想法,但出于另一个原因很有用。 事实证明,在 1980 年代的沃尔克时代,当我开始理解那些混蛋们在做什么时,我用恒温器作为比喻来解释,通过调整一个旋钮,混蛋们就能破坏整个铁锈地带。 当格林斯潘用神秘的语言进行武断时,所有分析师和评论家都试图破译他的神秘智慧和秘密信息,但没有人与他对抗并问道: 格林斯潘先生,您只有一个拨号盘。 您可以将其调高或调低。 这就是你得到的全部。 没什么可说的。 请停止混淆这个简单而平庸的事实。 如果人们发现国王一直赤身露体这个平庸的事实,系统就会崩溃。 格林斯潘告诉我们,当他告诉一些记者他洗澡时喜欢用脚趾调节冷热水时,他大概是赤身裸体的。 也许他的浴室里也有利率恒温器旋钮。 但人们相信了,因为许多人都变得富有了。

    这是一幅为格林斯潘祈祷的卡通片,描绘了格林斯潘是上帝,市场的无形之手是他的右手的时代。

    http://www.art.com/products/p15063141112-sa-i6842934/warren-miller-and-please-let-alan-greenspan-accept-the-things-he-cannot-change-give-hi-new-yorker-cartoon.htm

  117. utu 说:
    @K-Drama Kumiho

    在这里 unz.com 我们有自己的本土哲学家 棱镜系数 他有时会产生有趣的见解,当他的思想火山爆发太多时,这些见解常常被埋藏在转瞬即逝的想法的杂乱之中。 在这种情况下

    https://www.unz.com/imercer/taking-a-knee-is-akin-to-taking-a-pee/#comment-2025837

    他指出了一些吸引我的东西,因为我对智商崇拜持强烈怀疑态度,因为我见过太多智商主义者可以参与的恶作剧。以下是普里斯所写的:

    有人说黑人因为智商较低而无法处理复杂的问题,但也有智商较高的黑人,但他们的态度大致相同。 为什么? 他们的情绪超出了他们的智力。 所以,即使他们和白人一样聪明,甚至更聪明,他们的情绪仍然停留在“扔矛”或“像莫福一样跑”的心态。 这就是为什么即使像康奈尔·韦斯特和迈克尔·戴森这样聪明的黑人听起来也像说唱歌手。

    对于任何因生气或害怕而失去智力的人来说,情绪会完全破坏你的智力,这一概念是显而易见的。 当文化上不受控制和不受缓和的情感使作为智力核心的理性处于停滞发展状态时,某些文化是否有可能因某种原因而发现自己处于智力潜力的负面井中? 难道有遗传因素在里面吗? 就我个人而言,我更喜欢前者而不是后者,但我不排除遗传因素的可能性。 如果文化和环境不能控制情绪,不能减少人们愤怒或害怕的情况,那么很难想象冷静的思考和思考以及反思、沉思和内省会发展起来。 每当恐惧和愤怒占据个人或整个社区时,它们就会抹去任何进展。

    因此,无论是哪种原因导致这种过度情绪化,无论是文化还是某些激活生理过程和激素释放的基因,它都会妨碍执行需要耐心和专注的任务的能力,例如进行认知测试以及和谐合作,而这些任务在以下情况下就无法成功。情绪的爆发打断。

    我曾经观察非裔美国人下棋。 他们中的一些人非常擅长。 然而,他们变得过于情绪化,他们采用垃圾话策略来分散注意力,并用戏剧性和威胁性的身体动作和手势来恐吓对手。 所以这场比赛有很强的物理成分。 由于在这方面失去了精力并且注意力分散,他们显然不可能像那些将所有精力都投入到分析游戏中的玩家那样好。 然而,卡尔波夫在这种充满辱骂的高压环境下有可能输掉比赛。 他的比赛将会受到影响。 显然,黑人如果继续走这条我认为主要由文化决定的道路,他们将永远不会最大限度地发挥他们的智力潜力。 所以他们将走在次优路径上。 我确信智商主义者不会喜欢这个想法,因为他们希望看到黑人被锁在基因的监狱里,无法逃脱。 曾经。 另一方面,我希望看到黑人有机会逃离其文化的监狱,这种文化是病态的(出于多种原因)并且完全不适合现代或现代化社会。 虽然我可以想象在非洲文化进化的过程是自然过程的一部分,但在美国黑人社会的最底层并没有表现出任何改善的迹象。

    我们的对话者是非洲人,他出生在海地,归化为法国人,他没有告诉我们欧洲、波兰或白俄罗斯应该采取什么措施来容纳黑人,他可以启发我们,海地的文化是否会朝着理想的方向发生变化。 那么我们的另一位对话者奥克丘库(Okechukwu),如果我没记错的话,是伊博人,但住在美国,他不会对欧洲白人种族的终结做出兴高采烈的预言,他可以告诉我们,他声称伊戈语和其他尼日利亚人的这种超级丰富的语言如何帮助他们做出重要而复杂的决定,例如如何在社区中分配预算以及如何在这种情况下不让情感战胜智力。

    • 回复: @K-Drama Kumiho
  118. 大卫·兰德斯(David Landes)在他的经济史中认为文化是关键。 他没有进一步研究这些文化的祖先。

    https://www.unz.com/jthompson/david-landes-economic-historian

    以下是对可能顺序的猜测:

    https://www.unz.com/jthompson/the-puzzle-comes-before-solution

    这是一个有些不同的观点

    https://www.unz.com/jthompson/does-culture-cultivate-or-do-you-need

    至于文化的相对优势,衡量标准之一是一种文化向另一种文化的借鉴。

  119. Anonymous • 免责声明 说:
    @Afrosapiens

    当英国人获得许可将新加坡作为自己的地盘时,新加坡还无人居住。 当地的马来统治者对整个交易感到“tak apa”。 新加坡的成功归功于英国的制度和中国的智慧和职业道德。

    这与马来西亚的情况完全相同,只是经济上的成功较小,因为华人在人口中所占比例要小得多,而且马来人垄断了政府并以其典型的平淡风格管理政府。 为了进一步比较,可以将槟城岛和马六甲等马来西亚半岛华人密集地区的发展与玻璃市和登嘉楼等马来人占主导地位的地区进行比较。

  120. Anonymous • 免责声明 说:
    @Santoculto

    热和冷是不一样的。

    只有在缺水的情况下,过热才会造成致命的威胁,因此在许多炎热的国家根本不构成威胁。 尽管如此,它仍然是一个巨大的进化指南,因为它鼓励最小的热量产生,因此最小的能量消耗。 在热带气候下,为未来做计划是有害的,因为总会有某种应季的水果,而在闷热的天气里继续经营的最佳方法就是尽可能少想。

    寒冷致死。 通过暴露和饥饿。 在寒冷的气候下,想办法储存食物并避免暴露在寒冷的环境中对于生存至关重要。

    太热->更愚蠢
    太冷->更聪明

    • 回复: @Santoculto
    , @Santoculto
  121. Svigor 说:

    为什么乌兹别克斯坦对这种情报材料如此狂热? 这完全没有任何科学价值,一切都是为了证实一个信念。 Ir 与生育论同等重要。

    如果一个人关心狂热,那么智力狂热似乎是一个奇怪的焦点选择。 反HBD狂热席卷了整个西方世界。 人类群体认知平等的错觉无疑是西方最普遍、最有害的反科学狂热。

    但你是对的,智商拜物教与生育论和神创论是同等的; 他们无法与人类群体认知平等的狂热相提并论。

    至于想要确认一个信念; 别让我笑。 左派神创论(人类群体认知平等的错觉)拿走了蛋糕,烤了另一个蛋糕,然后又拿走了那个。 其余的甚至连面包屑都没有。

    这就是为什么文明只出现在气候适宜、可以通过粮食生产维持大量人口的地区。 像这样的领域是在没有任何外界影响的情况下独立发明文字的地方。

    我发现更有趣的是,尽管与有文字的文明有接触,但写作和识字从未成为一件事,或者说根本没有成为一件事。 人类的其他创新也是如此。 我的预测是,尽管撒哈拉以南非洲人可以通过智能手机获取一切信息,但他们将继续创造很少的智力价值。

    还值得注意的是,说寒冷的冬天会增加智力是事后、故此的谬论。

    不,你正在陷入我所说的相反的事后、故此谬误:有人认为某事发生在其他事情之后,而你利用这一事实错误地归因于事后、故此谬误。

    如果他这样做,他就无法将那些讨厌的移民拒之门外。 他需要让全世界相信,他们因基因而丧失了智力。 而且它们具有与西方规范相反的不可改变的特征。 其实这都是一场闹剧。

    这毫无意义。 美国黑人与非洲黑人的相似度远高于与白人的相似度。 将黑人带到白人繁荣的地方并没有使黑人繁荣。 无论黑人有什么“问题”,它都是持续存在的。 最终,如果你称之为疾病或行为遗传学,我才不在乎。

    没有办法绕过布莱克的“失败”。 这是一种持续存在的东西,无论是疾病、智商,还是某种阻止他们接受教育的病理学(有些人的梦想是拉智商车的马)。 任何。 如果您愿意,您甚至可以将其归咎于 YT; 原因仍然无法挽回。 YT 无法治愈他“导致”黑色“失败”的原因,就像他无法治愈任何其他原因一样。

    世界上大多数人,尤其是非洲人都会嘲笑测量智力的想法。 世界上大多数国家,尤其是非洲国家,对人类价值有着其他的观念。

    这就是说法。 左派将智商等同于“价值”,一种价值判断,并从那里开始倒推。

    许多、大多数非洲人的狂妄程度令人印象深刻。

    如果我们能够利用自我获取能量,黑人现在就已经在比邻星殖民了。

  122. Svigor 说:

    对于任何因生气或害怕而失去智力的人来说,情绪会完全破坏你的智力,这一概念是显而易见的。 当文化上不受控制和不受缓和的情感使作为智力核心的理性处于停滞发展状态时,某些文化是否有可能因某种原因而发现自己处于智力潜力的负面井中? 难道有遗传因素在里面吗? 就我个人而言,我更喜欢前者而不是后者,但我不排除遗传因素的可能性。

    显然,人类行为是由基因驱动的,人类群体行为的差异很大程度上归因于基因差异。 例如,与其他种族相比,黑人更加外向和好斗,缺乏责任心或开放的经验。 告诉自己,如果它能让你快乐,那就是文化,但奥卡姆剃刀却恰恰相反。

  123. Santoculto 说:
    @Anonymous

    我知道,显然它们不一样,但我说的是恒心。 较冷的气候总是寒冷,较热的气候总是炎热(例如赤道地区)。

    太热->更愚蠢
    太冷->更聪明

    我认为我们不是在讨论愚蠢与智力,而是短期智力与长期智力。 我不认为生活在亚马逊流域的美洲印第安人和生活在萨凡纳的非洲部落“愚蠢”。

    短期思考者在长期环境/社会中的许多行为是由于他们的适应不良而不是本身的“愚蠢”(慢性失语)。

    我们总是拿苹果和橙子进行比较。

    狩猎采集者的后代与后“文明”的后代。

  124. Santoculto 说:
    @Anonymous

    更炎热的气候和周围巨大的生物多样性[昆虫、动物……]往往会选择较短的思维,但这并不一定是因为它比亚极地地区的危险要小得多。 它在不同方面是危险/困难的。

    在热带/赤道地区建造一座非常坚固的房子是没有意义的。

    我的主要假设是人类正在获得面向未来的智能[或增加其频率及其表达] ,我们将参加 他们的迁徙而不是 after,当他们到达极端的地理区域时。 因此,当他们到达这个非常寒冷的地区时,他们停下来增加/完善他们的抽象/未来思维智力。 当我们谈论文明时,我们谈论的是创造力。 创造力似乎非常适合气候不稳定的温带气候地区,特别是大陆性温带气候。 人类在心理和认知上不断进化并适应每种气候。 我并不是说创造力不是人类的普遍特征,但在面临许多挑战的地方往往更需要创造力。 即使是创造力不强的人,在面临强烈挑战时也往往会被迫以不同的方式思考,从而增加个人的创造力潜力。

  125. Factorize 说:

    我们正处于人类平均智力将急剧提高的时刻。 即将发生的事情没有先例。 基因奇点已经到来。

    不需要高科技。 胚胎之间的简单选择会增加
    geno-g 由单个 SD 到多个 SD。 如果配子或胚胎中的单个染色体的选择成为可能,那么智商的潜在提高将非常大,也许是 20 SD。 每个人的 DNA 中都蕴藏着极高智力的遗传潜力。

    当前方的道路如此光明时,为什么还要看后视镜呢?

    • 回复: @Daniel Chieh
  126. Okechukwu 说:
    @Wizard of Oz

    令我惊讶的是,你没有指出林恩、拉什顿和布劳恩等人所讨论的问题,即他们的数据样本太小且有限,无法代表非洲境内巨大的遗传多样性,这与非洲以外的狭隘遗传多样性形成鲜明对比其他人类的血统。

    林恩、拉什顿、布劳恩等人,这已经是定局了。 是骗局。

    你说到了一个有趣的点。 如果将人口分为智力上的优等和劣等,那么我们完全有可能在非洲找到世界上最聪明的人。 根据 HBD 理论得出正确的结论,如果我们在细节中描述人类基因,那么基因库较丰富的人群被赋予了其他相对瓶颈和近交人群所缺乏的基因和智力频率,这并非不可想象。

    伊博人不是以聪明而闻名吗(我认为“非洲犹太人”被一些人视为不仅仅是隐喻)? 还有种姓?

    是的,聪明的伊博人是一种刻板印象。 当然,伊博人并不总是聪明,但他们因被认为聪明和“像犹太人”而遭受集体迫害。 比夫拉内战经常被北方穆斯林描述为针对犹太人的战争。 我对犹太人的联系持不可知论。 我们的一些文化习俗惊人但可能巧合地相似。

    伊博人不实行种姓制度。 他们最重要的情感是对家庭的奉献——广泛的大家庭,最终涵盖了整个伊博兰和侨民。 伊博语中没有相当于“表弟”这个词的词。 甚至远房表兄弟姐妹也被称为兄弟姐妹。 要明确地将某人认定为表兄弟,需要进行解释。

    伊博人崇尚教育、成就和成功。 这是我们所拥有的最接近种姓制度的东西,高成就者处于最高地位。 为了达到目标并超越彼此,甚至在兄弟姐妹之间,竞争可能是恶性的。 它确实促进了成功的结果,但也会引起嫉妒和恶意。

    也许布劳恩已经发现了在除了最简单的情况之外的任何情况下支持道德的抽象思维与智商之间的真正联系。

    道德是一个相对的、主观的概念。 有些奴隶主认为自己是美德和道德的典范。 基于肤浅的互动和根深蒂固的偏见对整个大陆做出有害的判断实际上是不道德的。 因此,具有讽刺意义的是,布劳恩本人就是不道德的。

  127. Santoculto 说:
    @Okechukwu

    相对就是相对…

    因为有些人认为自己在道德上是正确的,即使他们不是,但这并不意味着他们是正确的。 如果道德是相对的和主观的,为什么白人有罪? 如果道德是相对的和主观的,我的意思是,没有客观标准,为什么要通过奴役来指责白人???

    “事实上是不道德的”

    ...

    这不是相对的或主观的?

    • 回复: @utu
  128. @Factorize

    当前方的道路如此光明时,为什么还要看后视镜呢?

    我喜欢这种态度,但公平地说,由于程序的成本,我怀疑这只会引发更多的精英分类。

  129. Okechukwu 说:
    @K-Drama Kumiho

    这真是无心之失啊。 你基本上承认 SSA 整体上是不好奇的。 蒸汽机、有机化学、先进火器给亚洲儒家留下了深刻的印象……因为这是一个重视文明和技术进步的人的正常反应。

    非洲人也对这些东西感到好奇,笨蛋。 这就是他们去欧洲和美国学习的原因。 我的父母来这里读书。 这就是我在这里的原因。 非洲人对枪支也非常好奇。 以至于殖民列强之间达成了一项协议,以防止现代枪支落入非洲人手中。 他们这样做可能有充分的理由。 我的意思是,祖鲁人只用长矛就杀死了 1300 名精锐的英国军队。

    我们现在开发领先的消费技术产品; 您现在甚至可能正在使用其中之一。

    这些都不是韩国人在韩国有机开发的。 您是从他人的工作和辛劳中获益的人。 当我说少数美国黑人发明和创新了整个亚洲加起来的更现代技术时,我是认真的。

    一个黑人在发明你用来写亚洲至上主义垃圾的个人电脑方面发挥了很大的作用(见马克·迪恩)。

    一位黑人发明了现代视频游戏机,它构成了日本游戏业的基础(参见杰里·劳森)。

    一位黑人在开发我们今天拥有的手机技术方面发挥了重要作用,而你们的三星已将其变成了价值十亿美元的业务(参见亨利·T·汤普森)。

    追溯到更远的时期,在日益严重的种族主义和吉姆·克劳法的影响下,黑人发明了我们今天使用的许多变革性技术。 更进一步地说,我们甚至让黑人作为奴隶进行发明和创新。 你提到了汽船。 嗯,是一个黑人奴隶完善了螺旋桨技术(见本·蒙哥马利)。 因此,你声称亚洲人好奇的东西,虽然指责黑人缺乏好奇心,但部分是黑人的智力成果。 相反,亚洲人在自己的国家是自由的。 他们没有遭受使人衰弱和非人化的压迫。 然而他们什么也没产生。 他们什么也没创造。 他们什么也没发明。

    我向所有正派的亚洲人道歉。 但这个角色需要一剂冷酷的现实。

    韩国人发明了适合韩国语的字母表和第一台金属活字印刷机。 我们有精良的木工、高档的制衣、精美的陶瓷等等。虽然不多,但我们也算不上野蛮人。

    朝鲜战争之前和战争期间,朝鲜是一个原始石器时代社会。 我看过电影。 但与你不同的是,我并不是说他们愚蠢或低劣。 今天的朝鲜基本上是韩国的镜像,没有美国的慷慨。 你们是美国创造的,也是美国养育的。 你们被允许在这里倾销所有产品,同时对美国产品关闭市场。 由于美国的保护伞,您可以将资金转移到其他用途,否则将用于国防。

    你们的整个国家都是美国捏造的。 事实上,你拥有的一切和你想要的一切都是从美国复制或窃取的。 你们的韩国流行音乐是你们深恶痛绝的黑人发明的音乐风格的衍生品。 不仅是音乐,还有表演、舞蹈动作、态度、习语——都反映了非裔美国人的审美观。 因此,你不仅无法发明自己的技术,甚至无法发明自己的流行文化。

    至于“全亚洲”,诺贝尔物理学奖、化学奖、医学奖获得者名单不言而喻: https://en.wikipedia.org/wiki/List_of_Asian_Nobel_laureates。 这些领域唯一的“非洲”赢家是欧洲。 哎哟

    鉴于地球上有数十亿亚洲人,亚洲诺贝尔奖获得者的相对极少的数量掩盖了您关于亚洲霸权的争论。 顺便说一句,诺贝尔奖存在于北欧文化矩阵中。 呃……这就是为什么绝大多数获奖者都是北欧血统的原因。 现在越来越多的非洲人也加入其中。 绝对没有什么可以排除非洲人成为未来诺贝尔奖获得者的可能性。 事实上,鉴于你垂死挣扎的表现,非洲人在不久的将来超越你也不是不可想象的。

    美国黑人是欧洲人的混血儿,但即便如此,他们也无法开始与上面列出的成就相媲美。

    但如果黑人在基因上不如白人和亚洲人,为什么混血黑人会比 100% 白人和亚洲人更聪明、更有创造力、更有创新精神呢? 嗯……这没有任何意义。 对不起。

    像奇萨拉一样,你用诡辩和康兹式的虚张声势来忽视房间里的大象。

    那就和钱达一起去玩吧。 显然他对亚洲霸权的官样文章更加宽容。 钱达是一名移民。 很多时候,移民都有如履薄冰的倾向。 我,我是美国人。 我们不玩那种狗屎。

  130. @Okechukwu

    你提到嫉妒让人想起 1966 年(德语版,1970 年英语版)Helmut Schoeck 所著的《Envy》一书。 他特别讨论了克服嫉妒对经济和文化进步如此重要的方式,以及古希腊和基督徒处理嫉妒的不同方式。 我记得他指出,优秀的西非农民会因为害怕嫉妒而向他人隐藏自己的技能、辛勤工作和成果。

    为什么你把林恩、拉什顿和布劳恩都形容为“骗子”? 即使系统性错误或谬误通常并不意味着某人有欺诈行为。

  131. @Okechukwu

    DNA 分子的共同发现者詹姆斯·沃森 (James Watson) 表示,“我本质上对非洲的前景感到悲观”,因为“我们所有的社会政策都是基于这样一个事实,即他们的智力与我们相同,而所有的测试都表明并非如此” ”。 对我来说,这听起来是一个明智的意见。

    • 回复: @RaceRealist88
  132. @utu

    Utu,我的立场是,智商衡量的是一些有用的东西,可以说是有利于现代工业社会成功的技能,但声称智商代表一般智力是错误的。

    我发现来自相对平均得分较低的群体的平均代表个体与来自较高平均得分的智商相似的个体之间的社交技能存在显着差异,这对智商测量值构成了巨大的挑战。 智商似乎忽略了言语技巧和“常识”推理。 它还可能会错过卡拉哈里布须曼人或澳大利亚原住民等异常人类群体所表现出的令人印象深刻的视觉空间能力。

    我同意,明显的智力,如智慧或成就,似乎在很大程度上受到情感的调节。 大学教授的原始智商显然很高,但他们在无私的学术研究上失败了,可悲的是,这在 STEM 之外的学者中占很大一部分,他们几乎总是在情感上投资于某种“事业”或某种类型的部落身份。 这种选择性诚实和坚持叙述的问题并不是一个新现象,但 20 世纪哲学的某些发展使曾经失败的行为神圣化为学术界可接受的行为。

    我也同意,虽然遗传学可能在人类群体层面的差异中发挥作用,但认识到社会化的强大影响也很重要。 我主要的挫败感之一是“种族主义”如何从反对生物决定论和拒绝给予个人公平机会,变成了文化和宗教的掩护。 它使我们处于这样的境地:结构性社会问题是唯一“可接受”的批评目标。 这阻碍了多元化世界各方面的进步。

    • 回复: @utu
  133. @Okechukwu

    该死的兄弟,你绝对会杀人。 明天我会重新开始谈话。

  134. @Okechukwu

    我发现我的记忆可能已经失败了。 我在谷歌上搜索“非洲有种姓吗”,找到了很多例子,但当然没有伊博族。 在许多方面和维度上比较印度和非洲的种姓会很有趣。 并添加与阶级系统的比较。

  135. @Peter Johnson

    这似乎是对权威的诉求(尽管他在那里没有权威)。 为什么HBD人这么喜欢谬论? 事后、因此、诉诸权威、有些诉诸情感等。

  136. Anonymous • 免责声明 说:

    低等人有两种:

    尽管两者最终都无法拥有真正的知觉(这需要灵魂和神圣的火花),但其中一种猿类比另一种猿类更擅长模仿。

  137. Pat Boyle 说:
    @Hu Mi Yu

    中国人没有研制出火药。 有一场定义明确的火药革命,它描述了我们所说的黑火药首次在军事用途枪支中有效使用的时间。 它发生在十五世纪的法国和德国。 有效火药的开发经历了许多不同的阶段,包括康宁公司。

    当查理八世使用火药和枪支入侵意大利时,所有现有的防御工事立即被废弃。 几年之内,军事建筑师创造了一种全新形式的多边形防御工事。 再高又薄的墙都抵挡不住枪。 这场革命席卷了东方,直到很久以后才到达中国。

    那些喜欢认为中国发明了火药的人的思维过程是草率的。 当一位发明家添加了使他的发明变得有用的最后一个部分时,我们向他致敬。 我们不会将一项发明归因于第一个使用该发明的人。 如果我们这样做的话,我们就会把电灯归功于某个早期的玻璃吹制者,而不是爱迪生。

    • 回复: @Daniel Chieh
    , @utu
    , @res
    , @Jason Liu
  138. Pat Boyle 说:
    @RaceRealist88

    你这是在欺负读者。 我们大多数人在学校学习拉丁语,许多人学习形式逻辑。

    你没有让任何人陷入可怕的尴尬错误,因为有“时间之箭”。 原因确实先于结果。 因此,所有假定的原因首先需要先于其观察到的结果。 时光倒流的东西只存在于科幻小说和理论物理的某些部分中,我不明白。

    谬误部分是当你仅仅证明优先级时就认为你已经证明了因果关系。 这里没有人这样做。

    “寒冷的冬天产生智力”的假设只是一个假设,而且我认为这个假设可能是错误的。 对我来说,这似乎过于简单和错误,但这并不是谬论。

    • 回复: @RaceRealist88
  139. @Pat Boyle

    只有“经典枪支”设计才能定义火药时代的想法似乎与奥斯曼帝国早在 1420 年代就一直有效地使用大炮和炮弹作为攻城武器的观察结果不一致; 虽然君士坦丁堡的围困并没有让所有城墙立刻变得过时,但很难说,君士坦丁堡的围困没有给世界留下战争发生了根本性转变的印象。

    • 回复: @Pat Boyle
  140. utu 说:
    @Okechukwu

    非洲人也对这些东西感到好奇,笨蛋。 这就是他们去欧洲和美国学习的原因。 我的父母来这里读书。 这就是我在这里的原因。

    他们为什么把你遗弃在美国? 因为我猜想,一旦你的父母学会了如何制造蒸汽机或任何他们感到好奇的东西,他们就会回到尼日利亚,用他们新获得的知识来回报尼日利亚政府,尼日利亚政府可能用来自联合国的资金资助他们在美国的学习。 尼日利亚或任何非洲国家从你留在美国的遗憾中得到了什么? 您如何帮助将非洲变成下一个韩国,并如何为实现非洲诺贝尔奖遍布世界的伟大愿景做出贡献? 与此同时,你应该节制对亚洲人的坏话,因为你的未来可能取决于他们。

    • 回复: @Wizard of Oz
  141. utu 说:
    @Pat Boyle

    那些喜欢认为中国发明了火药的人的思维过程是草率的。

    要获取事实,不需要思考过程。 事实上,火药是中国发明的。 保留这一事实并将您的思考过程用于更有用的事情。

    • 回复: @Pat Boyle
  142. utu 说:
    @K-Drama Kumiho

    好好好。 我得到了它。 我的观点是,当可能受环境和文化影响的人格特质和情感构成主导智力因素时,智力问题就可以搁置一边。 这是普里斯所说的: “他们的情绪超出了他们的智力。” 这个提议可以被采纳,因为它更符合奥卡姆剃刀原理。 在这种情况下,奥卡姆剃刀对非洲人有利。 因为如果事实证明遗传因素决定了非洲人的智力较低,那么他们就真的完蛋了,没有希望了。 然而,如果环境和文化因素所决定的情感构成超出了他们的智力,那么他们就有希望。 一旦他们修复了病态文化并掌握了失控的情绪构成,他们与生俱来的智力将有机会发展和绽放。

    • 回复: @K-Drama Kumiho
  143. @Okechukwu

    Okechukwu,曾经和未来的康,写道:

    朝鲜战争之前和战争期间,朝鲜是一个原始石器时代社会。

    “石器时代”是一个具有实际人类学意义的概念。 韩国人从事金属加工已有两千多年的历史,因此我们并不是定义上的“石器时代”。 你的侮辱的微不足道和绝望是不言而喻的。 这次爆发让你对其他无端的反韩言论有了正确的认识。

    相反,亚洲人在自己的国家是自由的。 他们没有遭受使人衰弱和非人化的压迫。 然而他们什么也没产生。 他们什么也没创造。 他们什么也没发明。

    显然,您需要向您指出,历史并不是欧洲人开始在撒哈拉以南非洲施加影响时才开始的。

    您关于亚洲历史的言论表明,您对欧洲殖民主义及其在亚洲的影响力的无知,以及对毁灭性的亚洲内部冲突的严重无知。

    你所说的“什么也没创造”和“什么也没有发明”是指除了火药、纸张、指南针、机械钟、冶炼、丝绸生产、各种农业创新等等之外。

    当我说少数美国黑人发明和创新了整个亚洲加起来的更现代技术时,我是认真的。

    嗯,我发布了数十项亚洲发明、发现和诺贝尔奖获得者的链接,以及通过同行评审出版物对人均科学产出的一般衡量标准。 你提到了四名非裔美国人的名字。

    这些都不是韩国人在韩国有机开发的。 您是从他人的工作和辛劳中获益的人。

    哇,谈论无耻的双重标准。 尽管在白人占绝大多数的环境中工作,你们的非裔美国人却得到了你们的充分、不可分割的赞扬,但韩国人却没有因为我们的贡献而得到赞扬。 你还能再显得更加痛苦和绝望吗?

    ……为什么混血黑人比 100% 白人和亚洲人更聪明、更有创造力、更有创新精神?

    你的问题只是表明你不了解群体平均值如何与任何特定个体的状态相比,也不了解混合如何影响基因调节的行为。

    不值得你进一步参与。 你似乎体现了“We wuz Kangz”心态。

    事实上,鉴于你垂死挣扎的表现,非洲人在不久的将来超越你也不是不可想象的。

    QED

    • 回复: @Daniel Chieh
    , @Okechukwu
  144. @Pat Boyle

    你这是在欺负读者。 我们大多数人在学校学习拉丁语,许多人学习形式逻辑。

    谁是“我们许多人”?你怎么知道这一点? 这个评论是针对桑托库托的,我百分之九十九肯定他不知道这个谬论。

    谬误部分是当你仅仅证明优先级时就认为你已经证明了因果关系。 这里没有人这样做。

    桑托库托写道:

    人类接触寒冷地区变得更加聪明

    所以是的,确实有人这么做了。 “X发生在Y之前”(寒冷的冬天发生在智力增加之前)”。

    “寒冷的冬天产生智力”的假设只是一个假设,而且我认为这个假设可能是错误的。 对我来说,这似乎过于简单和错误,但这并不是谬论。

    是的,有点是。

    “P1:X发生在Y之前。
    P2:(未说明)Y 是由某件事(发生在 Y 之前)引起的。
    C1:因此,X导致了Y”

    P1:寒冷的冬天发生在智商/智力增加之前
    P2:智商/智力增加是由某些事情引起的(在智力增加之前发生)
    C1:因此,寒冷的冬天导致更高的智商/智力

    我同意 CWT 是错误的,如果你说 X 导致了 Y,那是一个谬论。

  145. @K-Drama Kumiho

    不值得你进一步参与。 你似乎体现了“We wuz Kangz”心态。

    我认为康兹心态不一定有什么错,只要它对未来的成长确实有用即可。 不过,目前预兆看起来很糟糕。 例如,考虑到未来可以选择更聪明的后代,我们的客人是否会追求这样的方法?

    • 回复: @Santoculto
    , @utu
    , @K-Drama Kumiho
  146. Santoculto 说:
    @Daniel Chieh

    “我认为康兹的心态不一定有什么问题”

    一切都错了,超级自信会毁掉国家。

  147. utu 说:
    @Daniel Chieh

    我认为康兹心态不一定有什么错,只要它对未来的成长确实有用即可。

    有趣的一点。 这种近乎肆无忌惮的自信一定来自于某种内在的力量。 为了维持它,它必须通过偶尔的成功来施肥。 军事上的成功就足够了。 或者精心策划和执行的种族灭绝(最好是针对白人)也可以。 但他们不太可能。 另一方面,效仿韩国人或台湾人的悄然成功的可能性很小。 这需要拥有忠诚的精英,他们将抵制外部腐败,并将社会的福祉放在心上。 在非洲很难找到它。 我在非洲还没有见过具有这些品质的领导人。 有一种文化。 非洲文化的哪些部分适合建设一个依赖纪律、坚持、诚实和合作的现代社会?

    • 回复: @szopen
  148. @utu

    关于情绪阻碍理性的运用,从而无法实现遗传智力潜力的论点,如果相对情绪控制本身受到基因的强烈调节呢?

  149. @utu

    @Okechukwo。 我也很难理解utu……

  150. @Daniel Chieh

    我认为康兹心态不一定有什么错,只要它对未来的成长确实有用即可。

    如果这只是一个典型的建国神话,我就不会费心去嘲笑它,尽管有明显的非历史因素。 伴随康孜言论而来的是更广泛的心态,这让我深感不安。

    塔内西斯·科茨的世界观就是这种观念的体现,即黑人仅仅受到外部力量的压制,而外部力量大多是“自认为是白人的人”。 这导致了对个人代理的拒绝,而那些有勇气挑战科茨的黑人知识分子几乎一致强调了这个问题。 它更广泛地延伸到对文化中培育的美德的拒绝,这些美德使其他群体改善了他们的状况。 只要看看奥克楚库所使用的借口和逃避就知道了。

    • 回复: @szopen
  151. szopen 说:
    @utu

    据我所知,美国黑人比白人有更多的自尊心。 “我们乌兹康”的心态最初可能对他们有帮助,但在过去的四十年里似乎并没有帮助他们。

    (是的,我知道有些人认为黑人与白人之间的差距仍在缩小,但问题是检查“黑人”和“白人”是否指的是与以前相同的人口)。

  152. szopen 说:
    @K-Drama Kumiho

    我认为,如果遗传主义假说是错误的,那么人们可能会说,压低黑人的不是种族主义或歧视,而是“我们是受害者”的有毒文化。

    当然,詹森夫妇对“x因素”的批评也适用于此(即,平均而言,这种文化必须对所有美国黑人产生同样的削弱作用,无论他们的社会经济地位如何等)。

  153. Jason Liu 说:
    @Okechukwu

    啊,原来的“黑人发明家”名单原来只是机器上的齿轮,与开发“他们的”发明无关。 我确信这些名字将得到主流历史的证实。

    一般来说,我不赞同严格的 HBD 等级制度,但你已经把自己暴露成了一个笑话。

    PS:平等主义心态是白人的发明,而且是一种妄想。 让自己摆脱它。

  154. Jason Liu 说:
    @Pat Boyle

    所有历史证据都表明火药是从东方传入欧洲的。 火药武器也是如此。

    • 回复: @Pat Boyle
  155. Santoculto 说:
    @utu

    他的自我矛盾令人尴尬,但这很常见,事实上,智力挑战的一部分就是始终与自我矛盾作斗争。

  156. Santoculto 说:
    @RaceRealist88

    我没说过这个..什么谬论? 一个?

    • 回复: @RaceRealist88
  157. @Santoculto

    是的,你做到了。 这个谬误是事后、故此的。

    • 回复: @Santoculto
  158. Santoculto 说:
    @RaceRealist88

    是的,在你的另一个现实中,我做到了……

  159. Santoculto 说:
    @RaceRealist88

    P1:寒冷的冬天发生在智商/智力增加之前

    an**

    这不是相反的*😉

    这句话没有道理。

    • 回复: @RaceRealist88
  160. @Santoculto

    是的,在你的另一个现实中,我做到了……

    返回并重新阅读您的评论。 你说寒冷的冬天先于智力的增加,也就是说寒冷的冬天导致了智商/智力。

    这句话没有道理。

    确实如此。 提高你的阅读能力。

    • 回复: @Santoculto
  161. szopen 说:

    你们知道吗? 我意识到我讨厌遗传论和环境假说。

    很长一段时间我嘲笑环保主义者,因为他们总是发明新的解释,而且几乎可以将任何证据解释为支持环境假说(“黑人因为自尊心低而在测试中得分更差!哦,他们有更高的自我”) -比白人更自尊?你看,这证明了他们是多么受到歧视,因为即使他们有很高的自尊,他们的得分却更差!”)。

    但前一段时间我认为遗传主义假说实际上也可以这样说。

    黑白差距并没有缩小——看,这证明了差距是不可改变的,也证明了生物学基础!
    黑人与白人之间的差距缩小了——我可以立即找到解释,为什么这实际上是遗传假设所预测的:(1)差距不是完全遗传的,所以它可以缩小1/3甚至一半(2)大约10%黑人婚姻中的大多数是黑人/白人夫妇,他们的孩子通常算作黑人,因此差距当然必须缩小!

    我一直在尝试思考无法用一种或另一种理论解释的事情,除了均值回归(或基因测试,现在是科幻)之外,我可以思考任何事情。

    • 回复: @Santoculto
    , @Santoculto
  162. Santoculto 说:
    @RaceRealist88

    不。 就是不行。 重读你但我放弃了,我已经解释了一千遍你还是不明白。

    我相信人类在从X地迁移到Y地的过程中变得“更聪明”/面向未来/面向抽象,当他们到达非常恶劣的地方时,他们停止进化他们的智力,因为太冷[或热或干燥]相反,选择更高的创造力,它会受到限制。

    更不可预测或更可变的是环境的“行为”更有可能选择更高的创造力[抽象的面向未来的思维/行为可塑性]。

    他们很可能进化了,或者更好,增加了[人]的比例和[这种特征的表达:抽象和面向未来的思维],特别是当他们到达温带环境时,正是因为这种气候是最可变的。

    我的主要问题是:人类如何能够在没有提高智力的情况下到达非常遥远的地方//抽象未来思维**

    • 回复: @Daniel Chieh
    , @RaceRealist88
  163. Santoculto 说:
    @szopen

    “黑白差距并没有缩小——看,这证明了差距是不可改变的,也证明了生物学基础!”

    因为黑人和白人的智商差距没有缩小,但这并不意味着

    是一成不变的 因为 它的生物学基础。

    它是不可变的

    个人水平

    目前为止。

    “让”“黑人”在数量上变得“更聪明”的唯一经过认证的方法 as “白人”将通过积极和消极的[认知]优生学[人们总是认为优生学只是为了提高智力,哈哈,纳粹优生学证明它并不总是这样]=改变这个群体的选择压力或降低白人的智商[在平均]。

  164. Santoculto 说:
    @szopen

    我认为“回归均值”的现象并不完全或严格地符合我们习惯的想法,但是……为什么一对认知能力较高的夫妇更容易生出认知能力下降的孩子,而不是认知能力提高的孩子*

    较高的认知技能和任何智力特征的其他类型的增加表达似乎与较高的身高相当,而较高的身高往往会发生相同的“回归”均值。 表型变异的最终边界和该表型的表达性。

  165. @Santoculto

    我相信人类在从X地迁移到Y地的过程中变得“更聪明”/面向未来/面向抽象,当他们到达非常恶劣的地方时,他们停止进化他们的智力,因为太冷[或热或干燥]相反,选择更高的创造力,它会受到限制。

    如果这是真的,我们至少需要一个合理的机制来实现这一点——也许温暖或炎热气候下的挑战更加多样化或“新颖”,因为个人无法通过死记硬背的学习或其他应对本能做好准备和。

    我不确定这是否属实。 温暖地区的生物多样性更多,但这是否会增加新挑战的数量? 一般的清洁会阻止它们吗? 如果有更多种类的敌对动物,标准化的防御形式会保护人类吗? 如果部落更容易分裂,然后阻止社会合作(这似乎对智力(大概是创造力)有最大的促进作用,那么它是否会促进更少的社会合作?)

    此外,我觉得“创造力”这个词的定义并不明确,需要进行更多研究。 根据芭芭拉·奥克利博士的学习理论,创造力似乎是知识分块的结果,然后知识被重新组合(通常是无意识的)来解决问题。 这表明缺乏自我抑制、广泛的常识以及我们尚未完全捕获的其他一些神经因素的某种组合。 也许甚至是持续的睡眠。

    • 回复: @Santoculto
    , @RaceRealist88
  166. Santoculto 说:
    @Daniel Chieh

    我不确定这是否属实。 温暖地区的生物多样性更多,但这是否会增加新挑战的数量? 一般的清洁会阻止它们吗? 如果有更多种类的敌对动物,标准化的防御形式会保护人类吗? 如果部落更容易分裂,然后阻止社会合作(这似乎对智力(大概是创造力)有最大的促进作用,那么它是否会促进更少的社会合作?)

    我认为,当我们的地方“一直”存在危险但气候本身可以补偿时,情况就会有所不同。 它选择短期思维来应对日常挑战。

    太冷的气候也会选择短期思维,但方式或程度不同。 因纽特人与任何热带人类群体一样都是“短期”/务实的思想家,但他们不在同一水平上。 此外,较低水平的社会复杂性,例如与不同的人打交道,往往会选择反对较高抽象的思想家。

    我认为短期思维本身可能会减少合作,因为它往往更本能/冲动,但我不认为大多数热带狩猎采集群体的群体内合作低于热带外群体。

    此外,我觉得“创造力”这个词的定义并不明确,需要进行更多研究。 根据芭芭拉·奥克利博士的学习理论,创造力似乎是知识分块的结果,然后知识被重新组合(通常是无意识的)来解决问题。 这表明缺乏自我抑制、广泛的常识以及我们尚未完全捕获的其他一些神经因素的某种组合。 也许甚至是持续的睡眠。

    我不一定认为创造力的定义不明确,它基本上是用不同的方式思考来解决已知或新的问题。 这本身确实是推理。 事实上,我们所拥有的一切都是由于创造力。 我的印象是因为自我驯化创造力的增加变得罕见和/或与我们所说的“智力”分离。 我们单方面理解的“智力”实际上是指“记忆/内化和使用创造性发明的能力”。

    通常环境决定生物的命运。 人类是唯一的,因为它有非常发达的自我意识,可以挑战环境在进化道路上的[通常]巨大影响。 我也不认为热带群体“没有创造力”,很明显他们有,但新的环境选择或增加了创造力,但也许不是所有人,而是某些人,如果每个人类社区都是等级制的。 或者,当一些部落提高了他们的[平均]智力时,那些更具创造力的部落也会遵循这种进化。

    我们还可以认为,当人类掌握某些环境时,他们是否倾向于探索新的土地,积累更多的文化/知识[并需要更高的记忆力来回忆它们]。 因纽特人和其他次极地群体继承了许多“动物”皮衣等文化习俗,并发明了“冰原”新闻。

    • 回复: @Daniel Chieh
  167. @Daniel Chieh

    如果这是真的,我们至少需要一个合理的机制来实现这一点——也许温暖或炎热气候下的挑战更加多样化或“新颖”,因为个人无法通过死记硬背的学习或其他应对本能做好准备和。

    Dobzhansky (1950: 221) 指出“热带环境比温带和寒冷地区的环境提供更多的进化挑战。=

    http://cyber.sci-hub.bz/MTAuMjMwNy8yNzgyNjMwNg==/10.2307%4027826306.pdf

    因此,由于极端寒冷/新奇的环境而在欧亚大陆进化出更高智慧的事实是站不住脚的。

    • 回复: @szopen
    , @notanon
    , @Wizard of Oz
  168. @Santoculto

    当他们到达非常恶劣的地方时,他们就停止进化他们的智力

    进化不会停止。

    更不可预测或更可变的是环境的“行为”更有可能选择更高的创造力[抽象的面向未来的思维/行为可塑性]。

    拉什顿希望你相信,在寒冷的气候下,它更容易预测,因此提前计划是可能的,从而增加智力。 这是寒冬理论的一部分。

    现代大脑/身体在移民之前就已经在非洲形成。 那是你似乎忘记的事情。

    特别是当他们到达温带环境时,正是因为这种气候是最易变的。

    您认为非洲有固定的气候(炎热和干燥)吗?

    我的主要问题是:人类如何能够在没有提高智力的情况下到达非常遥远的地方//抽象未来思维**

    它已经在非洲形成了。 一旦智人迁移到寒冷的气候,就会发生特定的适应(生理上的: https://notpoliticallycorrect.me/2017/07/30/human-physiological-adaptations-to-climate/)(和身体方面:宽大的骨盆以保持热量: https://notpoliticallycorrect.me/2017/07/04/homo-neanderthalis-vs-homo-sapiens-sapiens-who-is-stronger-implications-for-racial-strength-differences/).

    正如我在关于智人和尼安德特人之间力量差异的文章中所写的那样,智人的骨盆较窄,因此比宽骨盆的尼安德特人更快(每个骨盆都是对气候的特定适应),这也有帮助。

    总之, 人类以增加新陈代谢和隔热的形式进行的寒冷适应似乎是在最近的种群进化过程中发生的,但不能在温带和北极地区遇到的寒冷条件下在一生中发展起来。 因此,我们主要依靠我们的行为技巧来生活和度过寒冷.

    https://www.ncbi.nlm.nih.gov/pmc/articles/PMC4861193/

    • 回复: @Santoculto
    , @res
    , @notanon
  169. @Santoculto

    只有新颖的答案有助于解决这些问题,新的问题才会选择提高联想能力,但我不确定这是否属实。 如果“红色动物”的解决方案是避开它们,而“大型掠食性”动物的解决方案是防御/杀死它们,那么灰狼和恐狼同时存在的事实并不一定会选择创新方法,但围绕已知的解决方案进行神经支配。

    根据我对当前神经科学的理解,我们知道知识以神经上的“结节”的形式存在,然后将其分块,以便它们以一致的模式一起出现; 取决于使用一组知识的路径的频率,它所具有的白质或有髓鞘神经支配越多。 处理变得更快、更流畅。

    因此,为了找到新的解决方案,有些东西似乎必须存在:要混合的知识块、新的问题或目标以及一段时间的孵化。 某种想法产生器驱动着这个,可能与工作记忆有关(这似乎是一组可以保存概念的临时化学物质); 产生的大多数想法都是完全没有价值的,例如轻而易举地行走的梦想或其他大多是潜意识的胡言乱语,但有些想法将大量知识重新概念化为有用的方式。 这会产生“啊哈”的时刻,学习,我会说,整体创造力也是如此。

    这一切似乎都是一个相当敏感的过程,因此疾病负荷可能会对其产生相当不利的影响。

    • 回复: @res
    , @Santoculto
  170. Santoculto 说:
    @RaceRealist88

    进化不会停止。

    是的,有时或经常停止、倒退或保存。 我认为大多数时候物种会保护自己而不是进化,特别是当它们找到与环境的平衡时。

    您认为非洲有固定的气候(炎热和干燥)吗?

    很明显不是。 但没有大片大陆地区气候温和。

    拉什顿希望你相信,在寒冷的气候下,它更容易预测,因此提前计划是可能的,从而增加智力。 这是寒冬理论的一部分。
    现代大脑/身体在移民之前就已经在非洲形成。 那是你似乎忘记的事情。

    是的,你忘了完全注意我的评论,我说“我不认为‘非洲人’——没有创造力——”。

    我不认为他相信“寒冷的气候更容易预测”,任何消息来源*任何报价*

    我相信,由于季节振幅较低,这种类型的极端气候变得更容易预测,但分别以不同的方式。

    在温带和北极地区

    但温带地区往往与北极或极地有很大不同。

    • 回复: @RaceRealist88
  171. @Santoculto

    是的,有时或经常停止、倒退或保存。

    进化没有方向,它不是目的论的,所以你的话毫无意义。 进化只是发生,没有“目标”或“方向”。

    很明显不是。 但没有大片大陆地区气候温和。

    是的,但是整个大陆没有一种恒定的气候。 非洲有半湿润的地方,但我对这方面不太了解。 非洲有多种气候和生态系统。

    我不认为他相信“寒冷的气候更容易预测”,任何消息来源*任何报价*

    他做到了

    我相信,由于季节振幅较低,这种类型的极端气候变得更容易预测,但分别以不同的方式。

    正确的。 主观方式。 主观的方式,你可以硬塞东西以适应你想要表达的内容。

    但温带地区往往与北极或极地有很大不同。

    但它们更类似于北极和极地,而不是热带。

  172. @RaceRealist88

    进化没有方向,它不是目的论的,所以你的话毫无意义。 进化只是发生,没有“目标”或“方向”。

    你认为《老鼠乌托邦》中发生了什么?

    • 回复: @RaceRealist88
  173. res 说:
    @RaceRealist88

    因此,我们主要依靠我们的行为技巧来抵御寒冷。

    有趣的是,您认为这一点足够重要,以至于需要强调(粗体)。 您认为哪种行为技能最相关? 智力是其中之一吗?

    您认为日本的智商与纬度相关性的可能解释是什么?
    http://www.sciencedirect.com/science/article/pii/S0160289613000949

    感谢您之前引用 Eppig 的工作。 这是一个例子: https://www.researchgate.net/publication/44888621_Parasite_Prevalence_and_the_Worldwide_Distribution_of_Cognitive_Ability

    从摘要:

    当温度、与非洲的距离、人均国内生产总值和多项教育指标受到控制时,传染病仍然是国民平均智商最有力的预测因素。

    从表 1 可以看出,DALY 疾病(其对数)和冬季高温与平均智商的相关性分别为 -0.82 和 -0.72,并且彼此之间的相关性也为 0.71(两者还与距离的对数相关性约为 -0.4)来自中部非洲)。 因此,有证据表明疾病的重要性,但几乎没有证据反对寒冷冬季的重要性。 一个有趣的问题是,如果有这样的数据,每个因素如何与表型智商和基因型智商相关。

    表 3 显示了多元回归模型的结果。 DALY 疾病和冬季高温具有相似的 p 值。 一个有趣的问题(AFAICT 论文中未评论)是 DALY 疾病的 VIF(方差膨胀因子)为 6。根据本页,超过 4 的 VIF 表明多重共线性可能是一个问题,需要进一步调查: https://onlinecourses.science.psu.edu/stat501/node/347

    我希望他们做了方差分析。 也许还提出了其他模型,包括其解释变量的不同子集。

    讨论的摘录似乎相当合理,除了“大利润”之外,我认为这是对结果的夸大:

    多元回归表明,在传染病、温度、进化新颖性和 AVED 中,传染病在很大程度上是智力的最佳预测因子。 受教育年限的影响并不显着,而温度和进化新颖性似乎对传染病之外具有明显的预测能力。 尽管该模型不能排除距中非距离的独立影响,但由于对该变量的理论基础存在疑问,这种影响很难解释(Wicherts et al. 2010a)。 虽然在控制其他因素的情况下,教育和人均GDP的影响在统计上并不显着,但这并不是说这些因素不参与其中。 一个拥有更多聪明人的国家可能会产生更高的国内生产总值,但一个更富裕的国家也更有能力支付公共教育以及公共医疗和卫生服务。 教育和智力之间也可能存在间接联系,因为受过良好教育的人群可能对公共卫生措施更感兴趣——通过减少寄生虫压力来提高智商——前提是教育包括有关细菌理论和卫生的信息。 在解释我们的研究结果时必须考虑这些内生性来源(见下文)。 还应该提到的是,我们并不是说全球智力差异只是由寄生虫压力引起的。 相反,智力的差异可能是由多种因素引起的,包括我们在这里提到的因素以及未知的因素。

    注意最后两句话。

    下面是温度和 IQ 的对比: http://www.sciencedirect.com/science/article/pii/S0160289605000917
    该论文可从 libgen 获取。 该论文的一个很好的特点是,表 2 列出了所有国家/地区的数据,并明确说明了哪些 IQ 分数是计算得出的,哪些是估计值(表 4 给出了两个亚组以及所有国家/地区的相关性)。 这些数据可以方便地重现他们的分析。 看起来只需一点努力就可以将其转换为电子表格。 也许我应该自己做方差分析等?

    纸质 PDF 方便地包含上面链接中提到的三个注释。 我认为后两者提供了这里争论的一个很好的例子。 特别是,部分 4. 结论性意见 亨特和斯滕伯格的回应清楚地表明了“热点按钮”在哪里。

    有趣的是,这篇论文还包含了关于有争议论文的编辑说明 http://www.sciencedirect.com/science/article/pii/S0160289605000954
    该文件的 PDF 也在 libgen 中。

    • 回复: @res
    , @RaceRealist88
  174. res 说:
    @Daniel Chieh

    这一切似乎都是一个相当敏感的过程,因此疾病负荷可能会对其产生相当不利的影响。

    您能否从以下方面详细说明您对此的看法:
    1. 智力的进化压力。
    2. 个体的表型智力。

    对我来说,疾病负荷似乎与 2. 比与 1. 更明显相关。 然而,我对为什么疾病负荷可能影响智力进化的争论感兴趣。

    我将首先提供一个。 疾病对表型的影响可能会减轻代谢消耗较高的智力的进化压力,从而导致代谢消耗剩余,同时降低由此产生的智力。 换句话说,由于疾病负荷而无效的大大脑在代谢上仍然是昂贵的(并且保留了大大脑的骨盆尺寸问题)。

    • 回复: @Santoculto
    , @Daniel Chieh
  175. Santoculto 说:
    @Daniel Chieh

    只有新颖的答案有助于解决这些问题,新的问题才会选择提高联想能力,但我不确定这是否属实。 如果“红色动物”的解决方案是避开它们,而“大型掠食性”动物的解决方案是防御/杀死它们,那么灰狼和恐狼同时存在的事实并不一定会选择创新方法,但围绕已知的解决方案进行神经支配。

    嗯,我认为身体适应也可以被描述为“创造力”,如果所有新颖的适应都是创造性的,创造性地思考作为一个大概念,但我们谈论的是人类抽象思维,预测性/文化性,而不是长期直接引起的身体适应。术语环境相互作用/压力。

    非人类是由他们所处的环境选择/适应或进化的。 即使这个过程也发生在人类身上,与其他生物相比,“我们”对我们周围的环境有更大的认识,并且可以以适应的方式预见、预测或思考。

    这一切似乎都是一个相当敏感的过程,因此疾病负荷可能会对其产生相当不利的影响。

    所以你认为在高疾病负荷下不可能变得更聪明*

    • 回复: @Daniel Chieh
  176. res 说:
    @RaceRealist88

    进化没有方向,它不是目的论的,所以你的话毫无意义。 进化只是发生,没有“目标”或“方向”。

    我不同意。 我认为说这两点是非常有意义的(是的,我知道其中存在哲学上的争论):

    – 进化的目标(也许“结果”更好?)是在给定环境中提高适应性。
    – 特定群体在特定时间的可测量特征的演变很可能有一个方向。 这个方向不必在不同时间或群体中保持不变。

    我们可以争论桑托库托在给你的回复中所说的细节:

    进化不会停止。

    是的,有时或经常停止、倒退或保存。 我认为大多数时候物种会保护自己而不是进化,特别是当它们找到与环境的平衡时。

    但恕我直言,他提出了一个合理的(而且很重要的!)观点。

    胡椒蛾的进化很好地说明了他的观点: https://en.wikipedia.org/wiki/Peppered_moth_evolution

    您是否不同意所观察到的蛾颜色的方向演变及其逆转?

  177. szopen 说:
    @RaceRealist88

    尽管如此,不同的气候似乎可能会导致物种内大脑结构的差异:

    Timothy Roth 等人“不同气候下储存食物的鸟类海马神经胶质细胞数量的变化”
    另外,早些时候(2010)
    Timoty Roth 等人“在恶劣环境中增强学习能力:常见的花园方法”
    http://rspb.royalsocietypublishing.org/content/early/2010/05/29/rspb.2010.0630?sid=1580b0b0-1c12-4ba1-abd8-2e0bc92fd7fd

    现在,你当然会说,对于储存食物的鸟类来说,情况是不同的,拥有更好记忆力的压力与提高智力的压力并不相同,而且他给鸟类的谜题可以说并不能衡量一般智力[1] 。 但考虑到它是种内差异,并且保存在实验室饲养的鸟类中(因此,很可能是遗传性的),而且它不太可能是由气候温暖的山雀的种族主义引起的,这似乎确实表明,更恶劣的气候有时可能会造成至少对某些物种来说,更多的挑战会导致明显的大脑结构差异,不是吗? 关于较短的白天(因此寻找食物的时间更少)的观点也适用于人类。

    OTOH,“两种不同的极端环境”似乎不太令人信服。

    [1] 摘自2010年论文:

    解决问题的测试涉及从钻的 3.5 厘米井的 1.5 × 15 网格中取出覆盖有透明 3M 醋酸盐的镀锌钢垫圈(直径 3 厘米,直径 5 厘米的孔;大致等于家禽的质量,约 1.5 克)放入装有蜡虫的木板(40 × 18 厘米)中

  178. Santoculto 说:
    @RaceRealist88

    进化没有方向,它不是目的论的,所以你的话毫无意义。 进化只是发生,没有“目标”或“方向”。

    是的,但这并不意味着进化是完全随机的,就像一只鸟在一两代内进化成人类一样......

    所以进化不会停止,也没有方向…… 不是*

    “进化不会停止”=目的论*

    是的,但是整个大陆没有一种恒定的气候。 非洲有半湿润的地方,但我对这方面不太了解。 非洲有多种气候和生态系统。

    所有气候在某种程度上都是不稳定的“或”多变的,至少在其典型的天气行为中是如此,但我说的是变化更大的气候:温带。

    正确的。 主观方式。 主观的方式,你可以硬塞东西以适应你想要表达的内容。

    嗯。 不。

    不同且不主观。

    寒冷的气候是持续存在的,但是,如果你不适应或不采取保护措施,天气很冷,空气已经是致命的。

    平均而言,温暖的气候、赤道甚至海洋热带气候都是恒定的。 你不会因为呼吸热带或赤道空气而死亡。

    寒冷的气候本身就是挑战,热带气候则不然。 温带气候[比寒冷地区]具有更多的生物多样性以及严寒时期和更大的热幅。

    但它们更类似于北极和极地,而不是热带。

    也许您将温带[海洋和大陆变体]与寒冷气候混淆了,寒冷气候是极地气候和温带气候之间的中间气候,在加拿大/卡克纳达的大部分地区都有发现。

  179. Santoculto 说:
    @res

    如何解释热带地区人类大脑数量的增加*

    • 回复: @res
  180. res 说:
    @Santoculto

    我不确定我是否理解你的观点。 我的疾病负荷(或感冒)和进化思想更关心不同的速率。 因此,热带(温暖且疾病较多)地区的智力可能会提高,但速度比寒冷(且疾病较少)地区慢。

    PS 以下是世界卫生组织按国家列出的 DALY 数据,供需要的人参考: http://apps.who.int/gho/data/view.main.ENVDALYSBYCOUNTRYv

    • 回复: @Santoculto
  181. @Santoculto

    嗯,我认为身体适应也可以被描述为“创造力”,如果所有新颖的适应都是创造性的,创造性地思考作为一个大概念,但我们谈论的是人类抽象思维,预测性/文化性,而不是长期直接引起的身体适应。术语环境相互作用/压力。

    我指的是对环境危害的文化适应,即认识到红色动物是危险的并避免它们。 继续这个例子,如果你住在一个有危险动物的地方,并且你想出了“刺伤美洲虎”的想法,这样你就不会失去你猎取的食物,这是解决问题的一种简单形式。 如果你想出了“大声喧哗来吓唬动物”的想法,那就是解决问题的另一种形式。 如果你想出了“放火”的想法,那又是另一回事了。

    上述所有例子都是个人聪明才智在成为文化知识之前的某个阶段的例子。 非人类的例子很少见,但我们确实在猕猴身上看到了一些类似的例子,它们教它们的孩子“将红薯浸入海水中”以改善味道。 这是最近发生的事情——人们第一次观察到这种现象是在 1950 世纪 XNUMX 年代,随后父母将这一知识传授给了自己的孩子,现在岛上所有猕猴都普遍这样做。 我想说,第一只想到“将食物浸入海水中”的猕猴表现出了创造性思维,不是吗?

    所以你认为在高疾病负荷下不可能变得更聪明*

    我认为脑干,即“爬虫类大脑”,是一个例子,说明如果某些事情变得难以改变,进化就会“停滞”。 它是大脑的一部分,控制心跳、呼吸和其他基本生命过程等基本功能。 它在整个开发过程中大部分都保持不变。 任何改变都可能导致致命的结果,这就是为什么人类和灵长类动物继续使用并未完全适合高度依赖视力的生活方式的脑干进行操作。

    同样,我认为如果学习过程像上面 B. Oakley 的工作所示的那样复杂,那么可能影响其一个或多个部分的疾病负荷可能会使其脱轨。 它不会像“高病使人哑”那么简单,而是增加了难度。

    • 回复: @Santoculto
  182. Pat Boyle 说:
    @Daniel Chieh

    我不明白你的言论与我的相反。 也许是我说不清楚。 君士坦丁堡和哈弗鲁尔是十五世纪的围城战,枪支和火药在其中发挥了至关重要的作用。 这就是我的观点。 中国人利用硝石资源与蜂蜜混合来制造一系列烟花。 这是相当早的,但它并没有直接导致枪支和火药的出现,从而摧毁了堡垒的墙壁,也没有直接导致了手炮和小型武器的出现。

    十三、十四世纪,火药遍布欧洲,但并没有多大用处,因为它作为战场武器的性能较差。 将一批自制的黑火药混合起来,整天放在火药喇叭里随身携带,当你去使用时,它会再次分离而毫无用处。

    真正的火药不仅仅是简单的混合物。 它是由皇家授权工厂生产的制成品。 国家对真正的(咸)火药的生产实行严格控制。 该过程的一部分是制作饼,然后将其研磨至适合特定种类火药的适当稠度。 如果你在打磨时不小心——繁荣。

    军事历史学家普遍认为查理八世的意大利战役结束了高墙式防御工事,这种防御工事一直是自杰里科以来抵御敌人的标准手段。 这次战役标志着火药时代的开始。 您可以通过墙壁的风格来追踪火药传入某个地区的时间。 请注意,中国的长城虽然在基督之前就开始修建,但直到 XNUMX 世纪才开始修建(尼克松和游客参观的部分)。 这么晚了,欧洲还没有以这种方式建造新的重要防御工事。

    所以你可能会说,中国的长城证明了中国没有发明火药。 (也许这样的说法有点强硬)。

    • 回复: @Daniel Chieh
  183. Pat Boyle 说:
    @utu

    你对“事实”的构成有一个有趣的概念。 中国人发明火药的观点在大众媒体中很流行,但在严肃文学中却并不流行。 请阅读一些军事历史。 请考虑其他“发明”。 你不仅错了,而且还没有礼貌。

  184. Santoculto 说:
    @res

    大脑尺寸的增加一直是人类智力增加的标志,据我们所知,人类首先是在热带地区进化的,传染病负荷较高。 我说的是自第一个原始人类以来大脑尺寸/智力的首次增加。

    • 回复: @Santoculto
  185. Pat Boyle 说:
    @Jason Liu

    你说的都是非历史的。 硝石在欧洲并不像在中国和印度那样常见。 确实,硝石早在西方就已在东方得到开发。 但火药是十五世纪欧洲发明的一种特殊制成品。

    真正的火药——一种可用于枪支的粉末状物质——在大多数地方都是皇家垄断的,由专门的工厂生产。 它的确切制造过程一直是国家机密。 许多世纪前,中国就已经制造了硝石和蜂蜜烟花,但那不是火药。

    李约瑟(Joseph Needham)是一位恶毒的反美共产主义者,他可能是许多亲华宣传的来源。 在历史的大部分时间里,中国可能一直是最先进的文明,而且可能会再次如此。 中国人在针对欧洲人的标准化智商测试中似乎表现得非常好。 毫无疑问,十五世纪之前的中国拥有许多(如果不是大多数)世界各地的重要发明。 但中国人并没有发明一切。 他们并没有发明火药,除非你拷问“发明”这个词的定义。

    • 回复: @res
  186. Santoculto 说:
    @Daniel Chieh

    我指的是对环境危害的文化适应,即认识到红色动物是危险的并避免它们。 继续这个例子,如果你住在一个有危险动物的地方,并且你想出了“刺伤美洲虎”的想法,这样你就不会失去你猎取的食物,这是解决问题的一种简单形式。 如果你想出了“大声喧哗来吓唬动物”的想法,那就是解决问题的另一种形式。 如果你想出了“放火”的想法,那又是另一回事了。

    好吧,我现在明白了。

    上述所有例子都是个人聪明才智在成为文化知识之前的某个阶段的例子。 非人类的例子很少见,但我们确实在猕猴身上看到了一些类似的例子,它们教它们的孩子“将红薯浸入海水中”以改善味道。 这是最近发生的事情——人们第一次观察到这种现象是在 1950 世纪 XNUMX 年代,随后父母将这一知识传授给了自己的孩子,现在岛上所有猕猴都普遍这样做。 我想说,第一只想到“将食物浸入海水中”的猕猴表现出了创造性思维,不是吗?

    是肯定的。

    “同样,我认为,如果学习过程像上面 B. Oakley 的工作所示的那样复杂,那么可能影响其一个或多个部分的疾病负荷可能会使其脱轨。 它不会像“高病使人变笨”那么简单,而是增加了难度。”

    即使我不怀疑疾病负荷有一定的影响,我也怀疑它是否会成为最重要的因素,当然,如果得到证实,我完全愿意改变我的观点。

    低认知能力的病理学过程正在持续进行,就像是一种疾病或一种精神状况一样。

    也许我们可以争论,当有机体暴露于更多的微型入侵者时,它往往会额外适应它们[并且用更少的时间进化到更高的智能],而在微型人口较少的地方,有机体,我的意思是,所有或大多数,将更加自由地发展,更少的依赖,更少的担忧因素。 我们在其他物种中负荷疾病和智力之间有一个非常常见的趋势或相关性** 这是一个普遍趋势*

  187. Santoculto 说:
    @Santoculto

    ......在传染病负荷较高的地方,我想说的是 似乎,这个因素不会阻止早期人类智力的进化。

  188. Pat Boyle 说:
    @RaceRealist88

    桑托库托只是指的是一个常见的理论,即为什么离开非洲并北上的人们也变得聪明。 当然,这个理论可能是错误的,但这并不是逻辑谬误。 他可能是大多数。 许多人类学家也这么认为。

    你说得对。 也许读者对拉丁语的理解不如我想象的那么好。 但这个博客的读者智商非常高。 我总是在这里学到新东西。

    我记得引用过—— 事后,故此 ——一位正在教授基础训练课程的中士。 我在军队里遇到了麻烦。

    • 回复: @Okechukwu
  189. res 说:
    @Pat Boyle

    除了我之外,你似乎已经对这个话题的所有人做出了回应(评论149)。

    在我看来,你在这里玩的是“没有真正的火药”。

    https://en.wikipedia.org/wiki/Timeline_of_the_Gunpowder_Age
    https://en.wikipedia.org/wiki/Fire_lance

    我当然不同意你关于欧洲对火器和大炮技术的贡献及其对防御工事的影响的重要性的说法。 只是那些并不是火药的发明。

    • 回复: @RaceRealist88
  190. @Daniel Chieh

    你认为《老鼠乌托邦》中发生了什么?

    我不知道那是什么。

    • 回复: @Daniel Chieh
  191. Okechukwu 说:
    @K-Drama Kumiho

    Okechukwu,曾经和未来的康,写道:

    你就是表现出康心态的人。 你是那个情绪过度紧张的人,因为我不愿意认可你那可笑的错位的优越感。 你似乎在恳求肯定,希望能改善个人的缺点。 像你们这样的人实际上是一项有趣的心理学研究。 这是一种被低自尊所包裹的精神病,也许是某种创伤后压力,或者可能是未实现的野心,最终导致替罪羊。

    “石器时代”是一个具有实际人类学意义的概念。 韩国人从事金属加工已有两千多年的历史,因此我们并不是定义上的“石器时代”。

    是的,我意识到韩国人并不是真正的新石器时代民族。 当然,石器时代在这种情况下只是一种比喻。

    你的侮辱的微不足道和绝望是不言而喻的。 这次爆发让你对其他无端的反韩言论有了正确的认识。

    但我的帖子是防御性的和反应性的。 你就是我要回应的始犯。 在把你们撕成碎片之前,我确实向所有正派的亚洲人道歉。

    显然,您需要向您指出,历史并不是欧洲人开始在撒哈拉以南非洲施加影响时才开始的。

    不,我不。 请记住,我写了以下内容:

    在此之前,非洲人确实与欧洲军队和定居者交战并击败了他们。 欧洲人被限制在沿海的小飞地,充当非洲国王的附庸。 欧洲人以平等的条件与非洲人进行贸易,因为他们不能用武力夺取任何东西,包括奴隶。 他们试图袭击非洲村庄寻找奴隶,但遭到惨败。 随后他们签订了条约并支付了奴隶的费用。

    您关于亚洲历史的言论表明,您对欧洲殖民主义及其在亚洲的影响力的无知,以及对毁灭性的亚洲内部冲突的严重无知。

    我认为你没有敏锐的洞察力来认识到你是多么虚伪得可怕。 难道你们荒谬的意识形态通常不认为殖民主义是一种介于良性和仁慈之间的力量,至少对于非洲来说是这样吗? 因此,现在看来,殖民主义阻碍了亚洲的国家发展,但没有阻碍非洲的国家发展。 那是对的吗? 您打算将亚洲的失败视为殖民主义和冲突的副产品吗?

    嗯,我发布了数十项亚洲发明、发现和诺贝尔奖获得者的链接,以及通过同行评审出版物对人均科学产出的一般衡量标准。 你提到了四名非裔美国人的名字。

    这四位非裔美国人创新和发明的现代技术比整个亚洲的总和还多。 我能说出的远不止四个。 但只需要四个即可。

    哇,谈论无耻的双重标准。 尽管在白人占绝大多数的环境中工作,你们的非裔美国人却得到了你们的充分、不可分割的赞扬,但韩国人却没有因为我们的贡献而得到赞扬。 你还能再显得更加痛苦和绝望吗?

    非裔美国人文化 IS 美国文化。 将其中一个与另一个分开是不可能的。 这让我想起,我们真的应该要求发明爵士乐、摇滚乐、灵魂乐、蓝调、嘻哈、说唱等音乐的人们在智力方面落后于韩国人、中国人或其他任何人吗? 我想不是。 我认为那些必须复制他们的东西的人都是低劣的人。

    你的问题只是表明你不了解群体平均值如何与任何特定个体的状态相比,也不了解混合如何影响基因调节的行为。

    这些都是愚蠢的、重复的、胡言乱语的谈话要点。 首先,你对平均值一无所知。 还没有进行过全球性、大样本、文化中立或文化包容性的全球情报调查。 为了合理地回答这些问题,你必须在适当的科学控制下的某个岛屿上抚养不同种族的健康孩子。 让他们接触相同的文化、教育、饮食等。然后你就会得到你的答案,但你不会喜欢这个答案。 但除了不合格的道德问题之外,还有一个简单的事实:这将是彻头彻尾的时间浪费。 因为通过外在表现来评估一个人的智力需要一种特殊的愚蠢。 换句话说,除非你非常愚蠢,否则你应该几乎本能地知道种族和智力完全无关。

    顺便说一句,大自然不会做平均值。 有机体要么有某物,要么没有。 这就是为什么没有一只黑猩猩会做代数或操作汽车。

    我并不是对你说任何不寻常或非凡的事情。 你经常出现的互联网泡沫和回声室往往会强化一种错误的现实感。 我就是现实。 如果你带着你的可怜的屁股去实体科学会议,你会发现我就是这样的人。 在你们自己的韩国也这样做,那里黑人很少。 让我们看看你解释、展示和量化黑人为何不如白人和亚洲人。 事实上,请将我与任何与您观点相同的可信韩国科学家、遗传学家或院士联系起来。 祝你好运。 你会需要它。

    • 回复: @gda
  192. @res

    今晚我会回复你和其他人。 我现在所在的地方的 Wifi 很糟糕,而且我不喜欢输入长评论,因为我必须在移动设备上来回查看不同的内容。

  193. Okechukwu 说:
    @Pat Boyle

    桑托库托只是指的是一个常见的理论,即为什么离开非洲并北上的人们也变得聪明。 当然,这个理论可能是错误的,但这并不是逻辑谬误。 他可能是大多数。 许多人类学家也这么认为。

    然后说出一位相信这种无稽之谈的可信人类学家的名字。

    请停止编造事情。 看到这一点令人痛苦,尤其是你所声称的内容显然是错误的。

    • 回复: @Pat Boyle
  194. notanon 说:

    我认为有两个阶段

    1)耕种前:物理选择,例如漫长寒冷的冬季迫使选择更高的智商和更低的时间偏好

    2)农业之后:通过人口密度进行的文化选择导致社会更加复杂,对更多认知角色的竞争更加激烈

    我认为生物群落的想法适用于这两种情况,但分开:

    1)狩猎采集者的基线智力和时间偏好随着纬度从赤道向北(或向南)或多或少呈线性增加

    2)文明层基于每个生物群落适合高人口密度农业的难易程度以及随之而来的在更复杂的社会中争夺最舒适地点的竞争。

    例如,在第一阶段,智商范围为 70-85,赤道为 70,而远北地区的爱斯基摩人(ANE?)是最聪明的(但仍然不是很聪明),智商为 85,另外 15 个智商点如果特定生物群落中的人们能够发展高密度农业(中纬度生物群落首先到达那里),则可以从文明中获得。

    如果正确,则叠加第一阶段和第二阶段生物群系可能会产生更好的配合。

  195. notanon 说:
    @RaceRealist88

    进化没有方向,它不是目的论的,所以你的话毫无意义。 进化只是发生,没有“目标”或“方向”。

    当然可以:最大可能的寿命

  196. Factorize 说:

    丹尼尔,谢谢你的回复。

    我认为这个帖子上的海报应该更多地关注基因技术,这些技术将极大地重塑我们的未来,并且现在正在迅速逼近。 我不清楚为什么这么多人投入如此巨资来猜测数千年前可能发生的事情。

    人类智力在历史上从未有过超过 1 点的 IQ 点。 这种发展的社会后果将是深远的。

    几千年来,所有人类群体中都存在着稳定的阶级均衡结构。 那些智商相对优势为 1 SD 的人为那些不幸的人提供了教育、医疗、法律和其他服务。 存在共生关系。 将 IQ 添加 4 SD 会将孩子从队列后面移动到队列前面。 今天,基于智商的生活是高度可预测的,即使在多代人的时间尺度上,这很快将不再是事实。

    这对我们的社会意味着什么尚不清楚。 一种可能的结果是,即使增加一些(3 或 4 SD IQ)也可能导致监管状态的解除。 有些社区需要近乎持续的培育才能帮助其成员度过一生。 大量增加智商将导致世界各国政府退出服务提供。

    另一个完全合理的结果是,群体智商的巨大差异很快就会变得明显。 提高智商需要一定程度的资源(尽管不一定过多)。 一般来说,亚洲人和白人都可以接受此类手术。 然而,我想知道撒哈拉以南地区的人口能够在多大程度上获得这种机会。 大规模的智商差异可能会产生毁灭性的后果。 具有极端智慧的人可能能够消除非洲出口中的大部分(如果不是全部)附加值。

    {顺便说一句,让我印象深刻的是,在过去的几千年里,所有人类社区都具有心理测量意识。 为了形成阶级结构,必须深入了解人类智能的本质。 在 Spearman 1904 年突破性的论文中,他将 g 确定为一个数学上可导的量。 然而,这是基于询问老师根据智力对学生进行排名。 因此,从某种意义上说,斯皮尔曼的结果只是重申了教师们很早以前就已经意识到的事情,可能可以追溯到几千年前。 根据这些表型信息,一个基于智力等级的社会出现了,其典型范围为 2 或 3 SD。}

    这是过去 3000-4000 年来欧亚人认知能力提高的选择参考。

    https://www.biorxiv.org/content/early/2017/02/28/109678

  197. notanon 说:
    @RaceRealist88

    您认为非洲有固定的气候(炎热和干燥)吗?

    可能有无数的环境全年都可以获得食物,但我认为关键因素是拥有一个一年中大部分时间都无法获得食物的环境 *需要* 生存时间偏好较低的演变。

    就我个人而言,我想知道较低时间偏好的进化是否是放大智力进化的好处并从而加速其发展的关键先兆?

    • 回复: @Okechukwu
  198. @res

    这似乎很公平——而且可能只是因为在疾病负担极高的环境中,提高智力的压力可能并不重要; 例如,如果规划只可能在三十年内显现出效益,而平均寿命没有那么长,那么它会选择抗病能力而不是智力。

    这种差异可能是为了负重而饲养的马与在湿地生活的马之间的差异。 后者可能会进化出对湿蹄病的更强抵抗力,但它们极不可能像前者一样强壮,尽管力量也可能对它们有利。 它只是对他们在环境中没有多大帮助。

  199. notanon 说:
    @RaceRealist88

    Dobzhansky (1950: 221) 指出,“热带环境比温带和寒冷地区的环境提供更多的进化挑战。”

    http://cyber.sci-hub.bz/MTAuMjMwNy8yNzgyNjMwNg==/10.2307%4027826306.pdf

    因此,由于极端寒冷/新奇的环境而在欧亚大陆进化出更高智慧的事实是站不住脚的。

    它不需要更困难的挑战,只需要新的挑战。

    假设 A 组进化出足够的智力来应对最初的环境 A,然后停止,而脱离组转移到环境 B,环境 B 面临新的挑战,而聪明的人首先适应,从而每次将平均智商提高一两点 – 所以最北端的人口站在前面所有层的肩膀上。

    从逻辑上讲,我认为这可能以两种方式发生:1)随着人口迁移到每个新环境而逐渐增加,或2)冬季迫使较低时间偏好的进化,而较低时间偏好会放大智力的进化(或两者兼而有之) )。

  200. Okechukwu 说:
    @notanon

    可能有无数的环境全年都可以获得食物,但我认为关键因素是拥有一个一年中大部分时间都无法获得食物的环境 *需要* 生存时间偏好较低的演变。

    你为什么不解释一下为什么北半球一年四季都没有食物呢? 事实上,寒冷和冰块是很好的防腐剂。 食物可能被埋在永久冻土中。 洞穴可以成为原始的冰箱。 动物将被保存在倒下的地方,提供丰富的拾荒机会。

    我无法夸大冷藏和冰在保存食物方面的巨大优势。 考虑一下我们最近遇到的一些自然灾害。 由于断电和缺乏制冷,人们可能会挨饿。 字面上地。 例如,为了预防飓风和潜在的停电,人们被要求收集尽可能多的冰,以便能够尽可能长时间地保存食物。 人们还被要求收集尽可能多的水。 这些警告和诉状应该可以帮助您了解哪种环境最有利于维持充足的粮食供应。 温暖会破坏食物,寒冷会保存食物。 水当然至关重要,北半球拥有丰富且容易获取的水。

    • 回复: @notanon
  201. utu 说:
    @Factorize

    Spearman 在 1904 年发表的突破性论文中,他能够将 g 建立为数学上可导的量。

    他只是偶然发现了协方差矩阵的性质,但没有意识到协方差矩阵始终是对称的和正半定的,这意味着特征值是非负的,并且最大特征向量使瑞利商最大化,即解释了方差。 换句话说,总有一些因素可以解释大部分方差。 斯皮尔曼的认识的深刻性类似于茹尔丹先生的发现:

    茹丹先生:哦,真的吗? 所以当我说:妮可给我拿拖鞋,拿睡帽来,”这是散文吗?

    哲学大师:最清楚了。

    JOURDAIN 先生: 嗯,你对此了解多少! 这四十年来,我不知不觉地一直在用散文说话!

    ——莫里哀,《资产阶级绅士》,1670

    • 哈哈: Daniel Chieh
    • 回复: @res
    , @szopen
  202. @Factorize

    嗯,我实际上还没有听说过任何可以让基因编辑达到超过 1 SD 的程度的东西,可能是通过胚胎选择。 深圳华大基因仍在研究这个问题,我相信这可能是一个比我们想象的更复杂的问题。

    也就是说,我们可能会看到更多的精英分类,因为来自更注重认知价值文化的孩子继续从父母和政府的帮助中获得最大利益; 例如,当我发现我的妻子怀了我的儿子时,我自己做了研究,并且几乎尝试了尽可能地控制营养等变量,以便我们的儿子希望尽可能健康和聪明(这个包括增加 omega-3 的摄入量 http://journals.sagepub.com/doi/full/10.1177/1745691612462585 )。 我确信我不是唯一一个这样做的人,而且教育在亚洲是一个高薪且受人尊敬的领域,因此我确信任何可以干预以提高那里的能力的事情都会找到受众,而且肯定会在众多的万金油推销员中,有些方法可能确实有效。

    我个人怀疑,这将导致世界变得更加分层,并可能因精英过剩而引发更多冲突。 然而,很难做出预测,尤其是对未来的预测。

  203. res 说:
    @res

    我根据 Templer 和 Arikawa 表 2 数据创建了一个电子表格,并从上面给出的链接添加了 WHO 传染病 DALY 数据。 这样我基本上就能够重新创建这两篇论文的主要结果。 我做了一些额外的散点图来观察异常值,我看到的最引人注目的事情是亚洲国家高智商异常值的频率(相对于冬季平均高温、记录的 DALY 和肤色)。

    退一步来说,肤色和智商之间的相关性为-0.92,这让我感到惊讶。 Templer 和 Arikawa 的肤色数据来自 https://www.amazon.it/Razze-popoli-della-Renato-Biasutti/dp/B00GZKSDBW
    比亚苏蒂,R. (1967)。 《Le Razze ei Popoli Della Terra》。 意大利都灵:Unione pipografiza-Editrice Torinese。

    我发现他们的推理很有趣:

    肤色被概念化为与温度密切相关的变量。 本作者将其视为祖先数千年来生活的气候的多代反映。 预测智商与温度和肤色相关性的另一个原因是 Beals、Smith 和 Dodd (1984) 报告的颅容量与距赤道距离之间的乘积矩相关性为 0.62。 比尔斯等人。 他们的发现基于来自各大洲的 20,000 个个体颅骨,代表 122 个不同种族的人群。 Jensen(1998)推断,自然选择会青睐较小的头部和较小的球形,因为在炎热的气候下散热效果更好。 寒冷气候下的自然选择会青睐更球形的头部,以容纳更大的大脑并更好地保存热量。

    关于伤残调整生命年和一般传染病,我早些时候曾质疑过这在进化中可能有多重要。 我还要指出的是,我怀疑 DALY 与广泛使用的第一世界药物的存在高度相关。 鉴于此,观察到的相关性有多少来自以下反向因果路径:
    智商 -> 存在第一世界医学 -> 低 DALY ?
    我很确定智商不会导致(历史上的)寒冷天气,所以这似乎是支持寒冷天气假说的一个论据。

    • 回复: @Afrosapiens
    , @James Thompson
  204. res 说:
    @utu

    主导所解释的方差的单个主成分(又名 g)的存在纯粹是巧合。 知道了。

    • 回复: @utu
  205. notanon 说:
    @Okechukwu

    你为什么不解释一下为什么北半球一年四季都没有食物呢? 事实上,寒冷和冰块是很好的防腐剂。 食物可能被埋在永久冻土中。 洞穴可以成为原始的冰箱。

    是的——只要人们在冬季到来之前提前收集和储存食物,即较低的时间偏好,那么全年都可以获得食物

  206. @res

    热带地区所谓的“外来”疾病早已被旅行者、探险家等以及医生所注意到。 一个明显的原因是,全年气候炎热,降雨量较多,直接影响繁殖地的形成,自然宿主的数量和种类较多,以及可传播给人类的动物疾病(人畜共患病)是最大的威胁。可能传播疾病的昆虫媒介的数量。 较高的温度也可能有利于病原体在生物有机体内部和外部的复制。 社会经济因素也可能在起作用,因为世界上大多数最贫穷的国家都位于热带地区。 巴西等热带国家改善了社会经济状况,并在卫生、公共卫生和防治传染病方面进行了投资,在消除或减少其境内许多热带地方病方面取得了显着成果。

    https://en.wikipedia.org/wiki/Tropical_disease#Relation_of_climate_to_tropical_diseases

    咄!

    • 回复: @res
  207. Factorize 说:

    丹尼尔,

    我的评论与围绕以下帖子 133 的讨论有关:
    https://www.unz.com/jthompson/the-hsu-boundary/

    人类g值大幅增加的潜力对我来说还是很新鲜的。
    人类智商提高 100 个标准差将深刻改变我们的生活。

    基于遗传评估的仔细择偶可以提高智力,正如您提到的,在 10 个胚胎中进行选择将使智商提高 1 个标准差。 CRISPR 和其他基因编辑技术显然正在逼近。

    配子的修饰和/或选择将是另一项强大的技术。
    我非常有兴趣知道我可以生产的配子的优化基因 g 可能是什么。 然而,考虑到大约有 2^^23 种可能性,我怀疑这样的配子在与类似选择的另一性别配子受精时会表现出非常极端的表型。

    在一个广泛定义的群体中,人类平均智力可能需要数千年的时间才能提高 1 个标准差。 现在,使用简单且相对便宜的技术,每代人的智商提高 1 SD 已经完全合理。 我们现在正处于人类能力不断增强的新时代的黎明。

    心理测量领域的这种变化将对精英产生重大影响。 现在越来越清楚的是,每个人,不仅仅是精英,都具有天生的遗传潜力,可以创造出具有非常高认知能力的孩子。 可以选择任何父母的智商增加 1 SD 的相同技术也可以用于选择每代人的智商增加 2 SD、3 SD、4 SD、5 SD……。

    范围限制问题掩盖了当智商很快开始稳步向平流层迈进时会发生什么。 到目前为止,高端智商是有上限的。 当平均智商开始上升时,智力的社会价值就会消失。 在高端智商无限供应的世界中,它的预期均衡价格可能是多少?

    然而,如果人类对提高智商感到拘谨,那么我认为可以做出全面的努力来优化其他表现出心理测量学的动物的智力,例如小鼠、大鼠、乌鸦、狗、猫、浣熊、黑猩猩、大猩猩和别的。 宇宙中没有任何特定的法则规定人类应该永远是宇宙中已知的最聪明的生物生命形式。 如果我们对提高智力不感兴趣,那么其他动物的基因工程就可以用来取代我们。 如果相信我们在生物秩序中的地位在某种程度上不受这种审查,那就是物种歧视。

    如果老鼠的智商增加 100 SD,它的智商会如何? 如果没有适用于人类研究的相同道德约束,我认为天才级别的老鼠或黑猩猩随时都会出现。 我们的世界可能充满了超级聪明的动物,而不是那么聪明的人。

    人猿星球!
    只是不要让那些毛茸茸的大猩猩(不想说,但丑陋)生气。

    当然,计算机已经达到了与人类相当的计算能力。
    预计未来几年将推出售价 1,000 美元的家庭版。

    令人遗憾的是,与推进人类智商增强相关的犹豫不决仅仅意味着
    人类将变得越来越边缘化。

    • 回复: @Daniel Chieh
  208. res 说:
    @Afrosapiens

    咄!

    来吧非洲人,你比那更好(无论如何我认为)。

    你的观点到底是什么? 我的观点是,虽然我很清楚热带地区的疾病发病率增加,但医学也与此相关(正如您的引文也观察到的那样),并且我相信,正如我所描述的那样,认为从较高的智商到较低的DALY之间存在因果关系是合理的。 您认为国家平均智商越高,有效医疗越有可能得到普及的说法是不合理的吗?

    • 回复: @Afrosapiens
  209. @Factorize

    伍德利关于古代基因组的论文使用了皮弗方程,在这种情况下它引起的争议较少。

    • 回复: @res
  210. @res

    我也对坦普勒的数据感到惊讶,并发现相关性极高。 几个月前,当坦普勒去世时,我做了一些工作,作为他记忆中的帖子的一部分,但由于需要检查肤色数据而未能取得进展。

    • 回复: @res
  211. szopen 说:
    @utu

    现在,我的统计课已经是很久以前的事了,但 IIRC 你没有抓住重点。

    当然,它确实适用于所有协方差矩阵(即使对于那些变量之间的协方差为零的矩阵) - 但在这种情况下,我们有一个正相关的事实(即特征向量中没有零)。

    如果您放置随机噪声数据,您将得到特征向量,第一个特征向量直接指向一个方向,第二个特征向量直接指向另一个方向。 那确实很无趣。 但事实上,无论设计什么测试,即使是为了证明“g”不存在而创建的测试,它们似乎总是相关的,这一点很有趣。

  212. utu 说:

    现在,我的统计课已经是很久以前的事了

    不要紧。 这些不是统计数据涵盖的问题。 更像是线性代数,只是稍微高级一些。

    你的其余评论都是废话,只是表明你没有任何线索。 你甚至不是茹尔丹先生。

    • 回复: @szopen
    , @szopen
  213. Santoculto 说:

    早期人类由于缺乏文化知识而更容易感染热带疾病,就像今天在许多令人怀念的狩猎采集社区中仍然发生的那样。 因此,气候本身的问题,对基本卫生(特定因果关系)缺乏了解,有助于在早期人类适应中创造环境的主导地位。 如果一群更聪明的人类,甚至一群平均(100)人被放置在热带气候的自然环境中,我不相信他们会下意识地适应疾病,成为热带狩猎采集者。

  214. utu 说:
    @res

    你再次表现出不诚实。 这个问题我们之前已经讨论过很多次了。 但今天我很乐意对付顽固的孩子,所以我会再试一次。 假设一组给定的测试产生 k 个因子。 前两个最大特征值是 g 和 s。 并且g>s。 这里我们做出判断并说 g 占主导地位,因为 s/g<0.2。 让我们从具有最大 s 负载的测试中识别电池测试。 创建更多类似的测试并将它们添加到测试组中,同时删除一些具有最大 g 负载的测试。 你最终会得到新的协方差矩阵,分解后会产生比以前更大的 s/g 。 通过这样的过程,你可以使 s/g 等于 1。显然,你会反对说,人们不能搞乱我将回答的测试:谁说的? 斯皮尔曼或比奈是在西奈山接受测试还是组建天使莫罗尼?

    • 回复: @szopen
    , @res
  215. szopen 说:
    @utu

    当然,也许我不知道,所以请帮助我。 即使你认为我是一个无能的白痴,不值得关注,你的解释也可能会帮助其他比我更聪明的人。

    根据我的理解,你总是可以找到一个因素来解释一堆变量的最大变化部分,但如果这些变量不相关,那么“变化的最大部分”实际上将与这些变量之一的变化本质上相同。 这与斯皮尔曼的发现不同。

    IE。 想象协方差矩阵

    × 0.7 0
    是 0 0.5

    这里解释最大方差的特征向量是 [0.7 0],对吧? 其中最大方差实际上是 X 变量的方差,而 Y 方差没有得到解释。
    然而,这并不是我们在认知能力测试中面临的问题,因此我发现您之前的帖子没有抓住要点——这与“g”因素有何可比性?

    • 回复: @utu
  216. Factorize 说:

    汤普森博士,那篇论文建议智商提高多少? 如果文章中发现的智商提升幅度非常大,那将是一个真正令人震惊的结果。 这表明欧洲人口的高端认知能力是最近的发展,这种更高的能力可以迅速体现在现实世界的成就中。

    如果此类研究也能在非洲和亚洲人群中进行,那将提供非常丰富的信息。

    遗传结果广泛反映了欧洲历史。 类似的结果可能适用于亚洲吗? 在过去的几千年里,欧洲和亚洲是否真的发生了类似的认知进化,这是由于人口密度增加、认知复杂化等同一组力量的结果? 如果是这样,则表明存在可以选择更高智商的操纵。

  217. @RaceRealist88

    “更多的进化挑战”是否意味着“更多的挑战需要增强认知能力才能在生产中生存并将孩子抚养成人”?

    • 回复: @Afrosapiens
    , @Daniel Chieh
  218. Anon • 免责声明 说:
    @jason kennedy

    这比层出不穷的反犹太主义文章要好。 可悲的是,这个网站的所有者本人就是犹太人(如果我没记错的话),他通过发表让人们“讨厌”犹太人的文章,无意中使这里的一些人变得反犹太主义。

    • 回复: @Santoculto
  219. szopen 说:
    @utu

    假设我们有大量的认知测试。 然后我们制作了六个更大的电池,每次都进行随机选择的测试。 然后我们每次进行因子分析,找到第一个因子,并将其称为“g”(g1,g2,g3…)。 当然,由于测试的组成是随机的,如果发现的因素应取决于电池的组成,那么给定测试的 g 负载(g1 负载、g2 负载..)不应高度相关(g1 ! = g2 != g3 等)。 OTOH,如果该因子实际上反映了相同的潜在变量,则 g1 = g2 = g3 (大致),因此 g 载荷应该高度相关(比如 0.85)。 同意吗?

    • 回复: @utu
  220. @res

    你的观点到底是什么? 我的观点是,尽管我很清楚热带地区的疾病发病率有所上升,

    然而你刚才却说:

    从表 1 可以看出,DALY 疾病(其对数)和冬季高温与平均智商的相关性分别为 -0.82 和 -0.72,并且彼此之间的相关性也为 0.71(两者还与距离的对数相关性约为 -0.4)来自中部非洲)。 因此,有证据表明疾病的重要性,但几乎没有证据反对寒冷冬季的重要性。

    所以,呃!

    医学也是相关的(正如你的引文也观察到的那样),我相信,正如我所描述的那样,认为从较高的智商到较低的DALY之间存在因果关系是合理的。 您认为国家平均智商越高,有效医疗越有可能得到普及的说法是不合理的吗?

    除了我的链接之外,任何医学专业人士都不会声称需要智商来减轻疾病负担。 对卫生设施和城市化的投资将使人们远离蚊子、寄生虫和病原体的滋生地,获得药物将降低死亡率,但对降低发病率几乎无济于事,因为从长远来看,没有对大多数热带疾病有效的疫苗。

    我做了一些额外的散点图来观察异常值,我看到的最引人注目的事情是亚洲国家高智商异常值的频率(相对于冬季平均高温、记录的 DALY 和肤色)。

    这就是骗局,傻瓜。 无论您测试什么预测变量,这些亚洲分数都永远没有意义。 他们是假的。

    退一步来说,肤色和智商之间的相关性为-0.92,这让我感到惊讶。

    这就是骗局,傻瓜。 非洲(以及在某种程度上,南亚)的分数是为了产生这种相关性而编造的。 呃!

    • 回复: @res
  221. @Wizard of Oz

    这些挑战使得人类从中非的黑猩猩(我的意思是,共同的祖先)而不是日本猕猴进化而来。

    • 回复: @Wizard of Oz
  222. szopen 说:
    @utu

    你显然比我更有知识、更聪明。 我真的很感激你有时间和耐心来解释对你来说显而易见的事情,而这对我这个低于波兰标准的愚蠢家伙来说是遥不可及的。

    因此,如果您能有时间研究我下面的推理,我将不胜感激。

    我一直在思考具有图形类比的不同因素分析技术。 我将首先陈述显而易见的事实:假设您有两个变量 X 和 Y。您可以将它们绘制在图表上,其中的点代表给定的观察值,具有两个值 x 并在图表中具有 [x,y] 坐标。

    图形类比是,因子分析试图减少一些维度,因此新坐标系中的新轴将捕获图形中先前包含的大部分信息。

    考虑到这个类比,您之前的陈述“您总是可以找到解释最大方差的协方差矩阵的特征向量”,可以简单地重申您总是可以找到一个能够捕获图中最大部分信息的轴。

    对于完全线性相关的变量,显然会有一个新的轴来捕获图中的所有信息。

    对于白噪声类型的图,对于两个(或更多)完全不相关的变量,情况是相同的:我们只需删除方差较小的变量的轴(或多个轴)。 在这种方法中,我们保留一个变量的信息,同时丢失第二个变量的所有信息。

    然而,关于“g”的令人惊奇的事情是,对于“g”,对于任何测试,所有信息都不会丢失(考虑抽样误差时),只要为许多不同的测试计算“g”,就没有范围限制——以及 Free Introduction 在我看来,这是一个不平凡的发现。

  223. Santoculto 说:
    @Anon

    犹太人发现...

    可悲的是犹太人的存在......

  224. Pat Boyle 说:
    @Okechukwu

    北方人(欧洲和中国北方、日本和韩国)很聪明,他们步行到达那里。 这不是我编造的。 这是一个观察。 这些人比居住在赤道附近的人更聪明,这就是问题所在。 例如,林恩认为,这是因为北方寒冷的冬天给他们施加了选择压力,要求他们发展更好的大脑。 并非每个人类学家都同意这一理论,但许多人似乎至少部分接受它。

    这一切都是老生常谈的问题。 这个理论是整个线程的主题。 所以你的言论看起来很奇怪。 我没有写任何有争议的东西。 为什么这么有敌意?

  225. res 说:
    @utu

    固执的孩子

    LOL!

    Ad hominems——对与你争论的人说的最好的方式——“你赢了。”

    PS 有趣的是(也许不是巧合),你们都把恶意归咎于我,并认为这是斯皮尔曼、比奈等人的恶意。也许关于 g 最引人注目的事情是,即使人们试图避免它,它似乎也会出现。 例如,看看测量其他“智能”的尝试最终发现与 g 存在显着相关性的频率。 究竟是谁在这场讨论中恶意参与?

  226. utu 说:
    @szopen

    您从对角矩阵开始。 在主成分分析中,您最终得到对角矩阵,它相当于您开始使用的协方差矩阵(不一定是对角矩阵)。 上面句子中的“等效”意味着两个矩阵跨越相同的线性向量空间。

    分解任何协方差矩阵后,您会得到特征向量,它们是原始变量的线性组合。 这些特征向量相互正交,这意味着这些向量(变量)的协方差矩阵是对角的。 特征向量 v1 与特征向量 v2 不相关,依此类推,因此它不能解释 v2 解释的任何方差,反之亦然。

    然而,这并不是我们在认知能力测试中面临的问题,因此我发现您之前的帖子没有抓住要点——这与“g”因素有何可比性?

    在句子中使用“this”、“however”和“hence”这些词并不自动使该句子成为一个论点。 我不确定你在这里说的是什么,我怀疑你也不确定。

    • 回复: @szopen
  227. @Pat Boyle

    那么,请说出许多人类学家的名字。 仅供参考,林恩不是人类学家或人类生态学/生物学专家。

  228. res 说:
    @Afrosapiens

    其中大部分内容都不值得回应,但让我们看看这个:

    除了我的链接之外,任何医学专业人士都不会声称需要智商来减轻疾病负担。 对卫生设施和城市化的投资将使人们远离蚊子、寄生虫和病原体的滋生地,获得药物将降低死亡率,但对降低发病率几乎无济于事,因为从长远来看,没有对大多数热带疾病有效的疫苗。

    那么智商对医生没有帮助吗? 哈哈! 我认为很多医学专业人士不会同意 .

    尽管大多数热带疾病没有有效的疫苗,但可以采取许多措施来预防和治疗。 这些通常需要资源(例如人均GDP)和能力(两者都与智商相关,以及做出基于事实的成本效益决策的能力)的某种组合。 蚊帐就是一个例子。 以下是一篇研究其对 DALY 的成本效益影响的论文: https://www.ncbi.nlm.nih.gov/pmc/articles/PMC3348006/

    这份世界卫生组织报告着眼于全球伤残调整生命年: http://www.who.int/healthinfo/global_burden_disease/GBD_report_2004update_part4.pdf
    表 12 列出了主要原因。 排名前两位(占总数的2%)的是下呼吸道感染和腹泻病。 我的理解是,这两种情况都非常适合医疗。 疟疾排名第 10,发病率为 12%。

    PS 未经证实的欺诈指控并不是一个令人信服的论据。

    • 回复: @Afrosapiens
  229. @Wizard of Oz

    我认为这是一个公平的论点; 基本上是一个专门培育智力的环境,而不是抵抗疾病或捕食的环境。 我还要补充一点——似乎早期对智力最强烈的驱动力基本上是通过更多的社会接触(我们在灵长类动物中看到了这一点,那些必须欺骗和被欺骗的社会性越强的动物进化出更高的智力) 。

    因此,必须应对日益复杂的涉及欺骗和欺骗侦查的社会环境的人类群体可能具有增强智力的最强大动力。 我们可能会认为这是犹太人智力的一个因素,一般来说,对于“市场主导的少数群体”来说。

    • 回复: @res
  230. @Factorize

    然而,如果人类对提高智商感到拘谨,那么我认为可以做出全面的努力来优化其他表现出心理测量学的动物的智力,例如小鼠、大鼠、乌鸦、狗、猫、浣熊、黑猩猩、大猩猩和别的。

    中国有相当广泛的动物育种和实验设施,部分与实用的 CRISPR 开发有关,包括最近在狗身上取得的成功。 看看是否可以将更聪明、但在其他方面温和、友善的狗带入人群中,这将是一件很有趣的事情。

  231. szopen 说:
    @utu

    我必须说,当我看到它时,我很感激这种侮辱😀

    但请继续你的解释:
    特征向量 v1 与特征向量 v2 不相关,依此类推,因此它不能解释 v2 解释的任何方差,反之亦然。
    但这并不意味着 v1 解释了一个变量的方差,而 v2 解释了第二个变量,对吧? 如果我理解正确的话,变量 X 值的变化的一部分由 v1 解释而无法由 v2 解释,变量 X 值的变化的一部分由 v2 解释而无法由 v1 解释,变量 Y 也是如此,我明白吗?

    但是,在变量不相关的情况下,您仍然可以创建特征向量 v1 和 v2,但变量 X 中的所有方差都将由 v1 解释,而 v2 则不会解释。 变量 Y 中的所有方差都将由 v2 解释,而 v1 则不会解释。 我对吗?

    然而,在“g”的情况下,“g”解释了每个测试的一些差异。 这就是为什么我写了“这与我们有不相关变量的情况不同”的原因。

  232. Santoculto 说:

    需要指出的一件重要的事情是宏观、微观、纳米人类群体中的机械主义——心灵主义的水平。 我认为机械主义水平(对科学、工程、数学的兴趣)往往与较高的智商/认知技能呈正相关,即使不是非常正相关,当然,即与数学高于动词的倾斜相关。 现代社会在很大程度上是由这种类型的思想/人维持的,似乎撒哈拉以南的黑人往往不那么“机械主义”,而更“心智主义”/同理心[不要与“同情心”混淆,这是高度强调的人可能产生的影响之一,但不是所有人……是的,许多黑人,即妇女,非常富有同情心]。

    因此,在共情系统化谱中,我认为,东北亚人的平均得分更高,“白人”处于中间[北部白人与东北亚人一起,南部白人与其他群体一起],而撒哈拉以南非洲人则处于底部。 我的印象是,犹太人在系统化和同理心方面往往都较高,但由于许多“异常”的认知倾斜亚型,阿什克纳粹的认知特征已经不平衡。

    请记住,人类智力已经进化/强烈偏向于系统化技能。 这也许可以解释为什么人类有能力建造许多奇妙的东西,但在社会问题上仍然处理得很差。

    共情-系统化谱还与自闭症样心理特征[更系统化]、艺术倾向[两者]甚至与语言技能[共情]相关。

    类自闭症特征的水平与以下因素相关:秩序、较低的暴力和腐败水平。

    • 回复: @Santoculto
  233. res 说:
    @James Thompson

    考虑到地图和评估者的结合,除非他们保留大量记录,否则可能很难检查数据。 有川有原始地图和个人评估者数据吗?

    该书 4 年版的第 1959 卷可在 libgen 上找到,但我搜索了“colore della pelle”(意大利语,肤色的意思),尽管搜索了大约 10 次,但没有看到像地图或表格那样的东西。 它有 800 多页长,而且是意大利语,所以我当时就放弃了。

    PS 你看过这篇博文吗? http://www.gnxp.com/blog/2007/04/validity-of-national-skin-color-iq.php
    他们对另一个肤色来源重复了分析,结果与智商的相关性为 0.87,与 Templer 肤色测量的相关性为 -0.91。

    PPS 注意主要是给我自己的。 坦普勒和有川研究了种族、肤色和智商 https://www.ncbi.nlm.nih.gov/pubmed/17037466
    在黑人种族中,智商和肤色之间存在不显着(且符号相反)的相关性。 但在其他两个种族以及包括多个种族的群体中显示出显着的相关性。

  234. @res

    那么智商对医生没有帮助吗? 哈哈! 我认为很多医学专业人士不会同意这一点。

    当然,您是否经常被询问您的智商分数以进行任何诊断?

    您如何将疟疾发病率的下降与智商联系起来?

    我的理解是,这两种情况都非常适合医疗。 疟疾排名第 12,发病率为 2.2%

    上述疾病负担研究并未表明所有伤残调整生命年都重要,只有传染性、营养性、新生儿和寄生虫伤残调整生命年才重要。 因此,与心血管、精神或事故相关的 DALY 不计算在内。

    不管怎样,预防和治疗很重要,我并不是说这些疾病是不可预防的。 然而,它们在热带环境中更难控制,并且控制它们不需要特定的智商。

    PS 未经证实的欺诈指控并不是一个令人信服的论据。

    这不是无缘无故的,愚蠢的。
    http://www.iapsych.com/iqmr/fe/LinkedDocuments/wicherts2010c.pdf

    https://www.researchgate.net/profile/Han_Maas/publication/220018518_Another_failure_to_replicate_Lynn’s_estimate_of_the_average_IQ_of_sub-Saharan_Africans/links/0046352814aafa3c2b000000.pdf

    • 回复: @res
  235. res 说:
    @Daniel Chieh

    那很有趣。 谢谢。

    但我不确定这个论点是否足够。 中国与技术相当的对手发生过多少次战争? 我认为欧洲发展枪械技术的部分动力是技术相当的对手之间的频繁交战,从而导致各种层面的军备竞赛。 中国军备竞赛有一个很好的例子吗?

    您如何看待紧随中国墙理论之后的蔡斯假说? 我发现前者(蔡斯)不如后者引人注目,但我认为这可能提供了部分原因。

    莫里洛关于去中心化/中心化相关的想法也很有趣。

    PS 大通部分提到了这一点:“就四川而言,双方都使用了同等威力的枪支”,但没有给出参考或更详细的内容。 这是对我的第一点的反驳,还是证明规则的例外?

    • 回复: @Daniel Chieh
  236. utu 说:
    @szopen

    如果电池组 B1 包含有利于“语言智力”的测试,而电池组 B2 包含有利于“几何智力”的测试,那么与两个电池组以相同比例混合有利于两种类型智力的测试相比,g1 和 g2 的相关性会更小。 如果我们用 s1 和 s2 表示两个电池的第二大因素,则 g1 可能与 s2 良好相关,g2 与 s1 相关良好。 那么两个电池的主要两个因素就会颠倒过来:g1s2 和 g2s1。 g1 和 g2 都不是 Spearman 或 Binet 的“g”,因为它们是不同的测试组。 要点是 g 确实是任意的。 这取决于您如何组成一系列测试。

    对于某些人来说,我怀疑你就是其中之一,斯皮尔曼使用的论点 g 是某种东西(特殊的、独特的),因为它是他的因素分析中的主要因素,这是非常有吸引力的。 但这是一个非常不诚实的论点。 这里涉及到烟雾和镜子。 曼波巨无霸大字数学。 我只是想揭开它的神秘面纱。 由于各种原因,计算一些 g 并在某些研究中使用它来代替测试的原始数据是完全可以的。 但应该强调的是(事实并非如此),某些研究中使用的任何特定 g 只是特定于本研究中使用的一系列测试的 ag。 它不是全称 g,因为全称 g 不存在。 但用 g 的存在这一事实来作为智力理论的最高杀手锏是绝对不行的。 这是一个欺诈性的论点。 通常在语音或书面文本中,g 被用作纯粹的修辞手段,例如“因为有 g……”。 或“因为它已加载……” 。 这些都是错误的论点。

    • 回复: @szopen
    , @PandaAtWar
    , @CanSpeccy
  237. res 说:
    @Daniel Chieh

    因此,必须应对日益复杂的涉及欺骗和欺骗侦查的社会环境的人类群体可能具有增强智力的最强大动力。

    这。 我认为这说明了可能促进大社会形成(例如农业)或预防大社会形成(例如传染病)的事物的重要性。 我可以看到技术 -> 大型人类社会 -> 智商 -> 技术之间可能存在强大的反馈循环……
    然后是冲突中的大社会的推动力(参见火药讨论)。

    我还认为智力对社会功能的重要性提供了一个很好的理由,像“好吧,如果寒冷很重要,那么爱斯基摩人应该是最聪明的”之类的观察并不像某些人想象的那么有意义。 就像我之前说的那样。 有些挑战有助于智商的进化,但那些使生活或大规模社会变得不可能的挑战则无济于事。

    • 回复: @Wizard of Oz
  238. @Pat Boyle

    这一切都是老生常谈的问题。 这个理论是整个线程的主题。 所以你的言论看起来很奇怪。 我没有写任何有争议的东西。 为什么这么有敌意?

    这也是南非电网濒临崩溃的原因。 智商较低只是众多问题之一。 存在许多文化问题,包括几乎完全无法承认任何错误。

    有点有趣的是,我与一位来自津巴布韦的精英一起工作,也许并不奇怪,那里的精英大多对他们的孩子使用古典的欧洲方法,包括寄宿学校。 因此,在他们的一小部分人口中,他们知道自己需要努力和改进什么——但由于腐败猖獗,整体社会前景显得黯淡。 人才流失效应进一步剥夺了他们的本土人才,而且似乎没有像亚洲那样的“海龟”效应,受过西方教育的亚洲人返回自己的国家或受到有见地的欢迎。

    • 回复: @res
    , @Okechukwu
  239. @res

    火药使用后,中国似乎不再有军备竞赛——明朝从全面控制走向全面崩溃,晚清也曾努力追赶欧洲但惨败,或多或少,我们有现在。

    事实证明,缺乏尊重和低估世界其他地区是非常致命的,我希望这是我们能够学到的文化教训。

    • 回复: @res
  240. res 说:
    @Afrosapiens

    那么智商对医生没有帮助吗? 哈哈! 我认为很多医学专业人士不会同意这一点。

    当然,您是否经常被询问您的智商分数以进行任何诊断?

    完全不明白我的观点。 拥有相当高的智商(比如 115 以上,当然这是一个软限制)对于医生来说是很有用的。

    您如何将疟疾发病率的下降与智商联系起来?

    考虑相对变化和绝对变化率很重要。 我认为(传染病)伤残调整生命年可以更准确地衡量应对疾病的总体成功程度。 疟疾的基本发病率也极大地影响了减少疟疾的动力。

    上述疾病负担研究并未表明所有伤残调整生命年都重要,只有传染性、营养性、新生儿和寄生虫伤残调整生命年才重要。 因此,与心血管、精神或事故相关的 DALY 不计算在内。

    正确的。 这就是我使用世界卫生组织的传染病伤残调整生命年信息进行自己的分析的原因。 所以你说的是我对前两名贡献的 10% 估计应该更高。 您是否对下呼吸道感染和腹泻疾病的治疗是否合适存在争议?

    不管怎样,预防和治疗很重要,我并不是说这些疾病是不可预防的。 然而,它们在热带环境中更难控制,并且控制它们不需要特定的智商。

    感谢您澄清您的立场并且至少在某种程度上是合理的。

    我喜欢用“需要特定的智商”作为稻草人。 他们在某个智商很差的论证课上教过这个吗? 它的使用如此普遍,我不敢相信这是巧合。

    当然,不需要特定的智商(我没有在任何地方说过它是,因此“稻草人”)。 但智商对预防和治疗的许多方面都有帮助。 例如,识别问题、制定和选择多种解决方案、提供资源来支付解决方案的费用。

    我同意在热带环境中疾病更难控制。 在亚热带气候中也有类似的影响,但影响较小。 我认为,美国南部长期以来的疾病经历提供了一个很好的例子,说明有能力做出改变。

    感谢您提供 Wicherts 链接。 不过,包含林恩的一些回应是公平的:
    http://www.sciencedirect.com/science/article/pii/S0160289609001275
    https://lesacreduprintemps19.files.wordpress.com/2012/12/the-average-iq-of-sub-saharan-africans-assessed-by-the-progressive-matrices-a-reply1.pdf

    争论似乎集中在非洲智商研究样本的代表性上。 我想,如果你认为大学生代表了一般人群,那么威彻茨的观点是有说服力的。 美国的情况肯定不是这样,尽管有些人似乎相信非洲的情况也是如此(参见最近的 Chanda Chisala 帖子)。

    无论如何,我认为指控林恩欺诈是有过分的。

    笨。

    Ad hominems——对与你争论的人说的最好的方式——“你赢了。”

    • 回复: @Afrosapiens
  241. res 说:
    @Daniel Chieh

    你在这里提出了很多好的观点。 不过有一个问题。 你的经验是否表明,精英们的改进努力是否有助于解决“几乎完全无法承认任何错误”的问题? 我发现这种态度对于任何持续改进的努力都是致命的(例如日本制造质量改进技术)。

    我在自己的个人生活中看到了一个明显的肯定例子,但我在这些评论线程和其他地方看到的总体趋势并不令人鼓舞。

    对于大多数阅读此评论主题的人来说,“无法承认错误”(又名承担责任)可能是显而易见的,“无法承认错误”(又名承担责任)是用来观察我们当前(以及许多与非洲相关的)对话的强大镜头。 无论是在字面论证技术层面还是在我们所争论的元层面。

    • 回复: @Daniel Chieh
  242. res 说:
    @Daniel Chieh

    火药使用后,中国似乎不再有军备竞赛——明朝从全面控制走向全面崩溃,晚清也曾努力追赶欧洲但惨败,或多或少,我们有现在。

    谢谢!

    事实证明,缺乏尊重和低估世界其他地区是非常致命的,我希望这是我们能够学到的文化教训。

    同意。 并且适用于许多不同的“我们”。

    我刚刚读了史蒂芬·安布罗斯(Stephen Ambrose)关于第一条横贯大陆铁路的历史,值得注意的是,尽管当时存在种族主义,但人们对中国的某些特质(和成就)始终表示尊重,这些特质至今仍受到赞赏。 不确定其中有多少是电脑侦察和/或特定于当时的精英,但恕我直言,仍然值得注意。

    美国低估了中国的能力、意志和毅力,后果自负。 (而且,我想,反之亦然;)

    • 回复: @Daniel Chieh
  243. szopen 说:
    @utu

    不过我理解你的观点:

    ” 要点是 g 确实是任意的。 这取决于您如何组成一系列测试。 “

    (1) 所以你认为 Thorndike 的 [1985] 分析(插入到六个不同电池中的 0.85 个“探针”测试的 g 负载之间的相关性为 17,每个电池有 2004 个非重叠测试)和 JOhnson [XNUMX](三个电池的多个非重叠测试(多余的测试被删除,应用于同一组人,从电池中提取的“g”几乎相同)是无效的,因为它们都测量不同的任务?

    (2) 但这里的全部意义不就是要找出是否存在潜在的“通用智能”吗? 那么,如何通过仅研究一种能力而不是其他能力(最好是尽可能多的能力)来回答这个问题(“是否存在可以解释多个不同、不同领域的表现的潜在通用智力”)的答案?

    我还必须指出,我对 Jensen 更熟悉,而且我主要从 Jensen 和其他人概述他的想法的论文中了解 Spearman(因为 Spearman 语言对于非母语人士来说有点过时,并且阅读了他 1904 年的开创性论文) g 上的纸是一种折磨)。 然而,我并不认为“g是主要因素”是最好的论据,而是“每当我们进行不同的测试时,出现的主要因素总是高度相关的”,这是不一样的。

    • 回复: @utu
  244. @res

    完全不明白我的观点。 拥有相当高的智商(比如 115 以上,当然这是一个软限制)对于医生来说是很有用的。

    哦天哪……我希望你没有提出这一点。 好吧,那么根据经验准确地告诉我,智商提供的哪些内容对医生有用。 据我所知,没有任何智商分数被证明是任何事情的必要条件,并且每个智商分数都足以产生任何结果。 因此,如果您知道任何此类事情,请告诉我。

    考虑相对变化和绝对变化率很重要。 我认为(传染病)伤残调整生命年可以更准确地衡量应对疾病的总体成功程度。 疟疾的基本发病率也极大地影响了减少疟疾的动力。

    疟疾的基本发病率高度依赖于气候,而气候使得疟疾流行地区的控制变得更加困难。

    疟疾减少可能是应对疾病负担能力的最佳指标。 因为与其他疾病相反,疟疾病媒对杀虫剂和药物的抗药性进化得非常快,因此需要卫生当局非常积极的反应,而其他疾病的治疗更直接,预防更依赖于社会经济因素,例如药物成本、安全性等。食物和水、适当的卫生设施……

    此外,在人口激增的国家,疟疾发病率的下降更为惊人。

    所以你说的是我对前两名贡献的 10% 估计应该更高。 您是否对下呼吸道感染和腹泻疾病的治疗是否合适存在争议?

    是的,这就是我所说的,但疟疾和其他寄生虫病也应该如此。

    就这两个主要原因而言,下呼吸道传染病主要是肺炎,腹泻病则是由于食物和水不安全造成的。 对抗这些疾病与医生的智商无关,肺炎疫苗价格昂贵,获得安全食品和水需要在卫生设施和有益健康的住房以及食品控制和冰箱方面进行极其昂贵的公共投资。

    这比智商还要复杂,笨蛋。

    我喜欢用“需要特定的智商”作为稻草人。 他们在某个智商很差的论证课上教过这个吗? 它的使用如此普遍,我不敢相信这是巧合。

    当然,不需要特定的智商(我没有在任何地方说过它是,因此“稻草人”)。 但智商对预防和治疗的许多方面都有帮助。 例如,识别问题、制定和选择多种解决方案、提供资源来支付解决方案的费用。

    畏缩…

    您真的认为发展中国家的卫生专业人员不了解这些疾病吗? 或者人们有机会生活在安全的环境中,但因为太愚蠢而没有这样做?

    你能“用你的智商”来避免此类外星人指控吗? 如果你不明智,体面的评论者将不再在你身上浪费时间,你将只剩下 santoculto。 那是你要的吗?

    我同意在热带环境中疾病更难控制。 在亚热带气候中也有类似的影响,但影响较小。 我认为,美国南部长期以来的疾病经历提供了一个很好的例子,说明有能力做出改变。

    这是有可能的,特别是在亚热带地区,那里的疾病爆发是季节性的,并且获得卫生设施和药物可以减少人口的脆弱性。 但这样的改变几乎不依赖于智商,恰恰相反,愚蠢。

    感谢您提供 Wicherts 链接。 不过,包含林恩的一些回应是公平的:

    不,这不公平,因为这就是 Afrosapiens 揭露欺诈行为的方式。 无知是福。

    通过渐进矩阵评估撒哈拉以南非洲人的平均智商:答复
    致 Wicherts、Dolan、Carlson 和 van der Maas

    5. 不可接受的研究
    WDCM 包括许多研究,但这些研究无法被各种不同的标准所接受。
    的原因。 他们的大学生样本显然不具有代表性。
    Crawford-Nutt (1976) 样本包括高中
    数学课程的学生(智商 84)的录取“取决于
    学生在低年级表现的优秀程度”
    (第 202 页)并被描述为“人口的选定部分”(第 204 页)。
    学生们还接受了如何进行测试和“教授
    解决矩阵问题所需的策略会产生巨大的短期效果
    得分的增长”(Raven, Court, & Raven, 1996, p. 33)。 这显然是一个
    不具有代表性的样本。
    另一项不可接受的研究是 Okunrotifa (1976) 的样本报告
    结果二:(1)样本为50名尼日利亚农村小学生
    5.5 岁,WDCM 给出的智商为 88。得分为 25.9=
    英国 84 年标准化样本的第 1979 个百分位数=115 IQ; (2)
    100名尼日利亚城市7岁小学生的样本
    WDCM 的智商为 85。分数为 34.75= 第 96 个百分位数
    英国1979年标准化样本=126 IQ。 两人智商到目前为止
    超出了可信度范围,研究不得不被拒绝。
    WDCM 假设报告的平均值是百分位数而不是
    比原始分数高,但这只是猜测。 可能会认为
    WDCM 会询问 Okunrotifa 这种猜测是否属实
    正确,但似乎他们没有这样做。 WDCM 的假设是这些
    尼日利亚儿童的智商为 85,而 88 似乎不太可能。 这些是
    美国黑人的智商。 黑人几乎不可能
    拥有居住在美国的所有优势
    经济发达国家,收入高,医疗条件好,
    良好的营养和教育,将拥有与黑人相同的智商
    贫穷的尼日利亚。 如果是这样的话,我们就不得不推断这些
    环境优势对智商甚至智商没有任何影响
    大多数强硬的世袭主义者不会走那么远。 由于所有这些原因,
    WDCM 本研究的 IQ 无法被接受。 其他一些
    Lynn (2010b) 讨论了 WDCM 给出的不可接受的研究。

    是的,他们写道……

    1-大学生/富裕人士

    专业的人口统计学家不会排除此类样本,他会将它们纳入其中并调整样本的权重以使其在一般人群中具有代表性。

    2- 高分样本的可信度

    卧槽! 我从来没有见过这么不科学的说法,谁管非裔美国人的分数是多少,用同样的逻辑就应该拒绝所有得分超过90分的中国样本,因为他们的生活条件比非裔美国人差得多,这几乎是不可能的。

    Wicherts、Dolan 和 van der Maas (2009) 认为,撒哈拉以南非洲人的平均智商约为 80。对 WDM 提交的研究进行的严格评估表明,其中许多研究都是基于不具代表性的精英样本。 我们表明,对 29 项具有可接受代表性的研究 除渐进矩阵之外的测试样本显示,撒哈拉以南非洲地区的智商为 69; 对标准渐进矩阵上最令人满意的代表性样本的研究得出 IQ 为 66; 研究 23 个可接受的代表性样本 彩色渐进矩阵的智商为 71。数学、科学和阅读的国际研究得出撒哈拉以南非洲地区的智商为 66。对四个数据集进行平均,智商为 68,这是该地区的最佳阅读水平。撒哈拉以南非洲地区的智商。

    什么是“可接受的代表性”? 代表是谁、何地、何时? Lynn 和 cie. 在人口研究方面的资历如何?

    无论如何,我认为指控林恩欺诈是有过分的。

    也许你更喜欢无能?

    Ad hominems——对与你争论的人说的最好的方式——“你赢了。”

    再努力一点吧,笨蛋。

    • 回复: @szopen
  245. Okechukwu 说:
    @Pat Boyle

    北方人(欧洲和中国北方、日本和韩国)很聪明,他们步行到达那里。

    恭喜。 您刚刚发现步行是天才级别的活动。 让赤道血统的人向你的寒冷天气衍生的大脑注入一些急需的教育。

    早期人类没有计划、地图、国家、过境点或护照管制。 几千年来,人们只是四处游荡,平均每天走几步。 他们可能跟踪猎物。 他们可能只是迷路了。 走出非洲漫游并不比在非洲境内漫游更费力或要求更高。 非洲是一个很大的地方。 地图未按比例绘制。 您可以融入非洲、欧洲、英国、美国、印度、中国和日本。 事实上,从欧洲到北美的航行距离明显短于西非和东非之间的距离。 北非和南非之间的距离几乎是美洲和欧洲之间距离的两倍。 因此,走出非洲的早期人类并没有做出比非洲境内的早期人类更值得注意的事情。

    这些人比居住在赤道附近的人更聪明,这就是问题所在。

    你的意思是你希望、祝愿并祈祷他们更聪明。事实上他们并不更聪明。

    例如,林恩认为,这是因为北方寒冷的冬天给他们施加了选择压力,要求他们发展更好的大脑。

    这个理论在很多层面上都是无稽之谈,林恩是个爱开玩笑的人。让我提醒您,像直立人和尼安德特人这样的荣耀大猩猩在“寒冷的冬天”非常成功,顺便说一句,这只是四个季节之一。全年并没有出现寒冷的天气。

    并非每个人类学家都同意这一理论,但许多人似乎至少部分接受它。

    因此,发挥你在寒冷天气下的大脑来识别这些神秘的人类学家吧。

    这一切都是老生常谈的问题。 这个理论是整个线程的主题。 所以你的言论看起来很奇怪。 我没有写任何有争议的东西。 为什么这么有敌意?

    嗯……也许是因为你说了一个赤裸裸的谎言,暗示大多数人类学家都同意离开非洲的人变得更聪明的荒谬观念。仅供参考,他们不相信这一点。你刚刚弥补了。

  246. @res

    你的经验是否表明,精英们的改进努力是否有助于解决“几乎完全无法承认任何错误”的问题?

    非常稀疏且不协调。精英阶层大量移民,不同部落和家庭之间存在很多争夺。尽管黑人在美国作为一个整体投票,但非洲的团结似乎并不重要。它的盗贼统治方式令人难以理解;例如,巴基斯坦人称领导人阿西夫·阿里·扎尔达里为“巴基斯坦先生”。 10%”,因为据说他窃取了每笔交易的 10%。据我推断,津巴布韦政府会从每笔交易中窃取 50% 以上。我认识的那个人特别提到 俄罗斯黑帮 作为更理性的人。想象一下。

    因此,即使是当今的文化也鼓励发展与我们想象的成功不同的技能。例如,我的朋友在非洲有一个兄弟,他正在使用计算机化技术半成功地经营汽车零部件,但他必须故意削弱他的技术资产并隐藏他的业务,以不干净地解释事情,否则政府会注意到并基本上向他征税/抢劫。必须付出大量努力来逃避法律,当政府的行为本质上是犯罪时,很难将其称为犯罪。

    问题如此之多,而且这些问题在文化中如此普遍和普遍,很难想象他们将如何摆脱它,除了像埃塞俄比亚这样的少数地方。

    • 回复: @Okechukwu
  247. Okechukwu 说:
    @Daniel Chieh

    你可以尝试让自己的房子井井有条。

    腐败猖獗?中国目前是世界诈骗之都。只要问问曾经尝试在那里做生意的人就知道了。或者在线阅读他们的证词。

    智商低?中国人为了让它们的小啄木变得坚硬而无望地消灭了美丽的濒危动物。如果这不是智商低下的表现,我不知道什么才是。

    像你们这样的人,让中国赢得了“亚洲病夫”的称号。

    • 回复: @Afrosapiens
    , @Daniel Chieh
  248. @Okechukwu

    确切地说,他们总是像中国或韩国那样说话,就像这些是有序、诚实和功能正常的社会!哈哈!我是一名商业律师,我有一些关于在中国做生意的故事,或者只是从中国进口东西......或者像在中国开车这样简单的事情,你指出的迷信,腐败,黑手党......

    那些亚洲人对非洲人新发现的优越感是可笑的,如果智商应该意味着什么的话,相信林恩的东亚智商极高也是可笑的。

    • 同意: Okechukwu
    • 回复: @Daniel Chieh
  249. @Okechukwu

    典型的方法论是:无视现实,继续攻击,注重情感而非事实。然而,只有彻底的傻瓜才会认为中国的腐败程度达到了SSA的水平。

    你不值得花时间反驳,但请注意下面的来源。

    https://www.unz.com/akarlin/cpi-2016/?highlight=corruption

    https://www.transparency.org/cpi2015

    当然,这是非洲寻求中国援助的事实,而不是相反。请。如果你真的相信你所宣传的内容,我只能祝你一切顺利。祝电网好运。

    • 同意: res
  250. 对于那些对人类智力进化的实际理论感兴趣的人,本文将上述一些评论者提到的社会大脑假设(社交网络复杂性驱动智力进化)与一些领先的竞争理论进行比较。当然,冷冬理论不是其中之一,但看到有人用实际的专业场景来对抗 CWT 会很有趣。

    https://44984771-a-62cb3a1a-s-sites.googlegroups.com/site/dmobbs/relatedness_class/Dunbar_1998.pdf?attachauth=ANoY7cryF-9NybE4HRk8YGTdFeqKiX6-qzpLjbTZStFTfJlmpeO98oFnoOirgRVAclRBEdAUohPDgX_1ZVjxJBv53hrwnR81Glzl2U7Wz3Rjwqj1r5U82aw2JtwqH7W-fc-JuWkVD5T1B-m6SfUJqoOJR-Fuv3WSbFYbSWvyAdc5LlllHueUwIp3bgOmR28GctwCooI7YsXtJ6gBBECbtbCpW8ADqpMJZFOhzsItBQFXDz1ByWjutQ8%3D&attredirects=0

    • 回复: @res
  251. @res

    美国低估了中国的能力、意志和毅力,后果自负。 (而且,我想,反之亦然;)

    随着大数据带来的“软威权主义”变得越来越普遍,正如我们在德国、英国等国家加强言论监管所看到的那样,不幸的是,中国似乎正在成功地将其政府输出到西方,而不是变得自由化由西方。

    这不是我真正支持的趋势,但值得注意。

  252. Okechukwu 说:
    @Daniel Chieh

    非洲肯定使用计算机化技术半成功地经营汽车零部件,但他必须故意削弱他的技术资产并隐藏他的业务,以便不干净地解释事情,否则政府会注意到并基本上对他征税/抢劫。必须付出大量努力来逃避法律,当政府的行为本质上是犯罪时,很难将其称为犯罪。

    你一定是那些亚洲至上主义者之一,他们颂扬他们在西方安顿下来时在脑海中炮制的神话中国的美德。

    在中国,从最高层到最低层,腐败现象十分猖獗和普遍。此外,中国的商业行为阴险、欺骗、奸诈,对个人、企业和国家伤害极大。中国人似乎从未发展出一种光荣的商业方式,使交易对手方都能获益。这始终是一场零和游戏。他们规避、欺骗、偷窃。他们利用虚拟奴隶劳动来制造产品,然后将其作为出口品倾销,从而摧毁了整个行业。

  253. @Afrosapiens

    为什么是的,犯罪和暴力的事实完全支持你的幻想。

    https://en.wikipedia.org/wiki/List_of_countries_by_intentional_homicide_rate

    或者也许只有你的轶事才会这样做,而且还有一种相当可预测的拒绝承认错误的态度。无论如何,也祝你梦想成真。

    • 回复: @Afrosapiens
    , @Okechukwu
  254. szopen 说:
    @Afrosapiens

    智商所提供的对医生有用的东西。据我所知,没有任何 IQ 分数被证明是任何事情的必要条件,并且每个 IQ 分数都足以产生任何结果

    虽然一般来说,理论上讲,任何分数的人都可能在任何类型的工作中表现出色(一项研究测试了不同职业的样本,具有医学学位的医学职业的智商中位数约为 120,并且90%的人智商在105以上,3/4在110左右。医疗职业的平均水平(没有医学博士)更低,90%的门槛更低),真的令人难以置信有人会否认这一点。智商与工作绩效之间的关系。

    一个普遍的发现是,智商(或者更严格地说,“g”)确实可以预测各种工作的工作表现(尽管有理查森论文)。虽然我对医生不太确定,但我非常怀疑这会是一个例外。

    并不是说没有其他因素,但是,由于IQ分数表明了抽象思维、逻辑推理、阅读理解的能力;因为高分与学习能力和良好的记忆力密切相关;我认为高智商分数也表明了对成功医生有用的能力。

    无论智商如何衡量,似乎对成为一名医生都有帮助。

  255. res 说:
    @Afrosapiens

    谢谢。对于那些喜欢不那么复杂的链接的人来说,这可能会更好: http://psych.colorado.edu/~tito/sp03/7536/dunbar_1998.pdf

    我发现有趣的是,引用单一原因谬误(评论 40)的人发现这样的论点很有吸引力:

    为了检验相互竞争的假设,我们需要迫使假设发生冲突,使得它们的预测相互矛盾。这使得数据能够明确地区分它们。在本例中,我们可以通过询问哪种假设最能预测灵长类动物大脑大小的差异来做到这一点。为此,我们需要确定每个假设做出的具体定量预测,并确定大脑大小的适当衡量标准。

    我发现这样的推理更有吸引力: https://www.ncbi.nlm.nih.gov/pmc/articles/PMC3973910/

    对人类大脑进化的解释主要集中在物理环境(气候、饮食、食物供应)和社会环境(群体规模、联盟形成、父母照顾)的选择压力上。尽管已经尝试区分生态理论和社会理论(例如,参见 Leigh,2004;Dunbar 和 Shultz,2007a,b;Lefebvre,2012;Roth 和 Dicke,2012;Willemet,2013),但几乎没有做出任何努力来发展生态理论和社会理论。整合了已确定的大脑大小的许多社会、生态和生活史相关性的解释框架。显然,它们都在解释人类和猿类之间大脑大小的显着差异方面发挥了作用,但目前还不清楚以何种方式以及在多大程度上存在差异。我们需要更好地研究认知和行为,以及比较大脑研究,来回答这些问题。

    注意气候这个词。

    我发现他们对人类大脑进化极限的讨论内容非常丰富。

    • 回复: @Afrosapiens
  256. @Daniel Chieh

    什么!?中国,这个拥有50起死罪的警察国家,凶杀率却达到欧洲水平?不废话!

    或者也许只有你的轶事才会这样做,而且还有一种相当可预测的拒绝承认错误的态度。

    我有什么过错必须承认?您能详细解释一下为什么您的超级人部落还远未在所有效率排名中名列前茅吗?尽量不要找借口并承认自己的错误。

    • 回复: @Daniel Chieh
  257. @res

    我发现有趣的是,引用单一原因谬误(评论 40)的人发现这样的论点很有吸引力:

    我不记得在这篇论文中分享过我的观点。我也有同样的反对意见,一个假设不应该排除所有其他竞争理论,但我很欣赏为检验一种理论的优点和缺点而付出的智力努力。

    注意气候这个词。

    谢谢,我懂一些英语,尤其是那些源自法语/意大利语/拉丁语的单词。

    那么,是的,气候还有什么?您知道气候不仅仅是温度,对吗?例如,水对于人类的生存更为重要,因此可以合理地假设,解决水资源短缺问题会带来更直接且不可避免的进化挑战。直立人已经掌握了寒冷地区的生存能力,而智人却来自东非。

    我发现他们对人类大脑进化极限的讨论内容非常丰富。

    我不在乎

  258. Okechukwu 说:
    @Daniel Chieh

    为什么是的,犯罪和暴力的事实完全支持你的幻想。

    来自中国的每一个数据点都是不可信的、虚假的。从他们的智商分数到犯罪率和凶杀率再到国内生产总值数据。

    中国不是一个稳定的国家:

    仅180,000年一年,中国就发生了2010万起抗议、骚乱和大规模示威活动——平均每天约500起——此后数字可能还在增加。村民们的抱怨很常见:地方官员利用卖地谋取私利,并暴力镇压异见。

    https://www.theatlantic.com/international/archive/2012/01/how-china-stays-stable-despite-500-protests-every-day/250940/

  259. @szopen

    有人否认智商和工作绩效之间的关系,这真是令人难以置信。

    尽管许多人对相关性并不等于因果关系感到不安,但关系和因果关系是两个不同的东西。

    一个普遍的发现是,智商(或者更严格地说,“g”)确实可以预测各种工作的工作表现(尽管有理查森论文)。虽然我对医生不太确定,但我非常怀疑这会是一个例外。

    仍然是相关性,说到相关性,我想你想知道智商如何与 45 种人格特质相关,你会对一些相关性感到惊讶,尤其是负相关:

    https://blogs.scientificamerican.com/beautiful-minds/how-does-iq-relate-to-personality/

    性格确实对工作绩效很重要,我认为我们可以同意这一点。

    并不是说没有其他因素,但是,由于IQ分数表明了抽象思维、逻辑推理、阅读理解的能力;因为高分与学习能力和良好的记忆力密切相关;我认为高智商分数也表明了对成功医生有用的能力。

    它并不像你想象的那么确定,许多学习障碍发生在任何智商水平上。

    无论智商如何衡量,似乎对成为一名医生都有帮助。

    或者也许医生的生活有助于在智商测试中获得更高的分数。

    我不想重复自己的话,但将任何因果作用归因于智商(一种构造)都是具体化。

    • 回复: @RaceRealist88
    , @szopen
    , @szopen
  260. szopen 说:
    @Santoculto

    圣,

    问题是,虽然我们应该总是谈论“一般智力”(假定的、假设的潜在变量)和“g”,并记住智商只是一个不完美的测试,我们可以尝试从中提取“g”——大多数(如果不是全部)我读过的作者倾向于将这两者混为一谈。我的意思是,即使是那些首先写下区分两者的重要性的人,在另一篇论文中也很乐意使用智商分数,而他们应该使用“g”分数。我羞愧地承认我也犯了同样的错误,而且我经常谈论“智商预测某事”,而实际上我应该说“g 预测某事”。

    • 回复: @Santoculto
  261. res 说:
    @James Thompson

    有趣的观察。该论文的出版版本也可以免费获得(我发现格式更容易阅读): https://www.cambridge.org/core/journals/twin-research-and-human-genetics/article/holocene-selection-for-variants-associated-with-general-cognitive-ability-comparing-ancient-and-modern-genomes/BF2A35F0D4F565757875287E59A1F534

    图 1 很有趣。我想知道是否对这些个体的一般情况(例如位置、社会阶层、历史上著名的人),特别是公元前-1000 年左右的四个高分异常值有了解。

    我想知道他们是否计划将这一分析延伸到更早的时间。本文有一些较旧的例子: https://www.ncbi.nlm.nih.gov/pmc/articles/PMC4014435/
    还有这个: https://www.ncbi.nlm.nih.gov/pmc/articles/PMC4170574/

    • 回复: @James Thompson
  262. @Afrosapiens

    工作绩效

    智商是工作表现的糟糕预测指标。

    https://www.ncbi.nlm.nih.gov/pmc/articles/PMC4557354/

    即便如此,最初关于智商和工作绩效的研究,工作技能测试与工作绩效的相关性比实际智商分数更好。

  263. @szopen

    一个普遍的发现是,智商(或者更严格地说,“g”)确实可以预测各种工作的工作表现(尽管有理查森论文)。

    所以你声称智商(“g”;无论是什么)可以预测工作表现,然后说“尽管有理查森论文”?那是胡说八道,你知道的。

    顺便说一句,在最初的研究中,工作技能测试与工作绩效的相关性比智商测试更好。

    https://www.ncbi.nlm.nih.gov/pmc/articles/PMC4557354/

    为什么您选择不收录 Richardson 和 Norgate 的这篇论文?智商和工作表现的讨论不重要吗?或者是因为它给出了你不喜欢的结果……我想我知道答案。

    • 回复: @szopen
  264. @Afrosapiens

    嗯,这可能与中国人几乎不是超级人(或者就此而言,实际上只是一个种族)这一事实有很大关系。作为一个民族,中国人表现出了停滞、软弱、怯懦、狭隘和对外部世界缺乏认识。但有一件事。 1860 年被英法联军摧毁的圆明园废墟上有一块牌匾,上面写着:

    “我们的弱点是罪有应得。”

    你注意到它不责怪任何人,除了他们自己。所以,你看,中国人意识到了自己的错误。这就是为什么作为一个民族,他们愿意为一个模糊但最终真实的未来国家概念牺牲“鲜血和资本”。大多数情况下,它已经成功了。我说话不是出于无知;我以前住在上海,而且经常去那里。存在问题是很现实的,但也有成绩,而且政治话语和判断的总体水平还是不错的。

    所以不,根本不是超级人。矮小、软弱、缺乏创造力的人。但有时人们能够朝着同一个方向划船,愿意从过去的错误中吸取教训,同时寻求控制世界上他们唯一能控制的东西:他们自己。考虑到所有这些,他们愿意为教育投入大量的尊重、努力和金钱。

    • 回复: @Afrosapiens
  265. @res

    为了回答你之前的问题,Woodley 说除了说有轻微的上升之外,没有必要估计实际的智商点数。 这是因为样本相对较小和 DNA 回收不完整。
    我认为图 1 中的异常值没有得到更详细的研究,但我可以问这是否是有意为之,因为我认为一旦有更多数据可用,就会有更多的论文。

    • 回复: @res
  266. Santoculto 说:
    @szopen

    我会为你的灵魂祈祷。也许他会原谅你。

    我[认为我]知道的 G 将是所有 IQ 子测试中的共同点,而不是 IQ 本身。智商表现不一定是G,无论如何。

  267. szopen 说:
    @Afrosapiens

    关系和因果关系是两个不同的东西

    确实如此。

    它并不像你想象的那么确定,许多学习障碍发生在任何智商水平上。

    频率相同?

    或者也许医生的生活有助于在智商测试中获得更高的分数。

    不知道(因为我不知道任何专门关于医生的论文),但了解其他类型的论文我非常怀疑。例如,在一篇论文中,一个人在物理课程之前和之后对物理学生进行了测试。初步测试结果预测了学生期末考试的成绩。最终的智商测试结果与最初的测试没有显着差异。通常,年轻时的智商结果可以预测更好的生活结果(正如我在上一篇关于智商的帖子中提到的,即使在控制了社会经济地位之后,仍然是很好的预测因素)。我怀疑这对医生来说会有所不同。

    我不想重复自己的话,但将任何因果作用归因于智商(一种构造)都是具体化。

    智商可能确实是一种构造,因此不能“导致”任何事情。然而,智商旨在衡量认知能力并假设“一般认知能力”,能力水平的高低肯定会影响现实生活的结果。 “智商导致 东西那么,实际上只是诸如以下内容的简写:“假设的物理大脑在结构/代谢/有效性/其他方面的差异,我们假设智商是一个非常不完美的衡量标准,因为 东西“。

    还要考虑其他社会结构,例如“社会阶级”。社会阶层是一个不完美的衡量标准,因为“低阶层”、“中产阶级”等并不能真正捕捉环境的所有特征,并且在某种程度上是任意的区别(例如在我国很长一段时间)我们有“知识分子”课程,这并不适合任何英语课程,因为任何人都可以成为知识分子的一部分,无论他/她的收入如何)。但它们确实(不完美地)反映了现实。

    如果有人写下长篇论证“出生在低层阶级会增加被监禁的可能性”,你会通过说“将任何因果作用归因于社会阶级(一个构造)是一种具体化”来驳回这个论证吗?也许你会;但对我来说,这听起来在理智上并不令人满意,因为我觉得这更接近于挑剔,而不是攻击论点的要点。

    • 回复: @Afrosapiens
    , @RaceRealist88
  268. Santoculto 说:

    如果人类的进化正是为了对抗这种热带疾病,为什么根据这一假设,大多数人都受到了影响,据说降低了他们的平均智商/认知能力*

    因此,我们预计从他们生活在城市环境的那一刻起,他们的平均智商就会提高*

  269. @Afrosapiens

    这是一个足够清楚的观点,但可能与过去几百年的原始人类进化无关。

    • 回复: @Afrosapiens
  270. szopen 说:
    @Afrosapiens

    啊,。还有一件事。感谢您提供有关个性的链接。我不知道什么相关性会让我感到惊讶。他们中的某些我已经知道(对经验和幸福的开放性),其他一些我不知道,但那里没有什么令人惊讶的——除了“道德”,这可能会或可能不会令人惊讶,因为我不知道知道他们如何定义和衡量“道德”特质(因为谷歌给了我很多付费墙后面的论文,还有很多讨论道德是否可以衡量、应该如何衡量以及它实际意味着什么的论文)。

    • 回复: @Afrosapiens
  271. @Daniel Chieh

    如果这种所谓的承认错误并继续前进的倾向源自言论自由社会的民主选择,那么你确实可以相信中国人民的一些美德。

    但我们都知道,中国之所以成为现在这样,是因为共产党的唯一意志,你很幸运,你本来可以有一个朝鲜独裁政权,或者穆加贝式的独裁政权,对集体福祉的考虑要少得多——存在。

    将社会状况归咎于无权无势的公民是荒谬的。精英有自己的利益,有时这些利益与大众的利益相一致,但大多数情况下却不一致,如果大众对局势不满,他们就必须承受镇压的威胁。

    存在问题是很现实的,但也有成绩,而且政治话语和判断的总体水平还是不错的。

    这在任何地方都是如此,只是程度不同,与不同的情况有关。我无法告诉世界上有哪个国家客观上陷入了停滞或倒退,如果必须只有一个,那可能就是朝鲜。

    考虑到所有这些,他们愿意为教育投入大量的尊重、努力和金钱。

    对于每一个有机会投资同样多钱的非洲人来说都是如此。那么你想说什么?

    • 回复: @Daniel Chieh
  272. @res

    你提出这样的想法:我们可能期望找到有利于以一种或另一种形式屈服于强有力的领导的等位基因和/或以一种可能更复杂的方式对待任何领导者的等位基因,就像强大和令人钦佩的一样。

    昨晚有一部关于澳大利亚一战军官塞西尔·希利 (Cecil Healy) 的电视节目,他因在 1 年奥运会上表现出现在令人难以置信的体育精神而闻名,他坚持允许杜克·卡纳纳莫库 (Duke Kananamoku) 参加决赛,尽管他知道这会妨碍他尽管美国游泳运动员未能参加半决赛,希利还是赢得了金牌。第一次世界大战时,他最初担任军需官,但后来被调到前线,这样他就可以公平地分担其他人面临的危险。 (他在战争结束前 1912 天率领部队发动袭击时被杀)。成长经历、同伴和文化的影响显然是关键,但有一天知道这样的英雄在他们的 DNA 的某些部分是否往往具有独特的遗传特征将会很有趣。

  273. szopen 说:
    @RaceRealist88

    因为理查森论文声称,简而言之:
    (1) 他们对工作绩效的荟萃分析表示怀疑,在我看来,他们似乎在说“是的,通常荟萃分析比个人研究更好,但我们不喜欢结果”。当然,他们提出了有趣的观点,指出了真正的缺点,但“对以前的荟萃分析存在有效的怀疑”仍然不同于“我们证明以前的荟萃分析是错误的”
    (2)可能还有其他因素
    (3)最近的研究(据我所知没有进行荟萃分析)似乎不支持早期的结论。

    是的,这不是他们写的全部,但读了这些,我的印象是他们别有用心。这并不是说我认为他们错了。我认为一篇论文不足以反驳我认为文献中广泛接受的观点。我不是一个逆向投资者,当我阅读我的专业领域之外的文献时,我(通常)倾向于依赖当前最主流的观点,而不一定是最新的观点。

    我想我知道答案。

    RR,我们过去讨论过很多次,我真的不知道你为什么这么想 你知道答案。我已经接受了我过去不喜欢的发现。事实上,就在一年前,你似乎完全同意我的观点。事实上,我的印象是,就在几个月前,你似乎也同意这一点。为什么突然攻击我你不久前所担任的职位,并暗示我的动机与你不久前的动机不同?

    • 回复: @res
    , @RaceRealist88
  274. @Afrosapiens

    民主是对个人无知的集体智慧的可悲信仰。幸运的是,中国永远不会成为西方式的民主国家。

  275. Santoculto 说:

    丹尼尔·基耶(Daniel Chieh)
    雅典的民主与今天有很大不同,现代西方民主也不是直接的。例如,关于移民问题,西部地区的大多数人都反对大量移民。如果真正获得“集体智慧”,我认为我们有更好的政治选择。现在的民主是寡头制度,利用“人民代表”作为特洛伊木马。

  276. @szopen

    频率相同?

    我找不到任何量化它的研究,但这里有一些线索:

    本文的目的是检验支持“智力测试对于学习障碍的定义不是必需的”这一命题的逻辑和经验数据。检验了在学习障碍定义中使用智商测试分数的四个假设。这些假设是(a)智商测试衡量智力; (b) 智力和成绩是独立的,学习障碍的存在不会影响智商分数; (c) 智商分数预测阅读能力,智商分数低的孩子阅读能力较差; (d) 不同智商分数的阅读障碍儿童具有不同的认知过程和信息技能。有人认为,智商分数衡量事实知识、表达语言能力和短期记忆等技能,并且由于有学习障碍的儿童在这些领域存在缺陷,他们的分数可能会虚假地低。研究还表明,一些智商分数低的孩子也能成为良好的阅读者,这表明低智商分数并不一定会导致阅读能力差。经验证据表明,不同智商水平的阅读能力差的人都表现出相似的阅读、拼写、语言和记忆缺陷。从逻辑和经验的角度来看,智商测试成绩对于学习障碍的定义并不是必要的。

    http://journals.sagepub.com/doi/abs/10.1177/002221948902200803

    不知道(因为我不知道任何专门关于医生的论文),但了解其他类型的论文我非常怀疑。例如,在一篇论文中,一个人在物理课程之前和之后对物理学生进行了测试。初步测试结果预测了学生期末考试的成绩。最终的智商测试结果与最初的测试没有显着差异。

    在我看来,大脑可塑性需要比研究设计允许的时间更长的时间。

    通常,年轻时的智商结果可以预测更好的生活结果(正如我在上一篇关于智商的帖子中提到的,即使在控制了社会经济地位之后,仍然是很好的预测因素)。我怀疑对于医生来说情况会有所不同。

    它的预测程度是多少以及相对于 SES 测量的结果?我习惯的是智商研究人员根据 0.2 – 0.5 范围内的相关性得出结论,这实际上不足以令人满意地预测任何结果。

    智商可能确实是一种构造,因此不能“导致”任何事情。然而,智商旨在衡量认知能力并假设“一般认知能力”,能力水平的高低肯定会影响现实生活的结果。那么,“智商导致某些事情”实际上只是某些事情的简写,例如:“假设的物理大脑在结构/代谢/有效性/任何方面的差异,我们假设智商是一个非常不完美的衡量标准,导致某些事情”。

    简而言之,这是一个构造😉

    还要考虑其他社会结构,例如“社会阶级”。社会阶层是一个不完美的衡量标准,因为“低阶层”、“中产阶级”等并不能真正捕捉环境的所有特征,并且在某种程度上是任意的区别(例如在我国很长一段时间)我们有“知识分子”课程,这并不适合任何英语课程,因为任何人都可以成为知识分子的一部分,无论他/她的收入如何)。但它们确实(不完美地)反映了现实。

    你说对了一部分,但并不完全相同。您可以将特定文化和经济背景下的社会阶层与特定环境中的人们遇到的实际生活情况和挑战联系起来,这就是实地工作对社会科学如此重要的原因。民族学家/社会学家观察和采访人们,整理他们的轶事故事并发现总体趋势。这就是所谓的定性研究。还没有进行过这样的观察工作来比较具有任何特定智商的人的生活和行为。

    因此,虽然我们不能说“社会阶层让他这么做”,但你可以说“他的价值观/信仰/成长经历/社交网络/机会/困难让他这么做”,但你不能说“他的智商让他这么做了”。

    如果有人写下长篇论证“出生在低层阶级会增加被监禁的可能性”,你会通过说“将任何因果作用归因于社会阶级(一个构造)是一种具体化”来驳回这个论证吗?

    不,因为下层人民所面临的实际情况实际上在阶级与监禁的关系中发挥着作用。例如,较穷的人无法像富人那样接受救助,或者无法聘请优秀的律师,而这些在差异中发挥了直接的事实作用。

  277. @Wizard of Oz

    因此,寒冷的冬天就更不重要了,看看以前的原始人类如何在寒冷地区繁衍生息,看看动物王国中没有哪个北方品种比热带同类更聪明。

  278. @Daniel Chieh

    你的观点是指责,如果没有集体选择,就不可能有集体指责。当然,由于多数统治和间接代表权,并不是每个人都应该因民主选择而受到指责,但如果让群众对他们无法击败的独裁精英的行为负责,那就更荒谬了。

  279. Factorize 说:

    想想看,当我们知道智商和其他特征的因果变异时,我们很快就会对古代历史产生深刻的见解!

    我们可以对山上发现的冰人进行基因型智商和遗传行为评估。我们甚至可以通过等位基因的变化来追踪帝国的兴衰
    古代 DNA 样本的频率。

    如果能够出现更多的古代 DNA 样本,也许我们就能发现颜那亚对古代智慧的影响。如果有一个孤立的种群设法避免了这种入侵,那么我们就可以在欧洲种群被流离失所之前了解其遗传学吗?

  280. @szopen

    关于智商与性格的相关性,他们说:

    正相关:

    智力参与、智力创造力、思维敏捷、智力能力、内省、独创性、智力深度、想象力、组织力、韧性、挑衅性、领导力、自我表露、情绪稳定、节制和幸福。

    不相关的:

    对美的敏感度、群居性、友好性、自信、镇定、健谈、社会理解、温暖、愉快、同理心、合作、同情、责任心、效率、责任感、目的性、谨慎、理性、完美主义、冷静、冲动控制、冷静、冷静头脑清醒和平静。

    负相关:

    秩序、道德、养育、温柔和社交。

    我不知道这些相关性的重复性和跨文化有效性如何。与所有相关性一样,因果方向也是一个谜。我赞成性格导致智商差异的相反方向,因为比相反更容易弄清楚性格如何影响智商测试表现,特别是当你注意到智商并不使人们变得理性、道德、有序、善于交际时、合作、认真或冷静。所有这些我们期望的东西都是智能的产物,我想正如我们都想象的那样。

    出于这个原因,我很难相信智商只不过是铅笔和纸测试的分数,更不用说它可能受到自然选择的影响。

    也就是说,智商/人格相关性都很低:从智力投入的 0.423 到养育的 -0.193。

    • 回复: @szopen
  281. Factorize 说:

    丹尼尔,关于中国狗研究的非常令人兴奋的评论。我很高兴你了解了基因增强动物的想法。

    一段时间以来,我一直有一种预感,狗在人类的成功中发挥了重要作用,尤其是在战胜尼安德特人的过程中。人类本可以使用这些基因增强的动物作为对抗其他原始人类和其他动物的工具和武器。我看不出尼安德特人或其他人可以采取有效的对策来应对早期人类远距离控制的狗的攻击。

    开发更强大的动物版本作为工具可能非常有益。创造超级智能的黑猩猩、狗和其他动物将有一系列的应用。
    通过增加 100 SD 来优化动物智商将显着增强认知能力。
    那么可能就别无选择,只能承认动物与人类等同
    关于“人权”。

    这些动物将有一系列用途,但须经他们批准才能考虑。
    对于那些有认知障碍问题的人来说,超级智能动物可能是功能强大的伴侣动物。如果这些动物的智力足够高,我们完全有可能与它们进行某种形式的直接语言交互。

    此外,浣熊、乌鸦、渡鸦和松鼠等其他动物也可能经过基因改造,以充当人类和城市野生动物之间的桥梁。城市对自然环境的破坏很大,拥有缓冲动物可以让人与自然之间的关系更加和谐。例如,转基因鸟类可以帮助引导迁徙鸟类穿越城市环境或提供帮助。他们可能就像野生动物公园的护林员。

    给黑猩猩增加 100 SD IQ 将使它们的智力远远超过人类。
    我会收回我关于大猩猩的丑陋而毛茸茸的言论,因为你永远不知道。
    有一天,我们可能都会崇拜一种灵长类动物——大猩猩。

    对于增强人类能力有如此多的反对声音;似乎很自然地遵循这一点
    我们将被那些对基因工程毫无疑虑的动物统治
    例如老鼠或黑猩猩。

  282. Santoculto 说:
    @Factorize

    《中国对狗的研究》

    我也会为他们祈祷。

  283. Santoculto 说:
    @Factorize

    我们必须对你妈妈进行“基因工程”……

  284. res 说:
    @szopen

    但读了它们,我的感觉是他们别有用心。

    您可能会喜欢 Ken Richardson 的书的评论(以及相关评论): https://www.amazon.com/Genes-Brains-Human-Potential-Intelligence/dp/0231178425

    • 回复: @RaceRealist88
  285. @Factorize

    这可能会让您感兴趣,那么:

    https://www.technologyreview.com/s/542616/first-gene-edited-dogs-reported-in-china/

    http://www.nature.com/news/first-monkeys-with-customized-mutations-born-1.14611

    我特别认为狗会很有前途,因为它是更容易理解的动物之一,我想通过深度植入芯片和更高的智力/力量的结合,我们可以获得类似于机器人助手的东西。绝对充满希望。

    • 回复: @Santoculto
  286. Santoculto 说:
    @Daniel Chieh

    你们是愚蠢的怪物,支持所谓的“人性”中一切都会出错的事情。我希望“人类”尽快被毁灭,特别是所谓的“聪明”人类,无限的人类低能综合症。

  287. Okechukwu 说:
    @Daniel Chieh

    民主是对个人无知的集体智慧的可悲信仰。幸运的是,中国永远不会成为西方式的民主国家。

    民主是混乱的,但它比中国现在实行的任何治理模式都优越。最终,中国的长期前景黯淡。他们最终消亡的主要原因是他们的系统产生了什么样的人。我的女朋友是分子生物学的博士生。她的工作团队有各种各样的国籍,但她只抱怨中国人。中国人不会也不能偏离死记硬背的学习方式。他们的大脑似乎已经被重新连接,可以条件反射地拒绝任何与他们死记硬背的知识有微小偏差的东西。他们不会调查、挑战、质疑或探索。他们并不好奇。他们无法批判性思考。他们只是想机械地完成任务。除了一些被灌输到他们大脑中的核心能力之外,他们毫无用处。我女朋友必须不断地重新做他们的工作,她不希望他们靠近她的工作地点。这是因为中国人实际上什么都不知道,他们只是记住了某些东西。

    这些是中国最优秀的人,但他们一点也不令人印象深刻。许多年后,他们的英语仍然很糟糕,几乎不可能与他们交流。其他人都精通英语,但中文不行。这些机器人如何能够与像我女朋友这样的大脑不断渗透、搜索、探索、调查和创造的人竞争呢?我想他们可以通过继续作弊、偷窃和复制来进行一定程度的竞争。但这并不能带来非常光明的未来。

    • 回复: @random rand
  288. @Daniel Chieh

    另外,我很喜欢你的颐和园故事,但这是我发现的:

    http://www.bbc.com/news/magazine-30810596

    似乎与“我们应得的心态”截然不同。

    我还找到了一张内疚/恐惧/羞耻文化分布图。

    我再一次看不到中国人承认错误的美德,而这种美德会让中国跻身于负罪感文化之列。

    • 回复: @Daniel Chieh
  289. Factorize 说:

    丹尼尔,谢谢您的链接。

    我特别期待的是一次完成许多基因编辑的公告。
    性状是非常非常多基因的。

    要实现极端突破,需要对狗胚胎进行数千次编辑,或者正如我建议的那样,通过某种方法逐条染色体选择最佳配子/胚胎。

    一旦实现这一里程碑,一个新时代就开始了。
    我们不必等待人类道德批准可能发生的任何停滞。
    通过测量 g 来优化地提高一系列动物的智力就足够了。

    智商提高 100 SD 的黑猩猩或大猩猩肯定会成为宇宙中的阿尔法生物智能。人类关于智力至上的主张显然不再有效。

    • 回复: @Santoculto
    , @Okechukwu
  290. @Afrosapiens

    这是西方报道的典型特征,只关注他们想要的内容;然而,对自己负责并不排除对他人负责。需要两个人才能纠缠等等。

    是的,这种心态仍然存在,就像德比郡提到的“西方崇拜”心态一样。这是文化中的事情之一,就像天空是蓝色的一样明显。 “个人责任”是生活的重要组成部分,这就是为什么名誉自杀仍然是一种常见现象。这可以说得很大,而且这不仅仅是共产主义或中国意识形态的一部分。它也存在于整个台湾,任何儒家的地方,它是整个文化的重要组成部分。在禁止的角色范围内,您首先要对任何违规行为负责。

    这就是现实。你可以随心所欲地相信。

    • 回复: @Afrosapiens
    , @Okechukwu
  291. @res

    您认为哪种行为技能最相关?智力是其中之一吗?

    在这段对话中,智力是如何定义的?适应新环境的能力?行为可塑性?这是我们在非洲成立的。衣服、房屋和行为适应也有帮助。这些事情会妨碍“智商”的提高吗?

    您认为日本的智商与纬度相关性的可能解释是什么?

    老实说,我对日本历史一无所知,所以我无法给出一个好的答案(是的,我读了这篇论文),但我确实对意大利了解更多(论文中引用了这一点),林恩做了“智商研究” ' 使用 PISA 数据对他们进行评估。

    https://notpoliticallycorrect.me/2016/11/03/northsouth-differences-in-italian-iq-is-richard-lynn-right-part-ii/

    https://notpoliticallycorrect.me/2016/01/31/northsouth-differences-in-italian-iq-is-richard-lynn-right/

    以下是林恩对辩论的引用:

    http://citeseerx.ist.psu.edu/viewdoc/download?doi=10.1.1.613.5932&rep=rep1&type=pdf

    https://lesacreduprintemps19.files.wordpress.com/2012/03/iqs-in-italy-are-higher-in-the-north-a-reply-to-felice-and-giugliano.pdf

    等:

    https://www.gwern.net/docs/iq/2011-felice.pdf

    https://www.researchgate.net/publication/236631160_Problems_in_deriving_Italian_regional_differences_in_intelligence_from_2009_PISA_data

    https://www.researchgate.net/publication/223712957_The_mean_Southern_Italian_children_IQ_is_not_particularly_low_A_reply_to_R_Lynn_2010

    http://www.scienzeformazione.unipa.it/doc/225/D_Amico__Cardaci__Di_Nuovo___Naglieri_2012.pdf

    这种差异并不在于“基因”,南北之间不存在“种族差异”;较低的考试成绩可以用环境因素、重新上学来解释。

    这也是对林恩的一个很好的回应:

    http://www.emeraldinsight.com/doi/abs/10.1108/17465721111175056

    如果你能得到完整的论文(libgen 现在对我不起作用),作者给出了一个表格,比较了“IQ”的不同“衡量标准”(PSIA 和其他考试),并表明不同人之间的 IQ 没有差异。意大利北部和南部。

    注意最后两句话。

    我做到了。我是否声称寄生负载可以解释齿轮能力的整体变化?如果我这样做了,那么我道歉。

    以下是 Jensen 对 Templer 和 Arikawa 论文的看法:

    http://cyber.sci-hub.bz/MTAuMTAxNi9qLmludGVsbC4yMDA1LjA0LjAwMw==/10.1016%40j.intell.2005.04.003.pdf

    这种研究设计的主要局限性在于,从这种类型的分析中获得的相关性对于皮肤色素沉着的个体差异和智商的个体差异之间的任何因果或功能联系完全没有提供信息,也不能提供关于皮肤色素沉着的个体差异和智商的个体差异之间的因果关系的信息。相关性,例如,由于肤色和智商的交叉配种而导致的简单遗传关联与多效性相关性,其中两个表型不同但相关的性状都由同一个基因表现出来。

    此外,任何肤色基因(数量很少)与“智商基因”(“可能”有数千个以上)之间没有关系。那应该告诉你它是 只是相关性 并没有真正的意义。

    关于冬季寒冷的证据,《今日心理学》上有一篇很好的文章。

  292. @res

    你管这叫评论?

    “这可能是我一段时间以来读过的最无用的无聊读物之一。基本上,整本书都是在抨击科学和实验。太好了,那里有很多不好的数据,但是这本书只是一遍又一遍地重复相同的想法。如果你的智力正常,你不会从这本书中学到任何新东西。本书只是回顾了以前的研究,书中的假设也并不具体。这本书的基本总结是“很多关于基因的信息都是误导性的”。

    我拥有一份副本。出来的时候我就买了。我刚刚读了第二遍。理查森有很棒的想法,特别是,正如我们之前讨论的那样,智能系统/生理学以及智能细胞的发展。 (关于智能细胞和智能系统/生理学的第四章和第五章是我一段时间以来读过的最发人深省的内容。任何读过这本书的人都可以验证这一点。)

    这不是一个评论人。听起来就像有人读了第一章的一半并决定写评论。我也不是在开玩笑。阅读这本书并亲自验证这一点。

    现在,如果你想让我从他那里得到引言和参考文献,那么我们可以讨论这个问题,但是要链接到他的书的“评论”,该书实际上没有谈论理查森所说的任何内容(我怀疑这个人读过这本书)是不诚实的,你知道的。

  293. @Daniel Chieh

    这是西方报道的典型特征,只关注他们想要报道的内容

    LMAO,中国的“报道”从来不这样做。绝不。

    这就是现实。你可以随心所欲地相信。

    确实,你们的中国人道德优越论,从客观的现实来看,我是不会相信的。

    顺便说一句,说到自杀率,中国的标准化自杀率排名世界第115位,大多数非洲国家排名更高。
    https://en.wikipedia.org/wiki/List_of_countries_by_suicide_rate

    • 回复: @Daniel Chieh
  294. @Afrosapiens

    您应该知道,在亚洲文化中,自杀通常被认为是适当和光荣的。即使从流行文化中来看,这一点也应该是显而易见的。高考之后特别高,这不应该是一个谜,而恰恰是整个丢面子信仰体系的一部分。

    这次谈话没有进一步的目的。祝你的信念好运。也许你是对的。

    • 回复: @Afrosapiens
  295. @Okechukwu

    民主是混乱的,但它比中国现在实行的任何治理模式都优越。

    确实没有理由相信这一点。

    最终,中国的长期前景黯淡。

    也没有理由相信这一点。

    这些是中国最优秀的人,但他们一点也不令人印象深刻。 许多年后,他们的英语仍然很糟糕,几乎不可能与他们交流。

    这是西方神话中不会消失的神话之一,因为人们喜欢告诉自己这一点以使自己感觉更好。中国的人才流失并不是人们想象的那样。许多(大多数)在北美获得学士学位的中国人都没有获得最佳的中国大学入学考试成绩。另一大群中国移民和中国大学生是富人,他们可能腐败,必须离开,以防政府操他们。不用说,这些人并不是最优秀、最聪明的,他们中的许多人都是十足的白痴。事实上,其中很多人甚至可以进入北美顶尖大学,这实际上反映了北美顶尖大学的质量非常差。他们也并不比大多数北美“最优秀、最聪明”的人更笨。

    这些机器人如何能够与像我女朋友这样的大脑不断渗透、搜索、探索、调查和创造的人竞争呢? 我想他们可以通过继续作弊、偷窃和复制来进行一定程度的竞争。 但这并不能带来非常光明的未来。

    是的,考虑到中国目前正在研究时速 600 公里的磁悬浮列车,我想他们一定是在欺骗、窃取和复制瓦肯人的一些外星技术或其他东西,因为地球上还不存在这项技术。顺便说一句,唯一真正相信中国人无法创新的人是像张国登和詹姆斯·法洛斯这样的白痴。有人可能会说,在控制智商时,也许中国人的创新能力不如西方人,但中国人只能欺骗、窃取和复制的想法很搞笑。我怀疑你是张家辉的追随者。另外考虑到美国公司如何复制中国公司的电子商务平台,而不是相反,我想你可能会认为中国复制了脸书和亚马逊的超级秘密电子商务计划,除了中国人之外没有人看到。毕竟,如果他们只会欺骗、窃取和复制,他们怎么可能自己想出电子商务平台呢?

  296. @szopen

    不知道(因为我知道没有专门关于医生的论文)

    情报并不能独立预测退学、职业成果或其他指标。

    所有 511 名学生均在医学总理事会注册,但只有 464 名学生在 2001 年医学登记册上登记。离开登记册的 47 名医生(取得资格后平均 11.1 年;SD 5.9;范围 2-23)的 A level 成绩较低,但 AH5 分数并不较低(表 A, 宝马网); 看到 http://www.bmj.com 用于 ROC 分析。两名医生随后返回登记处。其余 35 人死亡,XNUMX 人的联系方式可查,XNUMX 人没有任何信息。

    智力不能预测职业,因此驳斥了能力论。 A 级别进行预测是因为它们评估成就,而结构模型显示过去的成就如何预测未来的成就。

    http://www.bmj.com/content/327/7407/139

    这表明智商对临床表现没有预测作用(晋升高级医生):

    https://bmcmedicine.biomedcentral.com/articles/10.1186/1741-7015-11-242

  297. utu 说:
    @szopen

    (1) 你认为桑代克在展示什么?我看了他1987年的论文 因子载荷的稳定性 他有 6 个不相交的电池组,每个电池组有 8 个测试,他在 17 个测试列表中附加了额外的测试,然后他观察负载向量(17 长)如何相互关联。他没有说明不同电池的 g 值如何进行比较。为了比较 g,应该关联 6×17 个不同的 g。我确信有些相关性很大,有些相关性很小。他不这样做。不管怎样,不同的电池产生不同的 g。

    (2) 但这里的全部意义不就是要找出是否存在潜在的“通用智能”吗? 对于那些倾向于极端还原论的人来说是个好主意。但它并没有成功。我们可以进行一组测试,产生两个强度相似的因素,或者一组测试,产生一个主导因素,而该主导因素与另一组测试产生的因素没有很好的相关性。这一事实足以推翻单一的通用智能概念。谁来决定哪一组测试是上帝赐予的?比奈?空军?法国塞夫尔国际政策与措施局?

    “每当我们进行不同的测试时,出现的主要因素总是高度相关的”
    没有真的。不总是。高度相关是多少?

    底线。

    每组测试都会产生一些g。这是一个微不足道的数学事实。因此,某人在某处得到一些 g 的事实不能用作任何事情的论据。

    不同电池测试的g值并不相同。它们之间可能具有高或低的相关性。

    一些测试组合可能会产生两个具有相似强度的特征向量。因此需要两个独立的 g 来解释大部分方差。

    • 回复: @szopen
  298. @szopen

    他们对工作绩效的荟萃分析表示怀疑,在我看来,他们似乎在说“是的,荟萃分析通常比个人研究更好,但我们不喜欢结果”。

    他们说这是因为调整夸大了相关性。这篇论文的要点是,所谓的智商和工作表现之间的高度相关性并不是对智商测试有效性的检验。其他生物功能都有描述它们的理论和模型,但智商却没有。呼气分析仪根据酒精如何流经系统的模型进行校准。没有这样的智商有效性测试。所谓智商和工作表现之间的高度相关性并不能证明智商测试是有效的。

    当然,他们提出了有趣的观点,指出了真正的缺点,但“对以前的荟萃分析存在有效的怀疑”仍然不同于“我们证明以前的荟萃分析是错误的”

    提出疑问很重要;指出还有其他解释很重要。我最大的问题是,正如他们指出的那样,主管评级是主观的,并且受到工作绩效以外的其他因素的影响。正如我所表明的,智力并不能预测医生的能力。

    最近的研究很少(据我所知没有进行荟萃分析)似乎不支持早期的结论。

    他们说,最近的研究表明工作绩效更加复杂。

    是的,这不是他们写的全部,但读了这些,我的印象是他们别有用心。这并不是说我认为他们错了。我认为一篇论文不足以反驳我认为文献中广泛接受的观点。我不是一个逆向投资者,当我阅读我的专业领域之外的文献时,我(通常)倾向于依赖当前最主流的观点,而不一定是最新的观点。

    你在激发动机的同时也暴露了你对这篇论文的偏见。

    RR,我们过去讨论过很多次,我真的不知道为什么你认为你知道答案。我已经接受了我过去不喜欢的发现。事实上,就在一年前,你似乎完全同意我的观点。事实上,我的印象是,就在几个月前,你似乎也同意这一点。为什么突然攻击我你不久前所担任的职位,并暗示我的动机与你不久前的动机不同?

    第一,我是一个逆向思维者。第二,我不断阅读新事物并挑战我的信念。在过去的一年里,我的观点得到了强烈的修正。

    事情的真相是这样的:没有 确凿的证据 智商可以预测工作表现(正如我向医生明确表明的那样)。

    • 回复: @szopen
    , @res
  299. szopen 说:
    @Afrosapiens

    非常感谢两位的评论。你有论文的链接吗?谷歌没有多大帮助(“考夫曼人格特质智力”没有给出答案),我很想知道他们如何衡量“道德”(或“理性”,因为我之前读过“理性”稍微有点)与智商相关)。

    我只想指出“0.423”的相关性并不低。我认为传统上这将被称为“中等”相关水平。

    至于 智商不会让人变得理性、道德、有序、善于交际、合作、认真或冷静,我不知道为什么应该这样。根据我的理解,“g”衡量神经系统的(某些)质量,尽管我不知道这种质量是什么。例如,如果高“g”分数表明更好的大脑连接性或更多的灰质(或者其他什么,我只是在这里提出假设),那么没有理由相信这也表明具有这种大脑的男性 必须 更加道德或更合作。好吧,说实话,相关性很低(我读过的其他热门文章中为 0.2 到 0.3) 做了 大约两三年前,当我第一次听到这个消息时,我感到很惊讶,但后来我意识到我的期望是没有根据的。

    这是一篇讨论智商和理性之间(缺乏)联系的热门文章:

    https://www.scientificamerican.com/article/rational-and-irrational-thought-the-thinking-that-iq-tests-miss/

    在我看来,这两个片段非常有趣:

    无论他们的智商如何,大多数人都需要被告知,完全选言推理对于解决这个难题是必要的,否则他们就不会费心去使用它。多伦多约克大学的玛吉·托普拉克(Maggie Toplak)和我已经证明,高智商的人在没有明确要求的情况下自发地采用析取推理的可能性只是稍微高一些。

    对于理性障碍的第二个来源,即心智缺陷,我们期望看到与智力的某种相关性,因为心智缺陷通常是由于缺乏教育而引起的,而教育往往反映在智商分数中。但与理性障碍相关的知识和思维方式往往要到晚年才被接受。聪明人很有可能在学校里从未被教授过概率思维、科学推理以及其他由 XYZ 病毒谜题和前面描述的四张牌选择任务衡量的策略。

    • 回复: @James Thompson
  300. PandaAtWar 说:

    @丹尼尔·切

    哈哈! 你不觉得,尽管可能有一些邪恶的、微弱的娱乐乐趣,但在这里与一些原始的磨斧智人进行争论,因为所谓的逻辑和事实的东西通常不会在他们的小大脑中记录下来吗?从进化的角度来说,它似乎才刚刚达到能够用无穷无尽的废话填充讨论线程的水平?

  301. Okechukwu 说:
    @Daniel Chieh

    这是西方报道的典型特征,只关注他们想要的内容;然而,对自己负责并不排除对他人负责。需要两个人才能纠缠等等。

    是的,将中国政府运营的宣传宣传品与经常与政府发生冲突的西方私人媒体进行比较确实很明智。西方媒体打倒西方政府。在中国,媒体就是政府。你口口声声吹嘘的高智商,似乎完全是你的想象。

    个人责任”是生活的重要组成部分……儒家等等等等

    是的,对。谷歌搜索中国商业诈骗或中国欺诈或中国金融犯罪或中国传销或中国出口诈骗或中国假冒商品或中国假食品或中国教育诈骗(这是他们向美国大学提交欺诈文件的时候)。一个仅仅因为欺诈、骗局和普遍渎职行为的严重性而在全面功能失调边缘摇摇欲坠的国家,并不是一个重视个人责任的国家。

    中国人在赚钱方面可以是无情的、毫无顾忌的,这就是为什么你必须是一个彻底的白痴才能把来自中国的任何东西放进嘴里。这可能是你吃的最后一样东西。

  302. PandaAtWar 说:
    @utu

    当然,将 g 用作万能的东西是不完整的,但 Panda 不会说这是“欺诈性的论点”。

    此外,某种普遍的“g”确实以某种方式仅凭共同直觉而存在,不是吗? 如果暂时不叫g,那么随着科学的进步,将来它会被称为“x”或“y”或“z”。

    最重要的是,目前大多数常见论点中的g并不是g本身,而是g的两层底层含义,具体体现为:

    1/ 迄今为止与所有其他度量的最佳共同相关能力,以及

    2/ 迄今为止最好的通用预测能力。

    简而言之,g 是预测。

    • 回复: @utu
  303. PandaAtWar 说:

    @詹姆斯·汤普森

    抛开“寒冷的冬天更难的长期计划”不谈,教授,在医学领域是否有关于绝对温度对功能性大脑作为生物器官的有效性的影响进行过研究?

    • 回复: @James Thompson
  304. Santoculto 说:
    @Factorize

    智商提高 100 SD 的黑猩猩或大猩猩肯定会成为宇宙中的阿尔法生物智能

    你妈妈比大猩猩更需要这个。你几岁*

    7岁*

    “数学倾斜”的典型案例:缺乏良性/接受性的同理心。

  305. szopen 说:
    @RaceRealist88

    感谢医生提供的数据。

    你在激发动机的同时也暴露了你对这篇论文的偏见。

    好吧,当我审视自己的动机时,我并不认为我对“反对智商有效性的论文”有偏见。然而,经过自我审视,我不得不承认,我可能潜意识里对任何与我观点相悖的论文抱有偏见。当然,我不应该,当然,你可以正确地指出这一点,但我认为每个人或几乎每个人都有类似的偏见,即使他/她试图这样做(当然我会努力克服我的偏见!)。我的偏见是暗示“我在这个想法上投入了很多思考,这个想法对我来说很有意义,我读过的很多论文都支持这个想法,因此当有东西与它相矛盾时,我不会立即放弃它,而是先想清楚”仔细地检查证据,等待对方发表评论,评估其答案的有效性,然后做出决定”。这种确认偏差是一种常见现象,如果你摆脱了这种现象,我不得不说你可以成为我的榜样😀

    至于理查森点,对于初学者来说,主管评级确实是主观的,但我不清楚它是否会偏向于智商得分较高的人(“对与我在社会上相似的人进行评级”),或者不是( “对那些更善于社交、和蔼可亲、不质疑命令、外向、和蔼可亲等的人进行评分”)。例如,我也不清楚,如理查森所说,对范围限制的修正是否真的会产生过度修正。当然,但不纠正范围限制肯定也会产生无效的结果(因为不是每个人都决定尝试成为一名医生,不是每个人都被大学录取并通过考试)。

  306. szopen 说:
    @utu

    utu,
    你从去年开始重复同样的论点,我从去年开始重复同样的反论点。

    ad 1)正如我在之前的评论中所写的:尽管测试的 g 负载是使用不同的电池计算的,但它们确实相关。如果在分析这些不同电池时出现的“g”确实不同,那么 g 负载不应该相关。然而它们是相关的,因此“g”不可能完全不同(与您的说法相反“来自不同测试组的 g 并不相同。” - 除非您认为 0.99 相关性仅仅是“高相关性”而不是“相同”我怀疑您是否会得到同一个人的两次血压测量之间的高度相关性!)。

    我没有写过桑代克比较“g”的内容。我写过他比较了不同电池中相同测试的 g 负载,并发现它们是相关的。
    我写过约翰逊的研究比较了不同电池的“g”(没有测试重叠),发现它们几乎相同。

    我还找到了她 2008 年的最新研究(“仍然只有 1 克”)。五种不同的电池,没有重叠的测试。 “g”提取了 Catell 的文化公平测试,其中包括矩阵运算,几乎没有其他内容,具有最低的相关性:与一般能力某事 (GATB) 的相关性仅为 0.76,与 TIB 的 ).79 相关,而与 GIT (Groninger Intelligence) 的相关性为 0.96测试),其中包括 4 项词汇测试中的 8 项。 AL0.95其他相关性在1.0或更高的范围内(TIB和GIT之间最高,1.0,GATB和TIB之间最高,2004)。 XNUMX 年的先前研究中未使用任何测试。

    再次,不同电池的“g”几乎相同。

    “每当我们进行不同的测试时,出现的主要因素总是高度相关的” 事实并非如此。不总是。高度相关是多少?

    当相关性 >0.5 时,传统上使用 IIRC“高度相关”。我在研究中读到相关性高于 0.7。如果您知道其他研究,我将不胜感激。

    2)常识。如果我们想检验是否存在“一般因素”,那么我们就应该利用不同的心理能力进行多种测试,检验所产生的因素是否是同一时期的。制造非多样化的电池对于提出这个问题没有任何意义。

    而且,据我所知,以这种方式构建的不同测试(具有许多不同测试的不同电池)创建的“g”是高度相关的(在我引用的研究中几乎相同)。

    ad 底线)一年前,当你第一次使用这个论点时,我已经写了这篇文章,并且我已经在这个线程中重复了这一点。事实上,一些“g”的出现是微不足道的。事实上,所有心理测试都呈正相关(尽管某些心理测试的相关性很低,这是事实),但事实并非如此。特别是当专门创建一些测试来反驳“g”理论时。事实上,不同测试的电池组产生的“g”是高度相关的(有时无法区分),这一事实并非微不足道。

    • 回复: @szopen
    , @Daniel Chieh
    , @szopen
    , @utu
  307. szopen 说:
    @szopen

    另一项研究发现:“不同儿童和青少年智力测试的一般因素是否相同? “五个样本、六个测试分析的结果”,2013 年,Randy 等人。对同一儿童进行的不同测试的“g”之间的相关性最低为 0.89,两个相关性“与 1.0 没有统计学差异”,

    他们还引用了另外两项研究,我还没有读过,据说发现不同“g”之间的相关性为 0.98、0.99 和 1.0,但他们似乎测试了相同的测试(使用不同的样本)。

    此外,在另一项研究(Salthouse 2013)中,我只发现了一个摘要,声称“对于理论上相似的结构”也具有高度相关性

  308. PandaAtWar 说:
    @random rand

    嘿,你先生! 你怎么敢用事实和逻辑?

    别说BLM了,你真的理解你对着一根木头弹钢琴所造成的心理创伤有多大吗?

  309. PandaAtWar 说:

    @詹姆斯·汤普森

    OP 中的年平均气温 (MAT) 图是当今世界的吗?

    您是否估算过至少 5,000 年前的 MAT 信息?

    • 回复: @James Thompson
  310. Factorize 说:

    对动物的研究将使基本心理测量概念变得更加清晰,并可能最终结束与这些概念相关的争论。

    如果实验室动物的 g 范围非常大,并且表现出显着不同的能力水平,那么就不再有理由认为这种差异不存在或它们与遗传学无关。

    尚未发现实验室动物是癌症和许多其他疾病/特征的良好模型。然而,他们可能会为 g 建立非常好的模型,因为 g 可以适应广泛的环境背景。

    由于g的高度多基因性质,现在人们知道有可能创造出具有非常极端智力的人类。由于各种原因,这可能需要一段时间才能在人类身上实现。

    g的高度多基因性质也应该存在于动物中,并且应该有可能创造出具有非常极端智力的动物。没有明显的理由说明为什么这些动物不会在中短期内被创造出来。

    这是一个非常令人兴奋的可能性,因为这些动物的智商很可能超过人类。

    • 巨魔: utu
    • 回复: @Daniel Chieh
    , @Santoculto
  311. @szopen

    我想补充一点,我非常喜欢你和@utu 之间的对话。根据我对大脑的理解,我稍微倾向于你的观点: g 不是简单定义的——但我确实认为它衡量了一些有意义的东西。我想说这相当于定义 g 作为一般健康状况的衡量标准,使用一定的心率,然后用它来解释人类的所有身体表现。科学家会发现相关性,但这肯定不是衡量一个人所有身体表现的最准确的标准。例如,对跑步速度的测试会发现,人类的“总体健康状况”与由于只有一条腿而无法跑步之间完全缺乏相关性。

    然而,这仍然并不意味着它作为人类健康的某种指标毫无用处。

    • 同意: szopen
  312. res 说:
    @RaceRealist88

    正如我所表明的,智力并不能预测医生的能力。

    这几乎没有被证明。你选择了一个经过精心挑选的群体(通过 MCAT 的智商以及通过医学院的坚持),然后将他们的智商作为预测因子。这有两个主要问题。

    – 范围限制。
    – 低智商医生可能会因与成为医生相关的其他特征(例如坚持不懈)而被高度挑选。

    逆向思维和持怀疑态度固然很好,但在某些时候,在批评别人的过失之前,最好先去掉自己眼中的梁木。

    我期待大家都说智商无助于医术,说到做到,从现在起专门光顾85位智商医生。

    PS 关于最后一点,这里有一项研究,考察了因涉嫌医疗错误而转介的医生的表现: http://www.sciencedirect.com/science/article/pii/S0887617704000769

    一个样品:

    这项对 148 名因医疗错误指控而接受评估的医生的神经心理学表现进行回顾性审查的结果表明,作为一个群体,他们的智力普遍处于平均水平。该医师群体的平均智力功能与之前评估绩效困难的医师报告一致,但低于医师整体功能的预期水平(Turnbull 等,2000;Madden,1988)。之前关于“未受损”医生智力功能的报告表明,拥有医学学位的人的平均智商为 125(Matarazzo & Goldstein,1972;Wecshler,1972),这比该群体的平均表现要高得多。 Matarazzo 和 Goldstein(1972)还检查了普通医学生的智商,以确定“进入医学生的智力水平下降”(第 102 页)的说法是否正确。这些作者发现,与所谓的论点相反,他们的医学生样本与 10 年至 1946 年间其他 1967 个医学生样本的智商表现相似。在多项研究中,医学生的平均全量表智商为 125,与当时医生的智商相似。

  313. @Factorize

    然而,他们可能会为 g 建立非常好的模型,因为 g 可以适应广泛的环境背景。

    我不知道为什么会这样。例如,如果情况总是如此,那么我们就会拥有高度聪明的蜗牛。智力仅在某些情况下才有用,例如,工具创建作为一种技能只有在存在操纵和使用工具的附属物时才有用。

    如果实验室动物的 g 范围非常大,并且表现出显着不同的能力水平,那么就不再有理由认为这种差异不存在或它们与遗传学无关。

    这已经是众所周知的了。

    https://medicalxpress.com/news/2014-12-adding-human-glial-cells-mice.html

    我怀疑我们是否能够像你建议的那样以极端的方式扩展情报。大多数生物过程都经过合理但不完美的优化。

    • 回复: @res
  314. Santoculto 说:
    @Factorize

    对动物的研究将使基本心理测量概念变得更加清晰,并可能最终结束与这些概念相关的争论。

    不。 G 因子是可以理解的,无需需要“测试”似乎要感知的显着内容的伪超级分析者。这就像“测试”研究一样,旨在证明贫困是否[总是、大部分]导致暴力行为。在理智的世界中,当很容易观察到这种模式时,这种类型的研究就是无聊的。大多数研究都是为了让学术界中处于重要等级地位的愚蠢人士相信,他们的隐性偏见和情感偏见是错误的。

    如果实验室动物的 g 范围非常大,并且表现出显着不同的能力水平,那么就不再有理由认为这种差异不存在或它们与遗传学无关。

    不存在“g范围广”这样的事情。只有一个g。

    尚未发现实验室动物是癌症和许多其他疾病/特征的良好模型。然而,他们可能会为 g 建立非常好的模型,因为 g 可以适应广泛的环境背景。

    你妈妈是肥胖意识的好榜样,反社会者!

    只有平庸的科学家才需要“动画模型”来测试你的假说。

    由于g的高度多基因性质,现在人们知道有可能创造出具有非常极端智力的人类。由于各种原因,这可能需要一段时间才能在人类身上实现。

    创造这样的生物是为了什么*
    他们会一有机会就消灭你。

    多基因性 统计 文物,耶稣永恒!!!
    卫生纸的基因。

    大多数智力研究人员甚至不知道智力、创造力或理性到底是什么……

    “没有明显的理由说明为什么这些动物不会在中短期内被创造出来。”

    我赞成非人类动物的天然优生模型。而不是那个疯狂的科学家测试。

    是的,有非常明显的道德理由来阻止那些疯狂的科学家及其不负责任的创造力。

  315. utu 说:
    @PandaAtWar

    1/ 迄今为止与所有其他度量的最佳共同相关能力,以及

    2/ 迄今为止最好的通用预测能力。

    这是您对我的观点的回应,即不同的测试组会产生不同的 g,而彼此之间不一定有很好的相关性?无论你说某个神奇的 g 是最擅长的,这两个不同的 g 中哪一个是“最好的”?

    • 回复: @PandaAtWar
  316. szopen 说:
    @szopen

    啊!我发现不知何故我删除了整个句子!

    这里:

    (与你的说法相反,“不同测试组的 g 值并不相同。”——除非你认为 0.99 相关性仅仅是“高相关性”而不是“相同的”。我怀疑你是否会得到两种血压测量值之间的高相关性是同一个人!)。

    首先应该有一句话“另外,我还提到过 Johnson [2004] 比较了对同一个人进行的三组测试,发现“g”是无法区分的 ((与您的说法相反,“不同测试组的 g 值并不相同。”——除非您认为 0.99 相关性仅仅是“高相关性”而不是“相同的”。我怀疑您是否会得到两种血液测量值之间的高相关性同一个人的压力!)”

    • 回复: @utu
  317. utu 说:
    @szopen

    事实上,一些“g”的出现是微不足道的。事实上,所有心理测试都呈正相关(尽管某些心理测试的相关性很低,这是事实),但事实并非如此。

    首先正确。第二个也是微不足道的,因为您承认自己确实发生了低相关性。还因为一系列测试是由具有共同信念和偏见的人创建的,其中包括共同的信念和偏见。

    你从去年开始重复同样的论点,我从去年开始重复同样的反论点。

    也许是真的。我对此承担责任,因为我缺乏说服力,你犯了错误,而且可能会继续如此。

    • 回复: @szopen
  318. Anon • 免责声明 说:
    @jason kennedy

    返回 tumblr,然后参加白色内疚课程。

  319. @random rand

    别对张家港小气,他是天国大英雄。随便问一个中国网友,他都是SFB*的重要成员。但老实说,他无意中很搞笑,我真的很喜欢他的文章。 2010年最佳汉奸.

    *战略傻瓜局。

    • 回复: @random rand
  320. utu 说:
    @szopen

    这是一条没有出路的路。我关于 g 不相同或相关性较差的主张得到了两组测试的证明,这些测试产生了两个相关性较差的 g。我认为有很多结果可以证明我的主张。你的说法要困难得多,而且无法通过经验得到真正证明。因此,您将继续列出具有类似 g 等的论文,但这仍然不能证明您的主张。任何两组产生显着不同 g 的测试都可以证明您的主张是错误的。

    • 回复: @szopen
  321. Anon • 免责声明 说:
    @Afrosapiens

    >世界上大多数人,尤其是非洲人都会嘲笑测量智力的想法。世界上大多数国家,尤其是非洲国家,对人类价值有着其他的观念。

    甚至中国当局也承认情报差距。

    >LMAO!我根本不在乎“和谐存在等等”。我只关心我的女朋友、我的家人和朋友,他们需要我,就像我需要他们一样。

    在这里,我们看到黑人承认不尊重他的东道国,并计划让所有的垃圾到处乱窜。

    >欧洲人真的不需要我们互相毁灭。如果没有我们,他们就会像日本一样走向衰落。

    啊,是的,日本是地球上犯罪率最低的国家,人均国内生产总值最高的国家,领导层和娱乐界远不像欧元那样对民众充满敌意,也不像黑人那样灾难性(津巴布韦怎么样?正在做?)。什么,不是他们在人口稠密的岛上没有更多的孩子吗?这就是所谓的自动化,将移民或不属于你的人拒之门外。

    请告诉我们多贡戈和贾马尔的许多贡献。我的意思是,是通过福利利用、恐怖主义、艾滋病和寻找白人妇女来强奸致富吗?

  322. Anon • 免责声明 说:
    @Daniel Chieh

    康的帖子表明了黑人的忘恩负义、寄生主义和自大妄想。

  323. Anon • 免责声明 说:
    @YetAnotherAnon

    日本需要更多的移民来利用他们的资源并丰富日本妇女。

  324. @PandaAtWar

    是的,有针对不同温度条件下的性能的研究。手头没有参考资料,但我记得应用心理学部门研究过这类问题。通常,表现会有所下降,但这取决于训练的程度和主体动机,如果这两者都很高,那么即使在困难的条件下也可以维持合理的表现。另外,海湾战争期间为军队进行了生物防护服的研究,我认为在沙漠条件下性能下降了 30%。

  325. @PandaAtWar

    赫特勒正在研究这一复杂的问题,即观察较长时期内的气候变化,以及人们从一个生物群落迁移到另一个生物群落时的变化。 这还需要估计几代人的选择压力,以便为这些生物群落变化的假定影响提供衡量标准。 正如我所解释的,这是试图改进关于温度本身影响的争论,并考虑更广泛的因素。

  326. @res

    正如你所说,医疗人员的选拔是为了达到一定的成就,这导致了范围的限制。克里斯·麦克马纳斯研究医学生数十年。这是他最近发表的一篇论文。
    https://www.unz.com/jthompson/representativeness-beats-size

    • 回复: @RaceRealist88
    , @res
  327. @Daniel Chieh

    您应该知道,在亚洲文化中,自杀通常被认为是适当和光荣的。即使从流行文化中来看,这一点也应该是显而易见的。高考之后特别高,这不应该是一个谜,而恰恰是整个丢面子信仰体系的一部分。

    无关紧要,这些统计数据只是表明,自杀无论是与荣誉还是其他有关,在中国都不是大事,所以不要用它来论证中国人有如此强烈的个人责任感,以至于集体自杀。

  328. @res

    昨晚我写了一些关于此的文章,并且刚刚发现了更多关于此的论文。关于您所链接的问题有大量文献。创伤后应激障碍(PTSD)、抑郁症等被认为是造成这种情况的一些原因。

    https://notpoliticallycorrect.me/2017/10/04/doctors-iq-and-job-performance/

    我发现的一项有趣的研究表明:

    大多数医学生的智力接近平均水平(88.3%),他们为医学入学考试进行了一到两年的准备,并投入了大量的时间(>6小时)。 10%的学生智商较高,自学时间较少,但上课很真诚

    智商接近平均水平的学生学习努力,他们的学习成绩与智商较高的学生相似。所以智商不能作为考医的依据;相反,对中学成绩给予年龄权重并限制尝试次数可能会缩短进入和完成 MBBS 学位的时间。

    http://imsear.li.mahidol.ac.th/handle/123456789/175546

    稍后当我可以使用我的笔记本电脑时,我将深入探讨。

    Szopen,今天下午晚些时候我也会回复你。

    • 回复: @res
  329. @James Thompson

    汤普森博士肯·理查森在他的新书中引用了麦克马纳斯 2013 年的研究,并指出智商并不能预测晋升为高级医生。

    McManus 等人也有一篇 2003 年的论文,他们写道:

    情报并不能独立预测退学、职业成果或其他指标。

    ...

    智力不能预测职业,因此驳斥了能力论。 A 级别进行预测是因为它们评估成就,而结构模型显示过去的成就如何预测未来的成就。

    http://www.bmj.com/content/327/7407/139

    • 回复: @James Thompson
  330. res 说:
    @Daniel Chieh

    我怀疑我们是否能够像你建议的那样以极端的方式扩展情报。大多数生物过程都经过合理但不完美的优化。

    我对此也有一些怀疑,但您可能想阅读一下 Steve Hsu 的博客(如果您还没有)。恕我直言,他与动物繁殖经验的类比相当引人注目。困难的部分是判断哪些特征更多/更少优化和/或受到物理限制。

  331. @RaceRealist88

    我认为这是范围限制问题。麦克马纳斯说,所有的预测因素都相当弱,一种解释是,选择是在申请医学院之前就完成的,因为每个学科都会宣传他们的要求,而就医学而言,他们的拒绝率很高。

    • 回复: @Daniel Chieh
  332. Factorize 说:

    丹尼尔,下面网址的第 19 和 20 页描述了人类最大智力表型一直被隐藏的逻辑。

    https://arxiv.org/abs/1408.3421

    研究表明,人类智商达到 1500 是可能的。
    这也意味着其他动物具有非常极端的智力表型
    被发现具有可测量g值的动物包括黑猩猩、乌鸦、狗和其他动物也应该是可能的。

    如果这个想法进入主流对话,将有助于推动技术向前发展。

  333. @James Thompson

    汤普森博士,我很好奇您对托伦斯测试作为测试创造力的衡量标准的看法。

    • 回复: @James Thompson
  334. szopen 说:
    @utu

    一系列测试是由具有共同信念和偏见的人创建的,其中包括共同的 g。

    但这就是重点!这些电池也是人们试图反驳“g”、试图证明“多元智能”存在而制造的,但它们仍然是相关的——尽管他们不相信存在一个共同的“g”!

    • 回复: @utu
  335. Santoculto 说:
    @James Thompson

    是的,老狼,他们错过了……你也错过了……你就是缺乏理性的活生生的例子,至少在最高标签上……

    人们理性地思考,就像他们思考认知同理心一样。理性不仅仅是智商测试所衡量或比较的认知技能,而是在与心理技能的互动中良好或出色地运用认知技能。

    理性几乎不能简化为非情境/非现实世界文化测试。

    简单的事实是,大多数高智商的人倾向于以极端的方式左倾或右倾:在社会或经济上自由,已经表明他们是多么不理性。

    但我同意最高理性水平和高于认知技能之间存在相关性,但并非如此……因为“智商不会错过理性=智商也衡量理性=大多数高/最高智商的人[更]理性”。

  336. szopen 说:
    @utu

    所以你的主张基本上是:

    是的,每次有人试图实际检查许多不同测试的不同电池的“g”是否相同时,它们总是相关的(我认为较低的值是0.76),但这并不重要,因为可能有一系列不同的认知能力测试,它们会产生与其他“g”不相关的“g”。我知道不存在这样的电池,但并不是所有电池都经过检查,这就是为什么我认为存在很多这样的电池,这就是为什么到目前为止检查的所有电池产生的“g”都是不同的。此外,加德纳实际上相信“G”,尽管声称“g”不存在,这就是为什么他的测试旨在证明许多智力存在,尽管如此,当提取“g”时,它与从中提取的“G”的相关性为 0.76 Wonderlic 人员测试。

    如果我错过了什么,我很抱歉。在波兰,我们实际上并不以情报而闻名,这可能是我对这种推理不感兴趣的最可能的原因。

    • 巨魔: utu
    • 回复: @utu
  337. utu 说:
    @szopen

    而且它们仍然是相关的

    不对。

  338. Okechukwu 说:
    @random rand

    许多(大多数)在北美获得学士学位的中国人都没有获得最佳的中国大学入学考试成绩。另一大群中国移民和中国大学生是富人,他们可能腐败,必须离开,以防政府操他们。

    与本例无关。这些是已经拥有中国本科学位的中国研究生。我不会贸然说中国人愚蠢或低劣。我将把霸权的主张留给这里的亚洲傻瓜。但可以肯定的是,造就这些人的是中国的制度,而不是某些先天的倾向。他们的教育体系向他们灌输了大量信息,但他们从未学会如何应用这些信息。这有点像为了考试而临时抱佛脚,然后在你的大脑中塞满该测试所需的数据。你的大脑可以在测试时回忆起这些信息,但你并不真正理解这些信息的深度和范围。

    是的,考虑到中国目前正在研究时速 600 公里的磁悬浮列车,我想他们一定是在欺骗、窃取和复制瓦肯人的一些外星技术或其他东西,因为地球上还不存在这项技术。

    这些都没有在中国有机地萌芽。磁悬浮技术当然没有,火车或铁路也没有。

    我怀疑你是张家辉的追随者。

    张家辉给我的印象就像一位亚洲汤姆叔叔。我鄙视这样的人。在
    从气质上看,他与我们这里的亚洲至上主义评论者相差并不远。他们走了不同的路,却到达了同一个地方,那就是白人中某些反动种族主义分子的认可。

    还要考虑美国公司如何复制中国公司的电子商务平台,而不是相反

    如果让他们自己使用设备,中国人就不会拥有任何电子商务、互联网或计算机。人们还效仿 M-Pesa 等非洲移动货币平台。但我们知道这项技术并非起源于那里。

    • 回复: @random rand
  339. utu 说:
    @szopen

    我对你是谁或你是什么不感兴趣。我不在乎你是不是华沙动物园里的一只猴子,只要你能提出令人信服和连贯的论点,并表现出良好的意愿来承认有效的论点,特别是当它们不符合你的喜好时。

    • 回复: @szopen
    , @szopen
  340. res 说:
    @James Thompson

    感谢您的连接。

    McManus 2003 年的论文(感谢 RR)似乎很相关,因为它明确尝试使用 AH5“高级”智力测试来测量智力。

    您知道 AH5 标准化(例如转换为 IQ)吗?经过大量搜索后我找不到该信息。他们给出的平均分是 40.4,但没有翻译成:

    AH5 平均总分(图 A, 宝马网)为 40.4 与标准相似,8 言语和空间分数也与标准相似(表 A, 宝马网)。 A level 平均分(图 A, 宝马网) 为 4.00,相当于 BBB 级。 AH5 分数和 A level 等级相关(Pearson r=0.285,P < 0.001;图 B, 宝马网).

    补充材料中的表 A 提供了有关 AH5 分数的更多信息:
    AH5手册对大学生的规范:
    总分 39.1 (8.3)
    医学37.5 (7.5)
    科学39.0 (8.0)
    艺术34.6 (7.5)

    对于研究医生,他们引用:40.4 (7.7; 19-62)
    我认为 7.7 是 SD,19-62 是范围。整个分布如图 A 所示。与受到严格上限限制的 A Level 相比,该分布呈正态分布。我想知道如果AH5最高分集中在45-50以上,分析结果是否会改变。

    了解更多有关图 B 中左下角异常值的信息将会很有趣。有些人在招生方面遇到了严重的困难。

    • 回复: @James Thompson
  341. Anon • 免责声明 说:
    @Okechukwu

    在这里,我们看到一个黑人正在全力以赴。

    >马克·迪恩

    https://infogalactic.com/info/Mark_Dean_(computer_scientist)

    确实发挥了作用,但没有发明它。

    >杰瑞·劳森

    https://infogalactic.com/info/Home_video_game_console

    拉尔夫·H·皮尔 (Ralph H. Pier) 是第一个设计家庭电子游戏的人。 Magnavox Odyssey 是第一款游戏机。

    >一位黑人在开发我们今天拥有的手机技术方面发挥了重要作用,而你们的三星已将其变成了价值十亿美元的业务(参见亨利·T·汤普森)。

    http://conservative-headlines.com/2014/02/black-history-month-myth-of-the-day-the-cell-phone/

    即使像维基百科这样的网站也无法为你的“We Wuzzing”提供任何可信度:

    https://www.wikipedia.org/wiki/Mobile_phone

    >更多我们伍兹

    https://www.wikipedia.org/wiki/Propeller#Early_developments

    https://www.wikipedia.org/wiki/John_Stevens_(inventor)

    https://en.m.wikipedia.org/wiki/John_Ericsson

    奖励:

    http://www.niggermania.com/tom/blackinventionmyths/index.html

    >诺贝尔奖

    低信任群体(黑人、穆斯林)的成员会说,北欧有某种阴谋将非北欧人拒之门外,就好像其他人都和他们一样。

    除此之外:

    http://www.jinfo.org/Nobel_Prizes.html

    https://www.wikipedia.org/wiki/List_of_Jewish_Nobel_laureates

    阿肯纳兹和塞法迪犹太人在诺贝尔奖获得者中所占的比例太高,以至于无法喋喋不休地谈论人口或北欧特权(或者犹太人现在突然被接受为雅利安人同胞)?

    • 巨魔: Okechukwu
  342. szopen 说:

    @詹姆斯·汤普森
    我们可以指望未来发表一些关于测试电池的“g”稳定性和“g”负载稳定性的文章吗?我发现了另一项工作,是温迪·汤普森指导下的梅杰的论文,再次发现从不同电池中提取的“g”之间存在高度相关性,但我想专业人士比我们这些凡人知道更多,并且可以详细阐述这个主题。

    • 回复: @James Thompson
  343. res 说:
    @RaceRealist88

    印度(!)的研究很有趣。我的第一点是,他们使用的智商测试是 智商测试网 测试。我认为这不会被认为是一个特别好的智商测试。

    如果我理解正确的话,他们会按时间和分数将分数分为一个 SD 范围(平均 85-114 除外)类别。考虑到丢失的信息量,这似乎是一个糟糕的想法。我无法确定地解释每个垃圾箱使用的确切阈值,但看起来可能存在一些严重的不匹配。比如55分钟得15分是平均分,21分钟得13分就高于平均分?!

    使用 15-20 分钟、11-13 分钟和 <11 分钟之间的差异来区分平均 (85-114)、高于平均 (115-129) 和天才/天才 (130-144 和 145-159) 似乎对我来说很荒谬。分数限制看起来更合理,但低于/平均差距仅为 15 至 20,相对于平均/高于 20 至 45 的最大跳跃。我真诚地希望我误解了他们的所作所为。

    坦率地说,在尝试解释表 2 后,我想知道作者(和同行评审员)的智商。如果有人认为有道理,请给我解释一下意思。

    整篇论文并没有给我留下特别深刻的印象,但最好让其他人自己评估一下。

    PS 你读过全文吗?你认为他们的智商评分方法合理吗?

  344. szopen 说:
    @utu

    Utu,但是当你发表这样的论点时,你对我有什么期望?是的,并非所有电池都经过测试。但我能找到的所有实际上试图检查不同测试的“g”的研究都发现了高度相关性。您声称“存在许多电池(通过许多不同的测试)”,这会产生不同的“g”,但您没有发布一项研究来证明您的说法,您只是说“存在一项这样的研究就足够了”。根据你的标准,不可能证明“g”存在(因为理论上有人可以设计一系列新的测试,满足“许多不同的心理能力测试”的标准,也许,只是也许,这不会与其他测试相关) “g”)。

    但这不是问题——我在方法论课上学的是波普尔方法,在这种方法中,你不证明假设是正确的,你只是不断地试图反驳它,并且你应该接受它为真,直到你反驳它为止。脚注中还提到,如果你收集了许多证实,那么不相信一个反驳也是可以的。这就是为什么我不认为你的主张是反对“g”的有效论据。

    但这里有一点要告诉你:我错误地认为这些测试是由加德纳设计的。草率地阅读了我的观点,我深表歉意。

    加德纳没有设计测试,他只是陈述并描述了多元智能的存在。 Visser 等人随后设计了尽可能符合加德纳描述的测试。是的,并非所有相关。运动智力(即单腿站立时保持平衡)测试呈负相关或零相关,准确性(准确反映自己的强项和弱点)和音乐(音调和节奏识别)具有混合相关性,左右摇摆。

    再一次——你和我撤回加德纳和他的军事情报人员的论点。不过,他的MI也没有人反对。

    • 回复: @Santoculto
    , @utu
  345. szopen 说:
    @utu

    经过一番思考,我对我的语气感到抱歉。我的脾气非常暴躁,多年来我努力在网络和现实的日常社交互动中保持礼貌。尽管如此,有时我还是表现得像个混蛋。我不应该取笑你的论点,我应该以不同的方式表达我对其有效性的反对意见。我道歉。

    • 回复: @Daniel Chieh
  346. Okechukwu 说:
    @Factorize

    智商提高 100 SD 的黑猩猩或大猩猩肯定会成为宇宙中的阿尔法生物智能。人类关于智力至上的主张显然不再有效。

    很搞笑。那永远不会发生。进一步研究的途径之一是通过阿尔茨海默氏症和帕金森氏症的研究,这可以消除许多伦理上的反对意见。大多数黑猩猩的身体学习能力(如果不是社交学习能力)与人类 2 岁或 3 岁儿童的水平相当。那些在较高年龄范围(比如 7 或 8 岁)发挥作用的动物是例外,并且在圈养条件下数量不多。这些是最合适的研究课题。也许可以通过化学方法增强神经连接,从而提高注意力。

    这项研究探讨了注意力和智力之间的联系:

    感官辨别与智力之间存在紧密的互动联系

    https://www.ncbi.nlm.nih.gov/pmc/articles/PMC3702042/

  347. @szopen

    题外话,我应该提一下,我非常钦佩你们国家的勇气。波兰似乎在日益疯狂的欧洲中发挥了重要作用。

    • 回复: @szopen
  348. szopen 说:
    @Daniel Chieh

    谢谢。不幸的是,我们也有不少白痴、SJW 和疯子。很难说要多久情况才会变得像西方那样糟糕🙁甚至我们的政府也有纯粹疯狂的时刻。例如,政府急于制定针对“假新闻”的新法律,似乎没有意识到它将被用来对付独立记者,很可能是针对右翼分子。

    • 回复: @Daniel Chieh
  349. Santoculto 说:
    @szopen

    我已经说过了,但没有人记得,也没有人关心,无论如何。我相信无论智商得分如何,都存在一种智力的最佳利用方式。请记住,智商测试是关于文化-遗传共同进化的,就像镜子一样,所以智商是文化的,但许多人相信的方式不同。这种最佳用途似乎是“可遗传的”(又名遗传/可遗传)或可预测的。

  350. @szopen

    我认为,随着欧盟的命运因无数自伤而继续螺旋式下降,波兰的右翼运动将越来越被证明是正确的,寄生虫将不再那么亢奋地继续颠覆,因此仍然有很大的希望。在逐底竞争中,您只需要避免先落地即可。并保持这些栅栏坚固。

    • 回复: @utu
  351. Factorize 说:

    Okechukwu,我不仅仅是在挑衅。

    你对黑猩猩认知能力范围的评估是相当准确的。然而,如果我们要转向更优化的认知基因型呢?

    下图表明人类基因组中有数千个 IQ SNP。黑猩猩基因组中肯定也存在数千个 IQ SNP。两者之间可能会有相当多的重叠。如果我们可以拥有 1500 个智商比正常人高出 100 个标准差的人类,那么为什么我们不能通过基因工程改造出比正常黑猩猩智商高出 100 个标准差的黑猩猩呢?智力与 8 岁儿童相当的黑猩猩仅比人类成年人的智商平均水平低 3 个标准差。智商加起来达到 100 SD 将使黑猩猩远远超出可观察到的人类认知能力范围。

    没有理由怀疑黑猩猩无法达到 100 SD。我发现特别值得注意的是,与人类相比,提高黑猩猩智商的障碍要少得多。如上所述,几年前通过基因技术提高了小鼠的智商。我认为现在可能正在研究一个更令人信服的动物增加 g 的例子,这是合理的。

  352. @Daniel Chieh

    没有使用它的经验。对于这个话题的总体看法,雷克斯·荣格(Rex Jung)是我的首选。
    https://www.unz.com/jthompson/heave-half-brick-at-creativity

  353. @res

    是的,有些医生很迟钝,但也有一些非常聪明的医生,和他们在一起很愉快。差异主要体现在他们的研究成果上。此外,最好的外科医生倾向于修正,即纠正能力较差的外科医生的错误。我今晚就推荐了一个。
    AH5。我咨询了一位德高望重的同事,她告诉我我们的书柜里没有手册,只有一些测试表格。尽管她完全是在爱丽丝·海姆的鼓励下转向心理学,但这一切还是发生了。
    我快速谷歌了一下,所以你可以尝试从中得出它

    https://www.psyjob.it/wp-content/uploads/2012/08/GRT2Man.pdf

    第 6 页的相关表

    • 回复: @res
  354. @szopen

    Ian Deary (passim) 是关于这个主题的好人。

    https://www.unz.com/jthompson/the-99-steps-of-intelligence-hunters

    自 1904 年发现以来,数百项研究重复了这一发现:人们在各种认知测试中的测试成绩差异中,大约 40% 可以由一个一般因素来解释。就其经验支持和对生活结果的重要性而言,一般认知功能在人类心理特征中是无与伦比的。一般认知功能的个体差异在生命历程的大部分时间里是稳定的。双胞胎研究发现,从青春期到成年再到老年,一般认知功能的遗传率超过 50%。基于 SNP 的一般认知功能遗传力估计约为 20-30%。迄今为止,这种巨大的遗传性还没有得到解释。仅发现了一些相关的遗传位点

  355. Factorize 说:

    谁能推荐一些我可以用来评估我的智商的在线网站。我已经参加了大量的认知测试,但我唯一确定的是我的智商在 70 到 170 之间。心理测量理论表明,当记录越来越多的反应样本时,应该可以进行非常准确的智力评估。

    有人帮忙吗?

  356. res 说:
    @James Thompson

    我无法找到从该 PDF 中导出它的方法。我想我需要一些可以建立共同参考点的东西。我可以尝试假设 AH5 为 40.4,智商为 125,但这几乎是假设我想要检查的结论。

    尽管如此,您的链接还是值得一读。感谢您的帮助和总体检查。

  357. utu 说:
    @szopen

    不要担心,也不需要道歉。

    按照下面的波普尔精神,我构建了一系列测试,产生两个具有相似强度的正交因素,因此它们必须对两种不同类型的认知“智力”有解释。我之前已经提出了 res 的这个论点,但在这里我将尝试在数学上更正式一些。

    让我们 B1={X1, X2,….} 是一组测试,其中包含 i=1,…,N 的单个问题测试 Xi。问题 Xi 取自我们能想到的所有智商和认知测试,因此 N 实际上可能非常大。让我们将这组测试应用于 M 个受试者,其中 M 足够大。让我们生成 N 维协方差矩阵 C0。让我们找到C的两个最大特征向量g-因子和s-因子,其中g-因子更强,即g_eigenvalue>s_eigenvalue。让我们 z0=s_特征值/g_特征值。请记住,g 因子和 s 因子是正交的。

    找到具有最高 g 负载(与 g 相关性最高)的 Xj。通过从中删除测试(问题)Xj 来修改测试 B 的组合。得到N-1维协方差矩阵C1并找到其两个最强特征向量并计算z1=s_特征值/g_特征值。

    找到 g 因子负载最大的 Xj,并将其从测试电池 B 中删除。得到N-2维协方差矩阵C2并得到新的比率z2。依此类推,继续这个过程,我们会注意到 z2、z3、z4……将会增加,在某个时刻我们最终会得到两个具有相似强度的因子 g 和 s。这两个因素将导致两种不同类型的智力,因为它们是正交的,所以无法通约。

    • 回复: @szopen
  358. utu 说:
    @Daniel Chieh

    我对欧洲以及欧洲正在发生的事情非常感兴趣,我对幕后真正发生的事情的看法是,其中大部分都是针对德国的。主要问题是孤立德国并防止其与俄罗斯联系。所有事态发展,例如:对俄罗斯的制裁、难民入侵、大众汽车、德意志银行、英国脱欧、加泰罗尼亚、加强波兰以及关于所谓的“Intermarum”的谈判,都可以在削弱德国并阻止其与俄罗斯联系的背景下解读。波兰和匈牙利有一些空间,并且似乎能够满足欧盟的要求,只是因为美国和以色列的支持。这也解释了波兰对俄罗斯的好战态度。

    齐奥美帝国真正的敌人仍然是德国。他们总是会被证明知道自己的位置。无论向犹太组织支付了多少数十亿美元、向以色列赠送了多少艘潜艇,第二次世界大战都不会被宽恕。德国知道,那里没有任何赦免,但第二次世界大战的另一个胜利者和受害者可以给予德国其所寻求的赦免。这是俄罗斯。只有通过俄罗斯,德国才能恢复其伟大和命运。

    政治的秘密?与俄罗斯签订良好的条约。 奥托·冯·俾斯麦

    只有通过德国,俄罗斯才可能成为与美国和中国的权力斗争中的平等伙伴。俄罗斯知道,疲弱的欧洲意味着它要么必须与中国结盟,要么与美国作为次要伙伴。

    • 回复: @Daniel Chieh
    , @Okechukwu
  359. @Okechukwu

    这些是已经拥有中国本科学位的中国研究生。

    那要看他们在中国就读的大学有多好。您可以找到许多拥有学士学位的中国人的例子 一流的 中国大学年纪轻轻就成为北美一流大学的教授。

    但可以肯定的是,造就这些人的是中国的制度,而不是某些先天的倾向。他们的教育体系向他们灌输了大量信息,但他们从未学会如何应用这些信息。

    也许对于非最好和最聪明的人来说是这样,但对于最好和最聪明的人来说肯定不是这样。你说北美的所有中国人都是中国“最好的”,这根本不是事实。

    这些都没有在中国有机地萌芽。磁悬浮技术当然没有,火车或铁路也没有。

    如果让他们自己使用设备,中国人就不会拥有任何电子商务、互联网或计算机。

    这就像说欧洲人无法创新,因为他们没有发明火药一样。中国早在欧洲人发明火药、枪支和大炮之前就发明了这些技术。这些发明并没有在欧洲有机地萌芽。 但任何理智的人都不会说欧洲人在火药武器方面没有创新。 能够大幅改进技术需要技能和智慧。这不是一件容易的事。对于现有技术而言,中国实际上没有理由重新发明轮子,但能够大幅改进这些技术需要创新能力。同样就电子商务平台而言,西方试图效仿的整个当前商业模式基本上都是中国发明的。当然,它不是一项科学发明,但“社交技术”是一个东西,尽管它很难定义。例如,我认为扎克伯格具有创新性,而 Facebook 是社交网络的一项发明,尽管他没有创造任何新技术。

    • 回复: @Daniel Chieh
    , @Okechukwu
  360. @utu

    默克尔当然不会让德国看起来像是欧洲理智全面受损的受害者,或者就此而言,德国是俄罗斯的朋友。无论如何,尽管 szopen 肯定会更清楚,但我觉得波兰宁愿不要过于依赖她的前攻击者。

    • 回复: @utu
  361. Okechukwu 说:
    @utu

    我对欧洲以及欧洲正在发生的事情非常感兴趣,我对幕后真正发生的事情的看法是,其中大部分都是针对德国的。主要问题是孤立德国并防止其与俄罗斯联系。

    我对美国外交政策的很多方面都持不同意见,但这是妄想的胡言乱语。国家逃离俄罗斯,而不是逃离俄罗斯。德国对美国怀有半开玩笑的“仇恨”。但归根结底,他们非常喜欢美国的霸权,他们认为这种霸权是良性的,具有趋同的民主和文化价值观。

    德国知道,那里没有任何赦免,但第二次世界大战的另一个胜利者和受害者可以给予德国其所寻求的赦免。这是俄罗斯。只有通过俄罗斯,德国才能恢复其伟大和命运。

    这真可笑。德国受益于一项名为“东德”的宝贵自然实验。你知道,这个国家射杀试图逃跑的人。虽然今天的俄罗斯并不完全是苏联,但它是苏联的核心和协调者。即使是最热切怀念过去日子的前东德人也不希望回到俄罗斯的轨道。

    • 回复: @utu
  362. utu 说:
    @Daniel Chieh

    波兰不想过于依赖她的前攻击者。

    这是很明显的,但波兰还有其他选择吗?把它的马车搭上齐奥美利坚帝国的车是相当冒险的。 1939年,波兰被英国和法国占领,给波兰带来了惨重的损失。波兰本可以在政治上发挥更微妙的作用,特别是第三帝国非常有兴趣让波兰成为盟友而不是敌人。所有不反对德国的国家,如捷克斯洛伐克、匈牙利、罗马尼亚、保加利亚,在二战后的境况都比波兰好得多,包括(捷克斯洛伐克除外)其犹太人口。

    • 回复: @szopen
  363. res 说:
    @Anon

    Comin 的论文很有趣: https://www.aeaweb.org/articles?id=10.1257/mac.2.3.65
    此链接包括数据(作为 Stata 文件)和包含文本形式数据的附录。

    以他们的衡量标准来看,非洲似乎做得相当不错(尤其是在公元 0 年)。

    按照您的链接显示了一些讨论 http://thealternativehypothesis.org/index.php/2017/01/09/civilization-powered-by-the-west-threatened-by-the-rest/
    但我有兴趣对技术趋势进行进一步思考。

  364. utu 说:
    @Okechukwu

    你对世界的看法是纳米级的。当你让自己超越媒体和大众文化传播的陈词滥调和陈词滥调时,事情会变得更加有趣。

    我所描述的与对 DDR 的怀旧或处于俄罗斯轨道无关。这是关于未来的。俄罗斯和德国就像双星。两个实力相似的伙伴可以共同抵抗中国和美国的统治。德国对所谓的良性美国霸权并不满意。欧盟的诞生并不是为了成为一块切碎的肝脏。

    • 回复: @Okechukwu
  365. Okechukwu 说:
    @utu

    你完全是痴心妄想。向 szopen 询问波兰愿意跨越哪些障碍加入北约。俄罗斯的所有邻国都会这样做。但要么他们没有资格,要么他们害怕俄罗斯的愤怒。例如乌克兰,可能还有白俄罗斯。

    欧盟只是美国全球秩序的附属品,如果你愿意的话,也可以说是初级合作伙伴。我认为你并没有真正了解美国的强大。我指的也不仅仅是军事力量。美国金融市场广阔而深入,使其能够控制整个国际金融体系。没有任何其他国家或国家联盟能够与这种能力相媲美。在最远的地平线之外也看不到任何东西。

    如果美国真的对俄罗斯实施有意义、严厉的制裁,俄罗斯就会屈服。俄罗斯的实力被严格限制在军事领域。它缺乏经济实力(例如,GDP 相当于加利福尼亚州的一半)。它缺乏金融实力,也缺乏文化软实力,而这些都是美国所拥有的。全世界都说英语不是因为英国而是因为美国

  366. szopen 说:
    @utu

    我们会注意到 z2, z3, z4,… 将会增加,在某个时刻我们最终会得到两个具有相似强度的因子 g 和 s

    这不一定是真的(从某种意义上说:对于真实数据来说并非不可避免)。

    由于您的示例是理论示例,为了表明您的策略不一定会为任何电池产生相同强度的因子,请假设我构建了一系列测试,以便我仅选择高 g 负载(> 0.75)的测试,非常接近彼此(同时仍然测量不同类型的智力)并且任何次要因素的负载较低。然后我就实行你的策略。很容易看出,在第二个因素“s”变得与“g”强度相等之前,我会用完所有测试。

    与你的例子类似;有可能但不能保证在电池停止满足我们的初始条件(进行许多不同的测试)之前,第二个因素确实会变得与“g”强度相同。直到有人真正尝试你的实验后我们才知道。也就是说,您最终会得到在原始分析中最初 S 负载较高的测试(例如,仅口头测试)。我们

    现在,这听起来是同义反复,但它是否是同义反复实际上取决于进行此实验的人(并看看我们是否真的会看到“s-loadings”对应于直观上仅利用一种能力或直观上相似的东西) 。我们不会根据分析后许多不同因素的负载来定义“多样化”。我们可以直观地定义“多样化”,说他们应该利用不同的心理能力。

    事实上,我坚信,在新的“g”停止与旧的“g”相关之前,会发生什么;我的信念基于这样一个事实:分析中出现的第二个因素倾向于将测试分组到广泛的直观合理的集合(空间视觉、语言等)中。

  367. szopen 说:
    @utu

    波兰还有其他选择吗?

    您已经提到了毕苏斯基的 Intermarium 梦想。一旦提出“如果不是俄罗斯或德国,那又怎样?”的问题,波兰每一个不亲德的政治家都会开始谈论它(有时使用其他名称,例如“雅盖隆政策”)。

    • 回复: @Daniel Chieh
  368. PandaAtWar 说:
    @utu

    罗夫! 反响好! …没有读过你其他相关的帖子,所以懒惰地把你当作菜鸟,熊猫不好。

    因此,如果我们假设您出于某种原因认为“不同的测试组产生不同的 g 值,而这些 g 值不一定彼此相关”是正确的,那么您试图从那里得出什么结论?

  369. @szopen

    据我所知,维谢格拉德集团已经开始运作,而且它似乎不会遇到欧盟所面临的致命问题。 “新德国人”则少得多。

    • 回复: @szopen
  370. szopen 说:
    @Daniel Chieh

    那是一种幻觉。 V4里有摩擦,利益分歧也很多。而且V4的经济和人口也不是德国可比的。我喜欢 V4exit,然后是某种联盟(不是联盟),但事实上,V4 还不够。

    更不用说目前波兰经济非常依赖德国,尽管中国最近的举措也可能对我们有所帮助😀

    • 回复: @Daniel Chieh
  371. @szopen

    更不用说目前波兰经济非常依赖德国,尽管中国最近的举措也可能对我们有所帮助😀

    呵呵,我的兄弟目前实际上住在克拉科夫,并且正在资助其中一项计划。

  372. Okechukwu 说:
    @random rand

    那要看他们在中国就读的大学有多好。你可以找到很多拥有中国一流大学学士学位的中国人在很小的时候就成为北美一流大学教授的例子。

    我没有做出价值判断。我只是纠正你的说法,指出这些学生是带着中国本科和研究生学位来的。

    也许对于非最好和最聪明的人来说是这样,但对于最好和最聪明的人来说肯定不是这样。你说北美的所有中国人都是中国“最好的”,这根本不是事实。

    不,我根本没这么说。可能有一些农民更优秀、更聪明,但没有机会。该机构是世界上最负盛名的研究中心之一。也许中国派出了他们的A级学者团队,也许他们没有。这是一个腐败的系统,所以谁知道呢。

    这就像说欧洲人无法创新,因为他们没有发明火药一样。

    我完全相信中国发明了火药以及许多其他开创性技术。我总是用现代(而不是古代)技术来表达我的观点。

    同样就电子商务平台而言,西方试图效仿的整个当前商业模式基本上都是中国发明的。

    如果你指的是像阿里巴巴这样的平台,那你就大错特错了。它们当然不是中国发明的。中国唯一发明的可能是为小偷和骗子创造电子商务安全空间的想法。多年来,阿里巴巴一直是全球商业诈骗的最大推动者之一。

    • 回复: @random rand
  373. @Okechukwu

    如果你指的是像阿里巴巴这样的平台,那你就大错特错了。

    微信、微信钱包、支付宝,微信钱包和支付宝融入更广泛的经济。将类似 uber 的软件集成到微信等移动平台中。将各类软件集成到微信中。极其快速高效的送货服务。中国目前基本是无现金社会。您可以使用支付宝或微信支付所有费用。

    • 回复: @Okechukwu
  374. @Daniel Chieh

    我认为这家伙有爸爸的问题。而且他小时候显然也受过很多欺负。有点心疼他。

    • 回复: @Daniel Chieh
  375. Factorize 说:

    该帖子并没有在应该强调的程度上强调非洲可能出现的巨大的智商差异,因为非洲已经达到了相当大的人口密度。

    心理测量研究人员广泛认识到城乡智商差距,尽管肯尼亚存在的智商差距高于预期。 2005年联合国人类发展指数报告显示,内罗毕的人类发展指数与土耳其相当(智商85-90),而肯尼亚图尔卡纳地区的人类发展指数低于任何其他国家(智商60-65?)。 HDI 与 IQ 相关。这表明非洲正在经历一场重要的智商排序过程,正如人们对不断增加的人口密度所期望的那样(参见本线程顶部附近的人口密度数据)。尼日利亚和埃塞俄比亚等其他人口稠密的非洲地区也应该进行类似的进程。

  376. Okechukwu 说:
    @random rand

    微信、微信钱包、支付宝,微信钱包和支付宝融入更广泛的经济。将类似 uber 的软件集成到微信等移动平台中。将各类软件集成到微信中。极其快速高效的送货服务。中国目前基本是无现金社会。您可以使用支付宝或微信支付所有费用。

    但他们只是采用了现有的技术。他们没有发明它们。美国没有像中国这样的中央政府机构可以从高层颁布命令:“你们应该成为一个无现金社会。”私营公司实施这些系统,并以反映客户需求和愿望的方式实施。今天的美国是一个半无现金社会,因为这是人们想要的。美国人对借记卡、信用卡和现金的组合非常满意。我什至有时看到人们写支票。

    你必须考虑威权制度和民主制度之间的巨大差异。例如,中国建设了大量的高铁。这常常被认为是中国在美国签证方面取得优势的证据,而美国并没有建立任何签证。嗯,显然美国有能力和资金建造子弹头列车。但由于多种原因,它们对美国来说并不实用。中国共产党不一定关心实用性,这是垂直独裁权力结构的最大失败之一。这就是中国建造无人居住的城市和无人购物的购物中心的原因。

    • 回复: @random rand
  377. @Okechukwu

    但他们只是采用了现有的技术。他们没有发明它们。

    社交技术没有被采用,因为它在中国发明之前并不存在。老实说,我不知道你在这里想说什么。正如我之前所说,社会技术需要创新,现有技术的改进也需要创新。 1)人们会同意欧洲人发明了先进的火药武器。 2) 人们会同意 facebook 是社交技术的创新,尽管他们使用的技术与 myspace 相同。 3)这些平台是创新,因为世界上没有其他人像中国公司那样应用这项技术。除非你想说欧洲人在火药武器方面没有创新,并且你不认为 Facebook 是一项创新,否则你可以提出你正在提出的论点。但大多数人并不这么认为。

    美国没有像中国这样的中央政府机构可以从高层颁布命令:“你们应该成为一个无现金社会。”

    并不是规定所有人都可以使用微信支付。人们仍然可以根据需要使用现金和信用卡,因为没有商店拒绝使用现金和信用卡。但人们喜欢使用微信和支付宝,因为它们更容易使用,而且微信已经与日常生活融为一体。

    例如,中国建设了大量的高铁。这常常被认为是中国在美国签证方面取得优势的证据,而美国并没有建立任何签证。

    我们不必与美国比较。中国高铁在任何方面都不逊色于日本铁路,而且在很多方面更胜一筹。中国拥有世界上最快的子弹头列车,这意味着他们改进了现有的技术, 这意味着他们有能力利用所获得的技术进行创新、改进和应用。否则,每个国家都可以制造出和中国一样快的火车。如果中国只能采用技术,而没有能力进行创新或应用,那么它永远无法制造出比日本更好的列车。

    • 回复: @Okechukwu
  378. @random rand

    好吧,我想对于那些成功地利用自己的精神病来谋取有利可图的职业生涯的人来说,可以说一些话。

  379. Factorize 说:

    精彩!

    填补离开非洲的人的基因空白将为我们提供一个心理测量时间表。
    这将使我们更好地了解哪些特定力量影响了人类智商的变化。
    智商是在离开非洲后早期(约 50,000 年前)增加的,还是在接近现代时(约 10,000 年前)发生的?我的猜测是它发生在最近。

    他们已经完成了以下 4 名早期人类的完整基因组测序。他们的基因型智商可能是多少?

    https://www.sciencedaily.com/releases/2017/10/171005141759.htm

    • 回复: @Okechukwu
  380. Okechukwu 说:
    @Factorize

    大脑中没有一个部分称为智商。我猜你指的是智力。

    你的言论中隐含着一个荒谬的说法,即在某个时候离开非洲的人比留下来的人变得更聪明。这是一个可爱的理论,但最终却很可笑。你必须考虑到像我和非洲人这样的人,他们并不是本次讨论中最不聪明的人。事实上恰恰相反。

    您似乎假设存在谨慎的遗传分化。但实际上,不同群体之间的原始智力有太多重叠,因此该假设没有任何科学意义。

  381. Okechukwu 说:
    @random rand

    社交技术没有被采用,因为它在中国发明之前并不存在。

    没有。它从互联网刚刚兴起的时候就已经存在了。抱歉,这里没有任何发明或创新。事实上,中国是其中许多技术的较晚采用者。

    但人们喜欢使用微信和支付宝,因为它们更容易使用,而且微信已经与日常生活融为一体。

    好吧,但这些平台已经存在了近 20 年了。您所描述的是采用而不是创新。作为个人,您只需在孟买或内罗毕雇用开发人员即可创建自己的这些系统迭代。

    我们不必与美国比较。中国高铁在任何方面都不逊色于日本铁路,而且在很多方面更胜一筹。

    请记住,日本子弹头列车的存在时间要长得多,中国可以从这一经验中受益。出于同样的原因,许多美国机场与建造较晚的发展中国家的一些机场相比,看起来绝对古老。

    中国拥有世界上最快的子弹头列车,这意味着他们改进了现有技术,这意味着他们有能力创新、改进和应用他们获得的技术

    不必要。由于多种原因,各国并不总是想要最快的火车(或者对于美国来说根本不需要任何火车)。这与超音速航空旅行的情况没有什么不同。更快并不总是更好。对于像法国 SNCF 这样的现有铁路系统来说,这些系统是在技术不太成熟的情况下建造的,花费数十亿美元进行升级和改造可能没有多大意义。当然他们有子弹头列车技术,每个发达国家都有。这很可能是中国采用的相同技术。中国实施这些技术要容易得多,因为它们相对来说是从头开始的。

    • 回复: @random rand
  382. Factorize 说:

    奥克丘库,

    令人着迷的是,现在不仅可以以如此具体的科学方式提出这些问题,而且还可以回答它们。直到最近,这些问题中的许多问题还无法给出明确的答案。古老的基因序列将为我们提供对人类认知进化的深入了解。

    文章中的四个人的完整基因组被测序,并且可以识别他们在人群中发现的与智力相关的单核苷酸多态性(SNP)的基因型。我怀疑他们的智商远低于欧洲人的平均水平。他们所受到的选择性力量可能不足以强烈改变他们的基因。

    此外,当基因型智商确实发生重大变化时,可以合理地预期现实世界的成就应该有明显的指标。最近的一篇文章发现欧洲人的智商在过去几千年中可能有所提高,这与那个时代的发展历史是一致的。

    你对这些早期人类的智力水平的猜测表示怀疑,这当然是正确的。目前的证据表明,多种力量导致认知能力增加或减少。然而,我们最终可能很快就会得到一些答案。

  383. @Okechukwu

    没有。它从互联网刚刚兴起的时候就已经存在了。抱歉,这里没有任何发明或创新。

    是啊,在微信出现之前,哪个国家已经整合了移动平台呢?您认为 Facebook 是一项创新吗?你认为社交科技是一回事吗?

    事实上,中国是其中许多技术的较晚采用者。

    如果后来者击败了世界其他国家,则更能证明中国的创新能力。

    好吧,但这些平台已经存在了近 20 年了。

    如果它们已经存在了 20 年,它们又是什么?比微信领先20年的平台是哪个?

    请记住,日本子弹头列车的存在时间要长得多,中国可以从这一经验中受益。

    并没有改变中国大幅改进现有技术的事实。这种说法与我最初的论点无关,即中国在现有技术的基础上进行了创新。

    不必要。

    如果你真的认为不需要技术创新来研究地球上尚不存在的 501 公里/小时列车,那么我没什么可说的。

    由于多种原因,各国并不总是想要最快的火车(或者对于美国来说根本不需要任何火车)。

    这与我的论点无关。你说中国人不能创新。我只是证明他们可以。也没有证据表明像法国这样的国家确实拥有如此高速的技术。

    当然他们有子弹头列车技术,每个发达国家都有。

    是的,中国的技术更好。

    中国实施这些技术要容易得多,因为它们相对来说是从头开始的。

    中国对现有技术进行了大幅改进,这意味着他们超越了现有技术,而不是简单地实施现有技术。如果中国没有创新能力,那么他们最多只能达到现有技术的能力,而不是对其进行改进。

    尽管有无数证据表明中国人确实具有创新能力,但你似乎执意要说中国人没有创新能力。你试图说所有发达国家都有子弹头列车技术,这与我的论点无关,即中国拥有更好的子弹头列车技术,因为他们在现有技术上进行了创新。你试图说中国的移动平台不是创新,尽管它们显然是创新,因为在微信之前没有任何国家拥有集成的移动平台,而且你没有提供反例。我不知道你为什么要相信中国没有创新能力,因为所有证据都反对这一说法。

    • 回复: @Daniel Chieh
    , @Okechukwu
  384. gda 说:
    @Okechukwu

    继续挖掘——也许你会到达中国(韩国)。

    “撒哈拉以南非洲人最终拒绝了殖民主义,因为在与欧洲人互动时,他们发现欧洲人在认知上并不优越。 他们常常发现情况恰恰相反。”

    “世界上最大的公司正在直奔非洲,以利用那里的智力资本。”

    “他们现在才开始认识到自己确实落后了,他们所谓的抽象思维能力无法想象或预见非洲人几千年来所知道的东西。”

    “你是否意识到,只有少数美国黑人发明和创新的现代技术比整个亚洲加起来还要多?”

    “好吧,这已经是定局了,林恩、拉什顿、布劳恩等人。 都是骗子。”

    谁知道这条线索会提供这么多令人捧腹的台词。

    那位善良的韩国女士已经把你的内脏掏出来让我们所有人看到,并礼貌地退出,但你仍然继续挥舞,即使你在沙滩上流血了。

    • 哈哈: utu
  385. Factorize 说:

    这不是一个悬而未决的大问题吗?也就是说,一旦实现全球主导地位,亚洲人会呆在家里吗?

    亚洲人显然早于欧洲人进行海上航行。在了解了其他人类的生活方式后,他们重新登船回家,再也没有冒险出去过。

    难道不应该担心这种行为会再次发生吗?

    日本近半个世纪以来一直处于人口崩溃状态,但仍未制定更开放的移民政策。

    如果亚洲人继续对承担全球领导责任不感兴趣,并承担由此带来的所有积极和消极后果,那么这一角色可能需要继续由西方或其他国家来填补。

  386. Okechukwu 说:
    @random rand

    是啊,在微信出现之前,哪个国家已经整合了移动平台呢?您认为 Facebook 是一项创新吗?你认为社交科技是一回事吗?

    你是认真的?尝试使用 Google 等众多工具之一。无论如何,这种集成有多可取?也许这就是为什么微信是中国独有的东西。每个人都可以使用它,但只有中国人在使用它。我个人不希望我的所有客户都受一个实体的控制,尤其是位于中国的实体。

    请用微信平息你的恋情。它只是一个社交网络和消息传递服务,全球有数百个,其中许多与微信相比非常有优势。

    如果后来者击败了世界其他国家,则更能证明中国的创新能力。

    那么为什么世界其他地方不使用它呢?也许你认为微信更好,但实际上并不更好。

    并没有改变中国大幅改进现有技术的事实。这种说法与我最初的论点无关,即中国在现有技术的基础上进行了创新。

    不,他们没有。即使没有立即计划引入新的和改进的系统,子弹头列车技术仍在不断审查和改进。每个供应商都设计、建造和测试子弹头列车,这些列车至少与最新的中国车型一样先进,甚至更先进。

    日本磁浮列车试运行时速600公里打破世界速度纪录

    https://www.theguardian.com/world/2015/apr/21/japans-maglev-train-notches-up-new-world-speed-record-in-test-run

    超高速“超级高铁”列车在加利福尼亚州进行测试

    https://www.livescience.com/50936-hyperloop-test-track-california.html

    新型子弹头列车时速 310 英里,是世界上最快的

    https://www.livescience.com/37195-japan-bullet-train-worlds-fastest-maglev-train.html

    布列塔尼-卢瓦尔河地区高速线上的测试列车时速达到 352 公里

    http://www.railjournal.com/index.php/high-speed/test-trains-reach-352km-h-on-bretagne-pays-de-la-loire-high-speed-line.html

    也没有证据表明像法国这样的国家确实拥有如此高速的技术。

    那是妄想。您认为中国首先从哪里窃取了技术?如前所述,除了速度之外,还有其他考虑因素。甚至中国在发生几起事故后也降低了火车速度。如果你想要真正的速度,那就乘坐飞机。您到达那里的速度会快 2 到 3 倍。顺便说一句,这就是在美国很少有人关心这些火车的主要原因。

    是的,中国的技术更好。

    中国的技术肯定不比日本好。日本是发达国家,中国是发展中国家。中国从一开始就利用窃取的技术进行运作。事实上,他们所有的武器系统和航天器都是偷来的俄罗斯设计。自从向西方开放以来,他们也一直在窃取西方技术。

    中国对现有技术进行了大幅改进,这意味着他们超越了现有技术,而不是简单地实施现有技术。

    正如我所证明的,证据表明情况并非如此。中国并不是高铁或磁悬浮技术的领导者。即使没有立即实施的计划,研究和开发仍在各地进行。

    • 回复: @random rand
  387. @Daniel Chieh

    是的,不敢相信我什至打扰了。我刚刚读到他与韩剧 Kumiho 的交流,他声称韩国人生活在石器时代,亚洲没有发明可言,而且……哇。很难与不了解现实的人争论。

    • 回复: @utu
  388. @Okechukwu

    这将是我最后一次回复。

    尝试使用 Google 等众多工具之一。

    谷歌没有类似微信的平台。 Facebook Messenger一直在积极尝试复制微信。所以不行。它们不等于微信。

    请用微信平息你的恋情。它只是一个社交网络和消息传递服务,全球有数百个,其中许多与微信相比非常有优势。

    我不喜欢微信。简单的事实是,世界上没有任何东西可以与之相比。谷歌没有像样的移动平台。 Facebook Messenger 的情况更糟。你没有提供任何反例。

    那么为什么世界其他地方不使用它呢?

    因为微信已经和中国经济融为一体了。

    新型子弹头列车时速 310 英里,是世界上最快的

    旧消息。 http://www.aljazeera.com/news/2017/09/china-unveils-worlds-fastest-bullet-train-170921131029503.html

    日本磁浮列车试运行时速600公里打破世界速度纪录

    那是磁悬浮而不是传统的轨道。中国目前正在研究时速600公里的磁浮列车。

    布列塔尼-卢瓦尔河地区高速线上的测试列车时速达到 352 公里

    很好,但请参阅半岛电视台链接。根据维基百科:“最初的 8 节车厢列车组 [CRH380A] 在试运行期间记录的最高时速为 416.6 公里/小时(258.9 英里/小时)[3],而更长的 16 节车厢列车组暂时保持着世界纪录最快的生产列车时速为 486.1 公里/小时(302.0 英里/小时)。[4]”“最快的列车组 CRH380BL 的最高测试速度为 487.3 公里/小时(302.8 英里/小时)。”

    那是妄想。您认为中国首先从哪里窃取了技术?

    来自火神大概是因为法国火车的测试速度比中国火车的测试速度慢。

    如前所述,除了速度之外,还有其他考虑因素。甚至中国在发生几起事故后也降低了火车速度。

    是的,只不过中国更快的不仅仅是运营速度。中国的测试速度是最快的。

    正如我所证明的,证据表明情况并非如此。

    NOPE。

    • 回复: @random rand
    , @Okechukwu
  389. Factorize 说:

    对牛弹琴,是没用的

    啊,是的,谷歌将其翻译为“弹钢琴是没用的”,这有点晦涩难懂。

    有人提供更好的翻译吗?
    也许“如果没有练习过,最好不要拉小提琴”?

    • 回复: @random rand
    , @res
  390. utu 说:
    @random rand

    很难与不了解现实的人争论。

    他的现实显然受到某种形式的非理性繁荣的影响:

    你必须考虑到像我和非洲人这样的人,他们并不是本次讨论中最不聪明的人。事实上恰恰相反。

  391. Factorize 说:

    utu,我很高兴扶手椅心理测量自我评估的时代即将结束。我怀疑当单靠 SNP 就能给出高质量的智商评估时,很多人都会感到惊讶。

    这应该会在两年内发生。我非常想知道我的基因 g 可能是什么。我会粗略地猜测 115,尽管我不确定这是否准确。

    • 回复: @utu
  392. @random rand

    由于我在原来的回复中犯了一个错误,我将在这里更正它,作为我对您的真正最后回复,我将不厌其烦地发帖。

    新型子弹头列车时速 310 英里,是世界上最快的

    由于这是磁悬浮,我错误地给出了有关传统轨道的链接。这是有关中国磁悬浮列车的链接。 https://www.theverge.com/2016/11/1/13487382/maglev-train-china-crrc-speed-record https://www.thebeijinger.com/blog/2017/09/02/china-debuts-ultra-fast-maglev-train-can-travel-beijing-shanghai-just-15-minutes

    我最初认为中国的传统铁路无论如何都更优越的论点仍然成立。

  393. @Factorize

    对牛弹琴的意思是对牛弹奏古琴(一种中国乐器)。这是一个惯用的表达方式,基本上是说,对无法理解或欣赏你正在做的事情的人做某事是没有用的。它主要用来表示向白痴解释某事是没有用的。是没用的被翻译为“没用”,不是一个惯用表达。整个句子的直译(对牛弹琴,是没用的)是“对牛弹琴是没用的”。

    • 回复: @Anon
  394. Anon • 免责声明 说:
    @random rand

    所以用英语成语来说就是“pearls before swine”。

  395. Factorize 说:

    好一个,阿昂!

    这是智商测试中有时会出现的问题类型之一。
    我想不出一个好的英语对应词,尽管你的选择是正确的。

    希望我们可以做一些秘密行动,让中国人和其他人加入英语作为世界标准语言的行列。也许可以鼓励孩子们认为英语很酷。

    我花了大约一个月的时间来了解日语。他们有大约三种不同的字母,上面有汉字。他们曾多次尝试使语言合理化,但最终还是成功了。

    转向单一的基于语音的语言将对所有人都有好处。

    • 回复: @Daniel Chieh
  396. Okechukwu 说:
    @random rand

    谷歌没有类似微信的平台。

    而且微信没有类似谷歌的平台。我告诉过你,这是你个人喜好的问题。在西方,人们抵制融合。我不希望我的 G-mail 连接到我的 Google 手机、环聊、钱包、Play、YouTube 等。集成并不是一个新颖或创新的想法。它已经存在了很长时间,但遇到了强烈的阻力。

    所以不行。它们不等于微信。

    但微信上的所有评论并不会让你发出这些热情洋溢的宣言。与其他平台一样,它也有优点和缺点。我不会用一百万美元来交换 Skype 或 Whatsapp。我避开 Facebook,所以我无法真正评论它的功能或特性。

    因为微信已经和中国经济融为一体了。

    当然,这使得它完全毫无价值。这就像将谷歌或 Facebook 与财政部整合一样。

    你没有提供任何反例。

    首先证明微信就是你所说的这个革命性技术。它使用的技术已经存在很久了,所有对它的评论都指出其他平台可以更好地解决这些功能。我并不是说它是垃圾。但它与你所声称的相差甚远。

    很好,但请参阅半岛电视台链接。

    我并不怀疑中国有高铁。我对你关于中国在高铁技术方面处于世界领先地位的论点提出异议。再次强调,速度并不是火车系统的首要目标。他们甚至有强制性的速度限制。

    来自火神大概是因为法国火车的测试速度比中国火车的测试速度慢。

    假设法国人正在与中国人竞争速度(值得怀疑),那么中国火车速度更快怎么就等于技术更好呢?谁报告了中国的这些速度?如果是中国政府的,那就不靠谱了。让我问你一个问题。来自法国的报告和来自中国的报告哪个更可信?

    是的,只不过中国更快的不仅仅是运营速度。中国的测试速度是最快的。

    请参阅上述关于来自中国的任何报告的可信度的评论。但中国似乎将高铁视为民族自豪感。似乎没有人关心。总有飞机可以让您更快地到达您想去的地方,而且对环境的影响也小得多。这就是子弹头列车在美国的支持者如此之少的原因。

  397. @Factorize

    疑。

    这种语言与太多的文化和历史联系在一起,以至于无法放弃(注意这句谚语是如何与古代乐器联系在一起的),而且它在紧凑和保持独特的细微差别方面都有优势。在 思想地理在本书中,作者探讨了这样一种观点:语言风格导致了东亚思想典型的整体性、语境重的形式。

  398. utu 说:
    @Factorize

    这应该会在两年内发生。我非常想知道我的基因 g 可能是什么。

    不要太急于向他们提供 SNP 进行分析。谁知道他们还会发现什么?如果您获得一张前往安乐死中心的单程票怎么办?

  399. Factorize 说:

    utu,是的,你的观点确实有道理。

    当我将 DNA 交给各个基因组公司时,我没有想到我们会在 2017 年全面解锁人类基因组。这是我现在关心的问题。如果我们能够立即对它们进行基因分型,然后进行表型分型,世界将会是什么样子?如果有些人错误地认为他们带有羞耻的遗传红字,并且当遗传字母并不代表他们所认为的含义时,他们就过着痛苦的生活怎么办?

    然而,驱使我打开基因自我认识的潘多拉魔盒的更强大的力量是知道我是谁。如果不了解密码,人们就无法确信自己的自我认知反映了客观事实。该代码应该使我们能够更好地将人们的基因型与适当的环境相匹配,从而使人们更加快乐、更加高效。

  400. @utu

    所有形式的智力很大程度上取决于一个因素的观点 g, 从大脑生理学、解剖学和发育的角度来看,这似乎非常难以置信。

    大脑是一个复杂的器官——非常复杂。据说大脑功能依赖于八十多种神经递质,每种神经递质都有自己的生物合成途径,可能还有自己的分泌、再摄取和分解代谢机制。大脑的每个部分和每个大脑功能同样依赖于所有这些神经递质的可能性有多大?我想说,不太可能,在这种情况下,有理由假设不同心理设施的速度和力量不会很好地相关,反映了不同心理过程的生理基础中由基因决定的差异。

    心理活动被划分在大脑区域和结构中,这些区域和结构在个体之间表现出比例差异。因此,也出于这个原因,人们会期望不同的心理功能之间缺乏确切的相关性。

    心智能力取决于发育变化,包括与学习相关的神经发生,这意味着即使没有其他导致变异的原因,人们也应该预料到智力各方面的变化取决于文化、教育、动机等。

    诚然,所有心理活动都必须依赖于大脑的一些特征:能量代谢、膜特性、血流等。因此,如果心理能力的各个方面之间存在低水平的相关性,人们不应该感到惊讶,情况似乎如此。

    但推动者所暗示的情报各方面之间的相关性并不低。 g。相反,通过他们谈论的方式 g, 人们理解,不接受他们关于核心地位的论点 g 就是将一个人标记为,嗯,不是很聪明,甚至是无知的人。

    • 回复: @Santoculto
  401. Santoculto 说:
    @CanSpeccy

    从大脑生理学、解剖学和发育的角度来看,所有形式的智力很大程度上取决于一个因素 g 的观点似乎非常难以置信。

    我认为当他们谈论 g 时,他们谈论的是模式识别,模式识别在所有类型的认知中绝对是决定性的。

    但他们将粗略的模式识别与一般的“智力”(认知技能)混为一谈。

    智力和行为的原始结构正是模式识别。

    G 并不是你或我的认知能力的对称程度。

  402. res 说:
    @James Thompson

    我在撰写另一条评论时正在重新访问此主题,想知道伍德利古代 DNA 情报工作是否有任何更新?

当前评论者
说:

发表评论 -


 记得 我的信息为什么?
 电子邮件回复我的评论
$
提交的评论已被许可给 Unz评论 并可以由后者自行决定在其他地方重新发布
在翻译模式下禁用评论
通过RSS订阅此评论主题 通过RSS订阅所有James Thompson的评论